toddlers, infants, newborns

144
A 2-month-old boy is brought to the emergency department for evaluation of cyanosis. He became fussy and sweaty while taking his bottle, and his lips "turned blue" for several minutes during the feed. The infant has had similar episodes during feeding and crying, but these resolved quickly. His temperature is 37 C (98.6 F), blood pressure is 80/513 mm Hg, pulse is 15G/min, and respirations are 55/min_ On examination, he is agitated, cyanotic, and tachypneic. Cardiac auscultation shows a grade 2/6 crescendo-decrescendo systolic ejection murmur at the left upper sternal border and a single S2. He is placed in a knee-chest position immediately. This maneuver will improve this patient's condition predominantly by which of the following mechanisms? A. Decreased pulmonary vascular resistance B. Decreased systemic vascular resistance C. Increased pulmonary vascular resistance D. Increased systemic vascular resistance E Increased systemic venous return 3 4- 5 6 7 8 9 10 11 12 13 14 15 16 17 18 19 20 21 22 23 24 25 25 27 28 29 30 31 Item: 1 of 31 F' Mark Previous Next Lab Values. Notes Calculator . 0 Feedback Suspend End Block

description

health and diseases of kids, newborns, and how to solve it

Transcript of toddlers, infants, newborns

Page 1: toddlers, infants, newborns

A 2-month-old boy is brought to the emergency department for evaluation of cyanosis. He became fussy and sweaty while taking his bottle, and his lips "turned blue" for several minutes during the feed. The infant has had similar episodes during feeding and crying, but these resolved quickly. His temperature is 37 C (98.6 F), blood pressure is 80/513 mm Hg, pulse is 15G/min, and respirations are 55/min_ On examination, he is agitated, cyanotic, and tachypneic. Cardiac auscultation shows a grade 2/6 crescendo-decrescendo systolic ejection murmur at the left upper sternal border and a single S2. He is placed in a knee-chest position immediately. This maneuver will improve this patient's condition predominantly by which of the following mechanisms?

• A. Decreased pulmonary vascular resistance

• B. Decreased systemic vascular resistance

• C. Increased pulmonary vascular resistance

• D. Increased systemic vascular resistance

• E Increased systemic venous return

▪ 3

▪ 4-

5 ▪ 6

▪ 7

▪ 8

▪ 9

• 10

• 11

• 12

• 13

• 14

• 15

• 16

• 17

• 18

• 19

• 20

• 21

• 22

• 23

• 24

• 25

• 25

• 27

• 28

• 29

• 30

• 31

Item: 1 of 31

F' Mark Previous

Next

Lab Values. Notes

Calculator.

0 Feedback Suspend End Block

Emaad
Highlight
Emaad
Highlight
Emaad
Highlight
Emaad
Highlight
Emaad
Highlight
Emaad
Highlight
Page 2: toddlers, infants, newborns

1 2

3

▪ 4-

5 ▪ 6

▪ 7

▪ 8

▪ 9

• 10

• 11

• 12

• 13

• 14

• 15

• 15

• 17

• 18

• 19

• 20

• 21

• 22

• 23

• 24

• 25

• 25

• 27

• 28

• 29

• 30

• 31

se

Item: 2 of 31

F' Mark Previous

Next

Lab Values. Notes

Calculator.

An 8-year-old girl is brought to the emergency department with fever for the past days. Her parents report that she has been very "cranky" and has developed a new rash_ Six weeks ago, she completed a 1B-day course of amoxicillin for streptococcal pharyngitis. She has no other medical problems and takes no medications. Her temperature is 39A° C (1D3° F), blood pressure is 110/60 mm Hg, pulse is 120/min, and respirations are 24/min. Physical examination shows injected lips and pharynx. Bilateral conjunctivae are also injected, with no exudates. A 1.7-cm mobile lymph node is palpated on the left neck. A blanching erythematous rash is present across her face, trunk, and extremities, including the palms and soles. Her mouth is shown below.

What is the most appropriate next step in management of this patient?

FC.) A. Amoxicillin

0 Feedback Suspend End Block

Emaad
Highlight
Emaad
Highlight
Emaad
Highlight
Emaad
Highlight
Emaad
Highlight
Emaad
Highlight
Emaad
Highlight
Emaad
Highlight
Page 3: toddlers, infants, newborns

1 2

3

▪ 4-

5 ▪ 6

▪ 7

▪ 8

▪ 9

• 10

• 11

• 12

• 13

• 14

• 15

• 15

• 17

• 18

• 19

• 20

• 21

• 22

• 23

• 24

• 25

• 25

• 27

• 28

• 29

• 30

• 31

Item: 2 of 31

F' Mark Previous

Next . Lab Values. Notes

Calculator.

erythematous rash is present across her face, trunk, and extremities, including the palms and soles. H- er mouth is shown below.

What is the most appropriate next step in management of this patient?

▪ A. Amoxicillin

• B. Aspirin and intravenous immunoglobulin

• C. Doxycycline

• D. Lymph node biopsy

• E. Reassurance and close follow-up

▪ Skin biopsy

• G. Throat culture

0 Feedback Suspend End Block

Page 4: toddlers, infants, newborns

Item: 3 of 31 V-• Mark Previous Next

Lab Values. Notes Calculator,

A 16-year-old female runner comes to the physician with excessive facial hair. She has used multiple creams and makeup over the last year but still considers the facial hair embarrassing_ She also complains of irregular menstrual cycles since menarche at age 12. The patient's last menstrual period was 4 weeks ago. Her blood pressure is 122/70 mm Hg and pulse is 87/min. Her body mass index is 31 kg/m2. Physical examination confirms hirsutism. The patient has normal female external genitalia, and the rest of the examination is unremarkable. Urine pregnancy test is negative. Laboratory results are as follows:

Serum luteinizing hormone

20 IU/L (0.5-15 IU/L) Serum follicle-stimulating hormone

10 lUiL (1-9 IU/L)

17-hydroxyprogesterone

350 ngimL (Normal <15 ng/mL) Serum testosterone

80 ng/mL (Normal 20-75 ngldL) Serum DHEA-S

390 ngimL (Normal 75-370 pgidL)

Serum glucose and electrolytes are normal. Which of the following is the most likely diagnosis?

• A. Adrenal carcinoma

O B. Congenital adrenal hyperplasia

O C. Cushing's syndrome

O ID. Germ cell tumor

• E Idiopathic hirsutism

▪ Polycystic ovarian disease

O G. Use of performance-enhancing agents

1111

IE

a Feedback . Suspend

End Block

• 1 2

3

4-

5

6

▪ 7

▪ 8

▪ 9

• 10

• 11

• 12

• 13

• 14

• 15

• 16

• 17

• 18

• 19

• 20

• 21

• 22

• 23

• 24

• 25

• 25

• 27

• 28

• 29

• 30

• 31

Emaad
Highlight
Emaad
Highlight
Emaad
Highlight
Emaad
Highlight
Emaad
Highlight
Emaad
Highlight
Emaad
Highlight
Emaad
Highlight
Page 5: toddlers, infants, newborns

Item: 4 of 31

.11V-Mark Previous

Next

Lab Values. Notes

Calculator.

• 1

• 2

3 4

5

▪ 5

• 7

▪ 8 ▪ 9

• 10

• 11

• 12

• 13

• 14

• 15

• 15

• 17

• 18

• 19

• 20

• 21

• 22

• 23

• 24

• 25

• 25

• 27

• 28

• 29

• 30

• 31

A 5-year-old boy is brought to the emergency department for a worsening cough. Two weeks ago, he developed rhinorrhea, congestion, and a mild cough_ Since then, the cough has worsened to the point that he often vomits after coughing. The cough is worse at night, during exercise, and with showering. He has taken over-the-counter antitussive medications with no relief. The boy is on a delayed immunization schedule per parental preference and has received the measles-mumps-rubella vaccination. His temperature is 37.2 C (99 F), blood pressure is 90/6G mm Hg, pulse is 98/min, and respirations are 20/min. Pulse oximetry shows an oxygen saturation of 98%_ Physical examination shows an alert and well-appearing child. He has bilateral subconjunctival hemorrhages and periorbital petechiae. The lungs are clear to auscultation. Which organism is the most likely cause of this child's illness?

• A. Adenovirus

• B. Bordeteffa pertussis

• C. Chlarnydophila pneumoniae

• D. Mycobacterium tuberculosis

• E. Mycoplasma ptieumorriae

• Nontypeable 0-faemophilus irrfluerizae

• G. Parainfluenza virus

• H. Respiratory syncytial virus

• I. Streptococcus pneumoniae

0 Feedback Suspend End Block

Emaad
Highlight
Emaad
Highlight
Emaad
Highlight
Emaad
Highlight
Emaad
Highlight
Page 6: toddlers, infants, newborns

• 1

• 2

▪ 3

4

5 6

▪ 7

▪ 8

▪ 9

• 10

• 11

• 12

• 13

• 14

• 15

• 16

• 17

• 18

• 19

• 20

• 21

• 22

• 23

• 24

• 25

• 26

• 27

• 28

• 29

• 30

• 31

Item: 5 of 31

F' Mark Previous

Next

Lab Values. Notes

Calculator.

A 5-year-old girl is brought to the physician for "wetting the bed." Since age 3, she has been completely toilet-trained during the day and at night However, for the past 2 weeks, the girl has wet the bed nightly and has slept poorly from trying to make it to the bathroom multiple times each night. She is also more thirsty than usual, urinating more often during the day, and having "accidents" at school. Review of systems is positive for fatigue and "crankiness" and negative for dysuria, hesitancy, and urgency_ She has had no major illnesses and has met all developmental milestones. One month ago, the family moved to a new house after her brother was born. Vital signs are normal. Weight and height are at the 25th percentile. Physical examination shows a tired-appearing girl. Mucous membranes are dry_ The rest of the examination is unremarkable. Which of the following is the most likely explanation for this patient's symptoms?

• A. Autoimmune destruction of the pancreatic beta cells

• B. Bacterial infection of the bladder

• C. Behavior regression from changes in the home environment

• D. Delayed maturation of sphincter control

• E. Impaired arousal during sleep

F. Impaired renal tubule response to antidiuretic hormone

a

0 Feedback Suspend End Block

Emaad
Highlight
Emaad
Highlight
Emaad
Highlight
Emaad
Highlight
Emaad
Highlight
Emaad
Highlight
Emaad
Highlight
Emaad
Highlight
Emaad
Highlight
Page 7: toddlers, infants, newborns

Item: 6 of 31

F' Mark Previous

Next . Lab Values. Notes

Calculator.

• 1 • 2

• a 4

5

A 1-day-old full-term boy is in the neonatal intensive care unit with cyanosis. His blood pressure is 80/40 mm Hg in all 4 extremities, heart rate is 140/min, and respirations are 55/min_ Pulse oximetry shows 80% and does not improve with 100% inspired oxygen by face mask. He is breathing comfortably, but his fingertips and oral mucosa are blue. A continuous machine-like murmur is heard on auscultation. Chest x-ray shows clear lung fields bilaterally. Which of the following is the best next step in management of this patient?

• A. Furosemide

• B. Intubation with 100% Fiat

• C. Indomethacin

• ID. Propranolol

• E Prostaglandin El

• Red blood cell transfusion

6

7

▪ 8

▪ 9

• 10

• 11

• 12

• 13

• 14

• 15

• 16

• 17

• 18

• 19

• 20

• 21

• 22

• 23

• 24

• 25

• 26

• 27

• 28

• 29

• 30

• 31

0 Feedback Suspend End Block

Emaad
Highlight
Emaad
Highlight
Emaad
Highlight
Emaad
Highlight
Emaad
Highlight
Emaad
Highlight
Emaad
Highlight
Page 8: toddlers, infants, newborns

• 1

• 2

3

• 4

▪ 5

6

7

8

▪ 9

• 10

• 11

• 12

• 13

• 14

• 15

• 15

• 17

• 18

• 19

• 20

• 21

• 22

• 23

• 24

• 25

• 25

• 27

• 28

• 29

• 30

• 31

Item: 7 of 31

V-Mark Previous

Next

Lab Values. Notes

Calculator.

A 17-year-old female is brought to the primary care physician by her mother, who is concerned because her daughter has lost 10 lbs (4_5 kg) in the last two months. The mother says that her daughter has suddenly become a "picky eater" since starting college and that she often refuses to eat now, insisting that she is "not hungry.' Friends of the girl have confided that she is behaving strangely at school. When questioned about these concerns, the girl denies having any problems and says that she feels just fine_ Her height is 5'2" (157.5 cm) and her weight is 100 lbs (45 kg). Physical examination reveals erythema of her turbinates and nasal septum. Which of the following is the most likely diagnosis?

• A. Anorexia nervosa

• B. Bulimia nervosa

• C. Normal adolescence

• D. Cocaine abuse

• E Eating disorder, not otherwise specified

• F. Marijuana abuse

0 Feedback Suspend End Block

Emaad
Highlight
Emaad
Highlight
Emaad
Highlight
Emaad
Highlight
Emaad
Highlight
Page 9: toddlers, infants, newborns

1

A 2-year-old child is brought by an anxious mother to the emergency department because of difficulty in breathing for the past 1 hours. The child has a history of runny nose that started 3 days ago. He suddenly developed a progressively increasing difficulty in breathing. The mother reports that the child "sounds like a barking dog when he coughs." The child's temperature is 37.7° C (99.9° F), respirations are 301min, blood pressure is 9016C[mm Hg, and pulse rate is 1041min. He is coughing and using his accessory muscles of respiration. His oxygen saturation is 98% on 2L of oxygen. He is diagnosed with laryngotracheobronchitis, started on humidified oxygen, and kept in an upright position_ After 20 minutes, he becomes more irritable, his oxygen saturation drops to 92%, respirations increase to 401min, and pulse increases to 120/min. His blood pressure and temperature are unchanged_ What is the most appropriate next step in the management of this patient?

• A. Continue the same therapy

• B. Take the patient to the operating room

• C. Consult the anesthesiologist

• ID_ Intubate the patient in the emergency room

• E Start racemic epinephrine

2

▪ 3

▪ 4-

▪ 5

▪ 6

7

a 9

• 10

• 11

• 12

• 13

• 14

• 15

• 15

• 17

• 18

• 19

• 20

• 21

• 22

• 23

• 24

• 25

• 25

• 27

• 28

• 29

• 30

• 31

Item: 8 of 31

A V-Mark Previous

Next

Lab Values. Notes

Calculator.

0 Feedback Suspend End Block

Emaad
Highlight
Emaad
Highlight
Emaad
Highlight
Emaad
Highlight
Emaad
Highlight
Emaad
Highlight
Emaad
Highlight
Emaad
Highlight
Emaad
Highlight
Page 10: toddlers, infants, newborns

1 2

▪ 3

▪ 4-

▪ 5

▪ 6

▪ 7

• 8

9

• 10

• 11

• 12

• 13

• 14

• 15

• 15

• 17

• 18

• 19

• 20

• 21

• 22

• 23

• 24

• 25

• 25

• 27

• 28

• 29

• 30

• 31

Item: 9 of 31

F'Mark Previous

Next

Lab Values. Notes

Calculator.

A 6-year-old African-American child is brought in by his father for complaints of easy fatigability and pallor_ These symptoms occurred after the son was treated with "some medication" for a recent diarrhea Physical examination is normal except for pallor and multiple petechiae. Laboratory values are as follows:

Hb 8.0 gidL WBC 12,000icmm Platelets 50,000Icmm Blood glucose 118 mg/dL Serum Na 135 mEq!L Serum K 5.3 mEq!L Chloride 110 mEq!L Bicarbonate 18 mEq!L BUN 38 mg/dL Serum creatinine 2.5 mg/dL Total bilirubin 3 mg/dL Direct bilirubin 0.5 mgfdL PT 12 seconds APTT 30 seconds LDH 900 IUiL Reticulocyte count 6%

A peripheral blood smear reveals giant platelets and multiple schistocvtes. What is the most likely underlying pathophysiology for this boy's pallor?

• A. Sickle cell anemia

• B. Thalassemia

▪ C. Vitamin B12 deficiency

▪ ID. Folate deficiency

• E Microangiopathic hemolytic anemia

• F. Lead poisoning

• G. Disseminated intravascular coagulation

▪ H. Idiopathic thrombocytopenic purpura

• I. Renal failure

0 Feedback Suspend End Block

Emaad
Highlight
Emaad
Highlight
Emaad
Highlight
Emaad
Highlight
Emaad
Highlight
Page 11: toddlers, infants, newborns

1

An 8-year-old boy is brought to the emergency department by his mother. Two weeks ago, he developed a low-grade fever followed by a persistent cough. He occasionally has severe paroxysms of cough that are precipitated by eating and do not resolve with antitussive medications. On physical examination, extensive subcutaneous emphysema over the anterior chest is noted. What is the most appropriate next step in the management of this patient?

• A. Chest x-ray

• B. Throat culture

• C. Ear examination

• D. CT scan of head

• E Blood cultures

2

▪ 3

▪ 4-

▪ 5

▪ 6

7

8

9

I 10

• 11

• 12

• 13

• 14

• 15

• 15

• 17

• 18

• 19

• 20

• 21

• 22

• 23

• 24

• 25

• 25

• 27

• 28

• 29

• 30

• 31

Item: 10 of 31

V-Mark Previous

Next

Lab Values. Notes

Calculator.

0 Feedback Suspend End Block

Emaad
Highlight
Emaad
Highlight
Emaad
Highlight
Emaad
Highlight
Page 12: toddlers, infants, newborns

1 2

▪ 3

▪ 4-

▪ 5

▪ 6

▪ 7

• 8

• 9

• 10

• 11

• 12

13

14

15

15

17

18

19

20

21

22

23

24

25

25

27

28

29

30

31

Item: 11 of 31 V-• Mark Previous Next

Lab Values. Notes Calculator,

A 6-year-old girl is brought to the physician with a rash and joint pains. She had previously been healthy except for a sore throat a few weeks ago that resolved on its own. Last week, the patient developed pain in her knees. The pain resolved after a few days, but now her ankles and wrists are tender. She has also developed a non-pruritic pink rash on her back. Temperature is 38.3 C(1 C1 F), pulse is 85/min, and respirations are 2G/min. On examination, there is pain and stiffness with manipulation of the wrists and ankles_ A faint, erythemataus, centrifugal rash is present on her trunk and proximal limbs. Lungs are clear and no murmur is heard_ Serum laboratory results are as follows:

Leukocytes

6,500/p L Neutrophils

56% Lymphocytes

33% Hemoglobin

12_5 WI:IL Platelets

200,004 L C-reactive protein

35 higicIL Erythrocyte sedimentation rate

38 hirnihr

Which of the following is the most likely diagnosis?

O A. Acute lymphoblastic leukemia

• 13. Acute rheumatic fever

O C. Juvenile idiopathic arthritis

O ID. Henoch-Schbnlein purpura

▪ E Systemic lupus erythematosus

IE a

Feedback. Suspend

End Block

Emaad
Highlight
Emaad
Highlight
Emaad
Highlight
Emaad
Highlight
Emaad
Highlight
Emaad
Highlight
Emaad
Highlight
Page 13: toddlers, infants, newborns

Item: 11 of 31 .111 Mark

EF. 11.

Previous

Next

Lab Values

Notes Calculator

Media Exhibit

▪ 2

▪ 3

▪ 4-

▪ 5

▪ 6

▪ 7

▪ 8

▪ 9

• 10

• 12

• 13

• 14

• 15

• 16

• 17

• -18

• 19

• 20

• 21

• 22

• 23

• 24

• 25

• 26

• 27

• 28

• 29

• 30

• 31

Media 1 of 1

0 Feedback Suspend End Block

Page 14: toddlers, infants, newborns

1

An 8-month-old child is referred by a nurse practitioner due to "pale mucous membranes, irritability, and listlessness." The stool examination is negative for occult blood, ova and parasites. Laboratory studies reveal:

Hemoglobin 6.G WI_ MCHC 25% MCH 16.5 pg MCV 68 fl Reticulocytes 0.6% Platelets 230,888im m3 Leukocyte count 5,5081m m3

Neutrophils 56% Eosinophils 1% Lymphocytes 33% Monocytes 10%

40 mcg/dL 460 mcgldL (normal 300-350 magidL)

Percent saturation of transferrin Total serum bilirubin

The peripheral blood smear shows marked anisocytosis, microcytosis, hypochromia, and poikilocytosis. Which of the following is the most likely diagnosis?

O A. Iron deficiency anemia

O B. Sideroblastic anemia

C. Dimorphic anemia

• D. Megaloblastic anemia

O E Anemia of chronic disease

2

▪ 3

▪ 4-

▪ 5

▪ 6

▪ 7

▪ 8

▪ 9

• 10

• 11

• 12 • 13

• 14

• 15

• 16

• 17

• 18

• 19

• 20

• 21

• 22

• 23

• 24

• 25

• 26

• 27

• 28

• 29

• 30

• 31

Item: 12 of 31

V-• Mark Previous Next

Lab Values. Notes Calculator.

Serum Iron

TIRC

8.7%

0.9 mg/dL

0 Feedback Suspend End Block

Emaad
Highlight
Emaad
Highlight
Emaad
Highlight
Page 15: toddlers, infants, newborns

1 2

▪ 3

▪ 4-

▪ 5

▪ 6

▪ 7

▪ 8

▪ 9

• 10

• 11

• 12

13

- 14

• 15

• 16

- 17

• 18

• 19

• 20

• 21

• 22

• 23

• 24

• 25

• 26

• 27

• 28

• 29

• 30

• 31

A 14-year-old African American girl comes to the emergency department with a low-grade fever and severe right-sided thigh pain. Her pain started yesterday and is worsening despite ibuprofen and oxycodone. She is agitated and requests quick pain relief. The patient has had similar episodes every summer that required hospitalization for intravenous pain management, but she has not received blood transfusions. Her vaccinations are up to date and she takes an iron-free multivitamin. Her brother had similar pain episodes and died of a bloodstream infection at age 4. Her temperature is 38 C (1014 F), blood pressure is 120/8G mm Hg, pulse is 120/min, and respirations are 18/min. Which of the following would most likely be seen on this patient's peripheral blood smear?

• A. Blast cells

• B. Burr cells

• C. Eosinophilia

• ID. Howell-Jolly bodies

• E. Pappenheimer bodies

• F. Thrombocytopenia

Item: 13 of 31

F' Mark Previous

Next

Lab Values. Notes

Calculator.

0 Feedback Suspend End Block

Emaad
Highlight
Emaad
Highlight
Emaad
Highlight
Emaad
Highlight
Emaad
Highlight
Emaad
Highlight
Page 16: toddlers, infants, newborns

2

▪ 3

▪ 4-

▪ 5

▪ 6

▪ 7

▪ 8 ▪ 9

• 10

• 11

• 12

• 13

1

A 4-week-old full-term boy is brought to the emergency department with persistent, forceful vomiting immediately after feeds for the past several days. The vomitus is always nonbloody and nonbilious. Vital signs show tachycardia, hypotension, and bradypnea. Examination shows a thin infant with a sunken anterior fontanel and dry mucous membranes. His abdomen is soft, nontender, and nondistended. An olive-shaped mass is palpated in the right upper quadrant Which of the following laboratory findings would be expected in this patient?

pH Pa 02 HCO3 Ft I

Item: 14 of 31

V/lark Previous

Next

Lab Values. Notes

Calculator.

• 14

• 15

• 16

• 17

• 18

• 19

• 20

• 21

• 22

• 23

• 24

• 25

• 26

• 27

• 28

• 29

• 30

• 31

0 A. 1 -i. 1 4. 1

O B- i t I Normal Normal

O C. Normal Normal Normal Normal Normal

O a t t 1 1 1

O E. I I I I I

0 Feedback Suspend End Block

Emaad
Highlight
Emaad
Highlight
Emaad
Highlight
Emaad
Highlight
Emaad
Highlight
Page 17: toddlers, infants, newborns

2

▪ 3

▪ 4-

▪ 5

▪ 6

▪ 7

▪ 8

▪ 9

• 10

• 11

• 12

• 13

• 14

15 • 16

• 17

• 18

• 19

• 20

• 21

• 22

• 23

• 24 • 25

• 25

• 27

• 28

• 29

• 30

• 31

1

An 11-year-old boy is brought to the physician for evaluation of scoliosis. His mother noticed that he always seems to be leaning even when he tries to sit or stand upright The boy has a history of myopia and upward lens dislocation for which he wears corrective glasses. He otherwise has been healthy and doing well in school_ His father had scoliosis and vision problems and died from "heart problems" last year. Physical examination shows a boy with a long face, high arched palate with crowded teeth, and upward dislocation of the lens_ He has a tall stature for his age; long arms and legs with minimal subcutaneous fat; long, thin fingers; and a prominent sternum. Joint hypermobility, skin hyperelasticity, and 15 degrees of thoracic scoliosis are seen. A diastolic murmur is heard in the aortic area Which of the following is the most likely etiology of this patient's condition?

• A. Cystathionine synthase deficiency

• B. Defective collagen production

• C. Mutation of the fibrillin-1 gene

• ID. Mutation of the fibrillin-2 gene

• E Nondisjunction resulting in an extra X chromosome

Item: 15 of 31

F' Mark Previous

Next

Lab Values. Notes

Calculator.

0 Feedback Suspend End Block

Emaad
Highlight
Emaad
Highlight
Emaad
Highlight
Emaad
Highlight
Emaad
Highlight
Emaad
Highlight
Emaad
Highlight
Emaad
Highlight
Emaad
Highlight
Emaad
Highlight
Page 18: toddlers, infants, newborns

Notes

Calculator.

0 Suspend End Block

se

1 2

3

4-

5

6

7

8 9

• 10

• 11

• 12

• 13

• 14

• 15

Item: 16 of 31 F' Mark Previous Next Lab Values.

A 6-year-old boy is brought to the physician by his mother with abdominal pain that began 4 days earlier. The pain is diffuse and associated with nausea The boy has had no fever, vomiting, or changes in bowel habits. He had a rash that was initially erythematous and macular but has now become confluent. Physical examination shows a diffusely tender abdomen without rebound or guarding_ A nonblanching rash is noted on his lower extremities. The patient's right knee is swollen and he complains of pain with passive range of motion. Laboratory results are as follows:

Complete blood count Hemoglobin 13_5 g/dL Platelets 350,0004.iL Leukocytes 9,100/pL

Neutrophils 68% Eosinophils 1% Lymphocytes 25% Monocytes 6%

Serum chemistry Sodium 140 mEq!L Potassium 3.6 mEq/L Chloride 98 mEq/L Bicarbonate 23 mEq/L Blood urea nitrogen 18 mg/dL Creatinine 1_3 mg/dL Calcium 9_2 mg/dL Glucose 118 mg/dL

Urinalysis Specific gravity 1.022 Protein +1 Blood Moderate Glucose Negative Ketones Negative Leukocyte esterase Negative Nitrites Negative

I 16

• 17

• 18

• 19

• 20

• 21

• 22

• 23

• 24

• 25

• 26

• 27

• 28

• 29

• 30

• 31

Feedback

Emaad
Highlight
Emaad
Highlight
Emaad
Highlight
Emaad
Highlight
Emaad
Highlight
Emaad
Highlight
Page 19: toddlers, infants, newborns

Item: 16 of 31 F' Mark Previous Next

Lab Values. Notes Calculator.

...ill IL

Hemoglobin

13_5 g/dL Platelets

350,000/p L Leukocytes

9,1134L Neutrophils

68% Eosinophils

1% Lymphocytes

25% Monocytes

6%

Serum chemistry Sodium 140 mEq/L Potassium 3_6 mEq/L Chloride 98 mEq/L Bicarbonate 23 mEq/L Blood urea nitrogen 18 mg/dL Creatinine 1.3 mg/dL Calcium 9_2 mg/dL Glucose 118 mg/dL

Urinalysis Specific gravity 1.022 Protein +1 Blood Moderate Glucose Negative Ketones Negative Leukocyte esterase Negative Nitrites Negative

Which of the following glomerular abnormalities is most likely present in this patient?

• A. Glomerular basement membrane thickening

• B. Linear deposition of immunoglobulin G on the basement membrane

• C. Localized areas of mesangial sclerosis and collapse

• ID. Mesangial deposition of immunoglobulin A

• E Podocyte fusion

1 2

▪ 3

▪ 4-

▪ 5

▪ 6

▪ 7

▪ 8 ▪ 9

• 10

• 11

• 12

• 13

• 14

• 15

I 16

• 17

• 18

• 19

• 20

• 21

• 22

• 23

• 24

• 25

• 26

• 27

• 28

• 29

• 30

• 31

0 Feedback Suspend End Block

Emaad
Highlight
Page 20: toddlers, infants, newborns

2

▪ 3

▪ 4-

▪ 5

▪ 6

▪ 7

▪ 8 ▪ 9

• 10

• 11

• 12

• 13

• 14

• 15

• 15

a 17

• 18

• 19

• 20

• 21

• 22

• 23

• 24

• 25

• 25

• 27

• 28

• 29

• 30

• 31

1

A 2 and a half-year-old child is brought to the office for the evaluation of easy bruising, nosebleeds, and decreased activity over the past week. He had an upper respiratory infection that was treated with an antibiotic 2 weeks ago. On examination, he is well-developed, seems well-nourished, anicteric, and pale. Pertinent findings include some small palpable posterior cervical lymph nodes, sinus tachycardia, a grade INI systolic ejection murmur, ecchymoses on his left shoulder and both lower extremities, and petechiae over his extremities and groin. There is no hepatosplenomegaly_ The laboratory findings are as follows:

Hemoglobin gidL Hematocrit 24% Platelet count 12,000imm3 WE C 3,000!mm3 Reticulocyte count 0_5%

A bone marrow biopsy reveal a markedly hypocellular marrow with decreased megakaryocytes and precursors of the erythroid and myeloid cell lines. What is the most likely diagnosis?

• A. Acquired aplastic anemia

• B. Fanconi's anemia

• C. Diamond-Black-fan anemia

• D. Transient erythroblastopenia

• E Acute myeloid leukemia

Item: 17 of 31 .11M Mark Previous

Next

Lab Values. Notes

Calculator.

0 Feedback Suspend End Block

Emaad
Highlight
Emaad
Highlight
Emaad
Highlight
Emaad
Highlight
Emaad
Highlight
Page 21: toddlers, infants, newborns

1

An 11-year-old boy is brought to the office by his mother because "he is sick.' He has had headaches for the past several weeks, and has vomited 4 times in the past 5 days. He drinks large amounts of water and goes to the bathroom all the time. He is no longer interested in playing football and going out with his friends. His temperature is 3L0° C (98.6° F), blood pressure is 118/78 mm Hg, pulse rate is 84/min, and respirations are 16/min. On examination, there is loss of peripheral visual fields. His laboratory findings are as follows:

WE C 7,800/mm3 Hemoglobin 12.6 g/dL Hematocrit 35% Platelets 199,000/mm3 Sodium 145 mEcilL Potassium 3.6 mEcilL Bicarbonate 24 mai/ Blood urea nitrogen 18 mg/dL Serum creatinine 1.0 mg/dL Blood glucose 88 mg/dL

X-rays of the head reveal a calcified lesion above the sells_ What is the most likely diagnosis?

O A. Pituitary adenoma

O B. Meningioma

O C. Empty sells syndrome

• D.Craniopharyngioma

O E. Ependymoma

2

▪ 3

▪ 4-

▪ 5

▪ 6

▪ 7

▪ 8

▪ 9

• 10

• 11

• 12

• 13

• 14

• 15

• 16

• 17

I -18

• 19

• 20

• 21

• 22

• 23

• 24

• 25

• 26

• 27

• 28

• 29

• 30

• 31

Item: 18 of 31

F' Mark Previous

Next

Lab Values. Notes

Calculator.

0 Feedback Suspend End Block

Emaad
Highlight
Emaad
Highlight
Emaad
Highlight
Emaad
Highlight
Emaad
Highlight
Emaad
Highlight
Page 22: toddlers, infants, newborns

2

▪ 3

▪ 4-

▪ 5

▪ 6

▪ 7

▪ 8 ▪ 9

• 10

• 11

• 12

• 13

• 14

• 15

• 15

• 17

• 18

19

• 20

• 21

• 22

• 23

• 24

• 25

• 25

• 27

• 28

• 29

• 30

• 31

1

The parents of a 5-year-old girl bring her to clinic for evaluation. They are frightened because their daughter has started to talk to herself, especially when she is alone in her room. When the parents ask who she is talking to, she says, "Nobody!" and refuses to discuss the matter. She becomes angry if they persist. Recently, after spilling milk during breakfast, the girl denied having done it, insisting that "Mindy" was responsible_ The parents state they have no idea who Mindy is. The girl had some trouble adjusting to kindergarten at the start of the school year, but is now doing well_ She is an only child, has no medical problems, and has reached all of her milestones on time. What should the parents be advised?

Item: 19 of 31

F' Mark Previous

Next

Lab Values. Notes

Calculator.

• A. "Your daughter's behavior is normal for her age, and there is nothing to worry about"

B. "Your daughter should start taking risperidone to decrease the likelihood of her symptoms 1---1 worsening."

• C. "Your daughter may be experiencing mistreatment and abuse in her kindergarten class."

• ID. "Your daughter should begin weekly counseling sessions with the school psychologist"

• E 'Your daughter has oppositional defiant disorder and needs more limit setting before it worsens.'

0 Feedback Suspend End Block

Emaad
Highlight
Emaad
Highlight
Emaad
Highlight
Emaad
Highlight
Emaad
Highlight
Emaad
Highlight
Emaad
Highlight
Page 23: toddlers, infants, newborns

2

▪ 3

▪ 4-

5 ▪ 6

▪ 7

▪ 8 ▪ 9

• 10

• 11

• 12

• 13

• 14

• 15

• 15

• 17

• 18

• 19

• 20

• 21

• 22

• 23

• 24

• 25

• 25

• 27

• 28

• 29

• 30

• 31

1

A 15-year-old boy is brought to the emergency department due to the sudden onset of difficulty breathing for the past 45 minutes. He also complains of nausea, colicky abdominal pain, and a swollen face. The patient has suffered from bronchitis for the past 4 days, and his condition had been improving. His mother says that he had a similar episode when he had a tooth extracted 2 years ago. On examination, there is an edematous swelling of his face including the lips, hands, arms, legs, and genitals without a rash. Temperature is 36.8 C (98.4 F), blood pressure is 12G/80 mm Hg, pulse is 82/min, and respirations are 18/min. Which of the following best explains the pathological process of this patient's condition?

• A. Antibody-mediated hypersensitivity

• B. Cl inhibitor deficiency

• C. Cell-mediated hypersensitivity

• ID. Depressed Cl q

• E Immune complex-mediated hypersensitivity

Item: 20 of 31

V-Mark Previous

Next

Lab Values. Notes

Calculator.

0 Feedback Suspend End Block

Emaad
Highlight
Emaad
Highlight
Emaad
Highlight
Emaad
Highlight
Emaad
Highlight
Emaad
Highlight
Page 24: toddlers, infants, newborns

1 2

▪ 3

▪ 4-

5 ▪ 6

▪ 7

▪ 8 ▪ 9

• 10

• 11

• 12

• 13

• 14

• 15

• 15

• 17

• -18

• 19

• 20

• 21

• 22

• 23

• 24

• 25

• 25

• 27

• 28

• 29

• 30

• 31

se

A 3-year-old boy from a refugee camp is brought to the physician for evaluation of a rash_ He recently came to the United States and his medical history is unknown. The rash has been present for a few weeks and seems to be spreading. He is hungry "all the time" and his family had limited access to food in their home country. Review of systems is negative for nausea, vomiting, diarrhea, headaches, numbness, or tingling. The boy takes no medications and has no known allergies_ His weight and length are <5th percentile for age and sex_ Examination shows a malnourished boy with minimal subcutaneous fat Scaling and fissures are present at the mouth corners and his lips are cracked and inflamed. The patient's tongue and oropharyngeal mucous membranes are swollen and hyperemic. There are erythematous scaly patches on his eyebrows, cheeks, and nose_ The rash is also present on the scrotal skin and extends to the medial aspect of both thighs. The skin and conjunctivae are pale and his fingers and toenails are brittle. Laboratory results are as follows:

Complete blood count Hemoglobin 9 gidL Mean corpuscular volume 82 fL Platelets 180,000/pL Leukocytes 7,500/pL

Serum chemistry Sodium 136 mEq!L Potassium 3.8 mEq/L Chloride 1DD mEq!L Bicarbonate 26 mEq!L Blood urea nitrogen 9 mg/I:IL Creatinine 12 mgldL Glucose 73 mg/I:IL

Which of the following is the most likely cause of this patient's condition?

• A. Vitamin A toxicity

• B. Vitamin 131 (thiamine) deficiency

• C. Vitamin B2 (riboflavin) deficiency

• ID. Vitamin (niacin) deficiency

Item: 21 of 31 F' Mark Previous Next

Lab Values. Notes Calculator.

0 Feedback Suspend End Block

Emaad
Highlight
Emaad
Highlight
Emaad
Highlight
Emaad
Highlight
Emaad
Highlight
Emaad
Highlight
Emaad
Highlight
Emaad
Highlight
Emaad
Highlight
Emaad
Highlight
Emaad
Highlight
Page 25: toddlers, infants, newborns

1 2

▪ 3

▪ 4-

5 ▪ 6

▪ 7

▪ 8 ▪ 9

• 10

• 11

• 12

• 13

• 14

• 15

• 15

• 17

• -18

• 19

• 20

• 21

• 22

• 23

• 24

• 25

• 25

• 27

• 28

• 29

• 30

• 31

Item: 21 of 31

al V- • Mark -4Z1

Previous Next

Lab Values. Notes Calculator.

seems to be spreading. He is hungry all the time" and his tamely had limited access to toad in their home country. Review of systems is negative for nausea, vomiting, diarrhea, headaches, numbness, or tingling. The boy takes no medications and has no known allergies_ His weight and length are <5th percentile for age and sex. Examination shows a malnourished boy with minimal subcutaneous fat. Scaling and fissures are present at the mouth corners and his lips are cracked and inflamed. The patient's tongue and oropharyngeal mucous membranes are swollen and hyperemic. There are erythematous scaly patches on his eyebrows, cheeks, and nose_ The rash is also present on the scrotal skin and extends to the medial aspect of both thighs. The skin and conjunctivae are pale and his fingers and toenails are brittle_ Laboratory results are as follows:

Complete blood count Hemoglobin 9 gidL Mean corpuscular volume 82 fL Platelets 180,000/p L Leukocytes 7,500/pL

Serum chemistry Sodium 136 mEcilL Potassium 3.8 mEcilL Chloride 1GD mEcilL Bicarbonate 26 mEdIL Blood urea nitrogen 9 mg/dL Creatinine 0.2mgldL Glucose 73 mg/I:IL

Which of the following is the most likely cause of this patient's condition?

O A. Vitamin A toxicity

O B. Vitamin Bi (thiamine) deficiency

C. Vitamin Bi (riboflavin) deficiency

• ID. Vitamin 13.., (niacin) deficiency

• E Vitamin B6 (pyridoxine) toxicity

• Vitamin C deficiency

O G. Vitamin K deficiency

0 Feedback Suspend End Block

Page 26: toddlers, infants, newborns

2

▪ 3

▪ 4-

5 ▪ 6

▪ 7

▪ 8

▪ 9

• 10

• 11

• 12

• 13

• 14

• 15

• 15

• 17

• 18

• 19

• 20

• 21

• 22

▪ 23

• 24

▪ 25

• 25

• 27

• 28

• 29

• 30

• 31

1

A 12-year-old African American male with known sickle cell disease presents with a 2 hour history of right-sided arm weakness and slurred speech. He has been hospitalized before for pain crises and pneumonia. He takes hydroxyurea, oxycodone as needed, and folic acid. His temperature is 98.0°F (36.6'C), blood pressure is 153/83 mmHg and heart rate is 11 /min with regular rhythm_ an physical examination, he has right arm weakness and mild dysarthria. His laboratory values are the following:

Hemoglobin 8.2 mg/dL

WBC count 14,000/me

Platelet count 210,000/me Creatinine mg/dL

CT of the head shows no evidence of intracranial bleeding_ Which of the following is the best initial management for this patient?

• A. Beta-blockers and aspirin

• B. Exchange transfusion

• C. Fibrinolytic therapy

• Heparin and warfarin

• E Plasmapheresis

Item: 22 of 31

V-Mark Previous

Next

Lab Values. Notes

Calculator.

0 Feedback Suspend End Block

Emaad
Highlight
Emaad
Highlight
Emaad
Highlight
Emaad
Highlight
Emaad
Highlight
Emaad
Highlight
Page 27: toddlers, infants, newborns

se

2

▪ 3

▪ 4-

5 ▪ 6

▪ 7

▪ 8 ▪ 9

• 10

• 11

• 12

• 13

• 14

• 15

• 15

• 17

• 18

• 19

• 20

• 21

22

23

• 24

• 25

• 25

• 27

• 28

• 29

• 30

• 31

1

A 5-month-old girl is brought to the physician for a routine visit The patient has been evaluated several times for poor weight gain. The infant takes 6 ounces of regular formula every 4 hours. Increasing the caloric density of her formula has not improved her growth. The patient has no abdominal pain, nausea, diarrhea, or vomiting. There is a family history of nephrolithiasis_ She was born full term without complications. The patient's birth weight was 3.6 kg (8 Ib, 50th percentile). Weight is <5th percentile: length and head circumference have been tracking along the 25th percentile_ The infant appears thin, but the remainder of the physical examination is unremarkable. Newborn screening results were normal. Laboratory results are as follows:

Serum chemistry Sodium 140 mEq!L Potassium 3 mEq!L Chloride 121 mEq/L Blood urea nitrogen 10 mg/dL Creatinine 0.5 mg/dL Calcium 9 mg/dL Glucose 98 mg/dL

Arterial blood gases pH 7.21 FaCO, 31 mm Hg Bicarbonate 14 mEq!L

Urinalysis pH 7.9 Potassium Normal Sodium Normal

Item: 23 of 31 F' Mark

UP. Previous Next

Lab Values, Notes Calculator.

Which of the following is the most likely cause of this patient's failure to thrive?

• A. Cystic fibrosis

• B. Gastroesophageal reflux

• C. Insufficient caloric intake I

0 Feedback Suspend End Block

Emaad
Highlight
Emaad
Highlight
Emaad
Highlight
Emaad
Highlight
Emaad
Highlight
Emaad
Highlight
Page 28: toddlers, infants, newborns

1 2

▪ 3

▪ 4-

5 ▪ 6

▪ 7

▪ 8

▪ 9

• 10

• 11

• 12

• 13

• 14

• 15

• 15

• 17

• 18

• 19

• 20

• 21

• 22

23

• 24

• 25

• 25

• 27

• 28

• 29

• 30

• 31

Item: 23 of 31 F' Mark Previous Next

Lab Values. Notes Calculator.

A 5-month-old girl is brought to the physician for a routine visit The patient has been evaluated several times for poor weight gain. The infant takes 6 ounces of regular formula every 4 hours. Increasing the caloric density of her formula has not improved her growth. The patient has no abdominal pain, nausea, diarrhea, or vomiting. There is a family history of nephrolithiasis_ She was born full term without complications. The patient's birth weight was 3f kg (8 Ib, 50th percentile). Weight is <5th percentile; length and head circumference have been tracking along the 25th percentile_ The infant appears thin, but the remainder of the physical examination is unremarkable. Newborn screening results were normal. Laboratory results are as follows:

Serum chemistry Sodium 140 mEq!L Potassium 3 mEq/L Chloride 121 mEq/L Blood urea nitrogen 10 mgldL Creatinine 0.5 Calcium 9 mgldL Glucose 98 mgldL

Arterial blood gases pH 721 FaCO, 31 mm Hg Bicarbonate 14 mEcilL

Urinalysis pH 7.9 Potassium Normal Sodium Normal

Which of the following is the most likely cause of this patient's failure to thrive?

• A. Cystic fibrosis

• B. Gastroesophageal reflux

• C. Insufficient caloric intake

• ID. Lactic acidosis

• E Renal tubular acidosis

0 Feedback Suspend End Block

Page 29: toddlers, infants, newborns

2

▪ 3

▪ 4-

5 ▪ 6

▪ 7

▪ 8 ▪ 9

• 10

• 11

• 12

• 13

• 14

• 15

• 15

• 17

• 18

• 19

• 20

• 21

• 22

• 23

& 24

1

An 18-month-old boy is brought to the emergency department an hour after drinking liquid oven cleaner from an unlocked kitchen cabinet His parents tried to give him water and milk, but he has difficulty swallowing. The boy also has blood-tinged oral secretions. His vital signs are stable. Examination shows an anxious child who is crying and drooling. His lips and chin are swollen and erythematous. He has no stridor and his breathing pattern appears normal. Lungs are clear to auscultation. His shirt is covered in oven cleaner. Which of the following is the best next step in management of this patient?

• A. Barium swallow study

• B. Clothing removal

• C. Intravenous corticosteroids

• ID. Nasogastric feeding tube

• E Nasogastric lavage

• F. Neutralization with vinegar

• G. Upper gastrointestinal endoscopy

Item: 24 of 31

V-• Mark Previous

Next

Lab Values. Notes

Calculator.

• 25

• 25

• 27

• 28

• 29

• 30

• 31

0 Feedback Suspend End Block

Emaad
Highlight
Emaad
Highlight
Emaad
Highlight
Emaad
Highlight
Emaad
Highlight
Emaad
Highlight
Emaad
Highlight
Page 30: toddlers, infants, newborns

2

▪ 3

▪ 4-

5 ▪ 6

▪ 7

▪ 8 ▪ 9

• 10

• 11

• 12

• 13

• 14

• 15

• 16

• 17

• 18

• 19

• 20

• 21

• 22

• 23

• 24

25 • 26

• 27

• 28

• 29

• 30

• 31

1

A 1-year-old African American girl is brought to the office for routine evaluation. She is learning how to walk and says "mama" and ''dada." She recently transitioned from breast milk to cow's milk. Review of systems is negative. Both parents have sickle cell trait and her sister has sickle cell disease. Her height and weight are appropriate for her age_ Physical examination shows a well-appearing, well-nourished child with no abnormalities. Laboratory results are as follows:

Complete blood count Hemoglobin 14 g/dL Hematocrit 42% Mean corpuscular volume 88 fL Reticulocytes 2%

Hemoglobin electrophoresis Hemoglobin A 60% Hemoglobin S 40% Hemoglobin F 0%

What is the most common complication of her condition?

Item: 25 of 31 V-Mark Previous Next

Lab Values. Notes Calculator.

• A. Acute chest syndrome

• B. Dactylitis

• C. Hematuria

• D. lschemic stroke

• E. Osteomyelitis

• F. Splenic infarction

• G. Urinary tract infection

0 Feedback Suspend End Block

Emaad
Highlight
Emaad
Highlight
Emaad
Highlight
Emaad
Highlight
Emaad
Highlight
Emaad
Highlight
Page 31: toddlers, infants, newborns

2

▪ 3

▪ 4-

5 ▪ 6

▪ 7

▪ 8

▪ 9

• 10

• 11

• 12

• 13

• 14

• 15

• 16

• 17

• 18

• 19

• 20

• 21

• 22

• 23

• 24

• 25

• 26

1

A 3-year-old boy is brought to the physician for help with toilet training. He recently started day care and screams "no" when teachers try to place him on the toilet He has bowel movements every other day and strains when he passes hard, pellet-like stools. The boy is a picky eater but loves milk and drinks up to 30 oz of chocolate milk daily. He has no medical problems and takes no medications. His weight and height have been tracking along the 75th percentile. Examination shows a cooperative, well-nourished boy. He runs well and can climb onto the examination table independently. He speaks in short sentences that are mostly understandable_ The boy's abdomen is soft, nontender, and nondistended_ He has normal Tanner I male genitalia. A small fissure is noted on the anal verge. Which of the following is the best next step in management of this patient?

• A. Abdominal x-ray

• B. Anorectal manometry

• C. Disimpaction with rectal enema

• D. Increase juice intake

• E. Oral laxative therapy

• F. Rectal biopsy

• G. Thyroid function testing

Item: 26 of 31

F' Mark Previous

Next

Lab Values. Notes

Calculator.

• 27

• 28

• 29

• 30

• 31

0 Feedback Suspend End Block

Emaad
Highlight
Emaad
Highlight
Emaad
Highlight
Emaad
Highlight
Emaad
Highlight
Emaad
Highlight
Emaad
Highlight
Emaad
Highlight
Emaad
Highlight
Page 32: toddlers, infants, newborns

2

▪ 3

▪ 4-

5 ▪ 6

▪ 7

▪ 8

▪ 9

• 10

• 11

• 12

• 13

• 14

• 15

• 16

• 17

• 18

• 19

• 20

• 21

• 22

• 23

• 24

• 25

1

A 12-year-old girl is brought to the clinic due to a 2-month history of headaches. Her headaches last 1 - 2 hours and have no fixed time of occurrence. She denies nausea, vomiting, chills or fevers. She has no other medical problems and takes no medication_ Her family history is significant for hypertension and diabetes. Her blood pressure is 156/90 mm Hg, pulse is 80/min, and respirations are 14/min. Examination shows an alert child in no distress. There is a soft to-and-fro bruit heard at the right costovertebral angle. Which of the following is the most likely cause of her hypertension?

• A. Coaratation of aorta

B. Renal artery atherosclerosis

• C. Pheochromocytoma

• ID. Fibromuscular dysplasia

• E Conn's syndrome

Item: 27 of 31

MY-Mark Previous

Next

Lab Values. Notes

Calculator.

• 26

a 27

• 28

• 29

• 30

• 31

0 Feedback Suspend End Block

Emaad
Highlight
Emaad
Highlight
Emaad
Highlight
Emaad
Highlight
Page 33: toddlers, infants, newborns

2

▪ 3

▪ 4-

5 ▪ 6

▪ 7

▪ 8 ▪ 9

• 10

• 11

• 12

• 13

• 14

• 15

• 16

• 17

• 18

• 19

• 20

• 21

• 22

• 23

• 24

• 25

• 26

• 27

1

A 7-year-old boy is brought to the office by his mother because, The is always daydreaming_" Sometimes, he stares for a few seconds and rolls his eyes for unknown reasons. His teacher has noted a recent decline in his school performance. He has no family history of any seizure disorder, and his psychomotor development is normal. His neurological examination is unremarkable. EEG during hyperventilation shows generalized, symmetrical 3-Hz spike-and-wave activity on a normal background. Which of the following medications is the most appropriate to treat the boy's disorder?

• A. Phenytoin

• B. Phenobarbital

• C. Valproic acid

• Prednisone

• E. Gabapentin

Item: 28 of 31

F' Mark Previous

Next

Lab Values. Notes

Calculator.

28

• 29

• 30

• 31

0 Feedback Suspend End Block

Emaad
Highlight
Emaad
Highlight
Emaad
Highlight
Emaad
Highlight
Page 34: toddlers, infants, newborns

2

▪ 3

▪ 4-

5 ▪ 6

▪ 7

▪ 8 ▪ 9

• 10

• 11

• 12

• 13

• 14

• 15

• 16

• 17

• 18

• 19

• 20

• 21

• 22

• 23

• 24

• 25

• 26

• 27

1

While making rounds in the newborn nursery, the nurses ask you to examine a 2-day-old infant who is not feeding well. The nurse reports that the infant is irritable, is not sleeping well, and has had several episodes of vomiting and loose stools today. A review of the maternal history reveals that she had poor prenatal care and the pregnancy was complicated by intrauterine growth restriction. On examination, the infant is diaphoretic and has a high-pitched cry_ The infant is also noted to have occasional sneezing and is mildly tachypneic. No dysmorphic features are noted and the remainder of the examination is unremarkable. This infant's symptoms are most likely caused by prenatal exposure to which of the following?

• A. Valproic acid

• Phenytoin

• C. Alcohol

• ID. Cocaine

• E Heroin

Item: 29 of 31

F' Mark Previous

Next

Lab Values. Notes

Calculator.

• 28

• 29

- 30

- 31

0 Feedback Suspend End Block

Emaad
Highlight
Emaad
Highlight
Emaad
Highlight
Emaad
Highlight
Emaad
Highlight
Emaad
Highlight
Emaad
Highlight
Page 35: toddlers, infants, newborns

2

▪ 3

▪ 4-

▪ 5 ▪

6 ▪ 7

▪ 8 ▪ 9

• 10

• 11

• 12

• 13

• 14

• 15

• 16

• 17

• 18

• 19

• 20

• 21

• 22

• 23

• 24

• 25

• 26

• 27

▪ 28

▪ 29

• 30

i• 31

1

A 1-year-old boy is brought to the physician by his parents for evaluation of bruising and blood in his stool_ The child has had multiple episodes of otitis media and has been hospitalized twice with pneumonia, but has never had bleeding or easy bruising in the past. He has been eating and drinking well with no vomiting or diarrhea. He is afebrile with a heart rate of 150/min and a blood pressure of 8G/40 mmHg. On examination, he is well-developed, well-nourished, and has a fair complexion_ There are dry, scaly patches on his cheeks and lower extremities. He also has bruising and purpura on his lower extremities_ His diaper contains a small amount of occult blood positive stool, but no fissures are seen on rectal examination. The remainder of the physical examination is unremarkable. Laboratory studies show:

Complete blood count

Hemoglobin 11.5 g/dL Platelet count 20,000/mm3 Leukocyte count 6,500/mm3

Neutrophils 76% Eosinophils 1% Lymphocytes 13% Monocytes 10%

A peripheral smear reveals a low number of platelets, and the platelets that are seen are small. Which of the following is the most likely cause of his bleeding?

• A. Bone marrow infiltration

• B. Platelet antibodies

C. Nutritional deficiency

• ID. Platelet activation and consumption

• E Impaired platelet production

Item: 30 of 31 F' Mark Previous Next

Lab Values. Notes Calculator.

0 Feedback Suspend End Block

Emaad
Highlight
Emaad
Highlight
Emaad
Highlight
Emaad
Highlight
Emaad
Highlight
Emaad
Highlight
Emaad
Highlight
Emaad
Highlight
Page 36: toddlers, infants, newborns

2

▪ 3

▪ 4-

5 ▪ 6

▪ 7

▪ 8

▪ 9

• 10

• 11

• 12

• 13

• 14

• 15

• 15

• 17

• 18

• 19

• 20

• 21

• 22

• 23

• 24

• 25

• 25

• 27

• 28

• 29

1

A 2-year-old girl is brought to the clinic due to fever, irritability and lethargy for the past two weeks. Over the past two months, she has complained of intermittent abdominal discomfort and has lost weight. Abdominal palpation reveals a firm nodular mass in the right flank. No bruits are heard. Abdominal x-ray reveals multiple calcifications in the renal area. Urine examination reveals increased levels of homovanillic acid and vanillylmandelic acid_ Which embryonic structure has this mass most likely arisen from?

• A. Metanephros

• B. Mesonephron

• C. Paramesonephron

• D. Neural crest cells

• E Lymphoid stem cells

Item: 31 of 31

F' Mark Previous

Next

Lab Values. Notes

Calculator.

• 30

i 31

0 Feedback Suspend End Block

Emaad
Highlight
Emaad
Highlight
Emaad
Highlight
Emaad
Highlight
Emaad
Highlight
Emaad
Highlight
Page 37: toddlers, infants, newborns

se

Overriding Aorta

• Pulmonary

stenosis 111

A 2-month-old boy is brought to the emergency department for evaluation of cyanosis. He became fussy and sweaty while taking his bottle, and his lips "turned blue" for several minutes during the feed. The infant has had similar episodes during feeding and crying, but these resolved quickly. His temperature is 37 C (98_6 F), blood pressure is 80/53 mm Hg, pulse is 15G/min, and respirations are 55/min_ On examination, he is agitated, cyanotic, and tachypneic. Cardiac auscultation shows a grade 2/6 crescendo-decrescendo systolic ejection murmur at the left upper sternal border and a single S2. He is placed in a knee-chest position immediately. This maneuver will improve this patient's condition predominantly by which of the following mechanisms?

A. Decreased pulmonary vascular resistance [4%]

B. Decreased systemic vascular resistance [513q

C. Increased pulmonary vascular resistance [21.6]

D. Increased systemic vascular resistance [67'3C]

E. Increased systemic venous return [2196]

Explanation:

Tetralogy of Fallot

▪ 3

▪ 4-

▪ 5

▪ 6

▪ 7

▪ 8

▪ 9

• 10

• 11

• 12

• 13

• 14

• 15

• 16

• 17

• 18

• 19

• 20

• 21

• 22

• 23

• 24

• 25

• 25

• 27

• 28

• 29

• 30

• 31

Item: 1 of 31

F' Mark Previous

Next

Lab Values. Notes

Calculator.

Feedback End Block

Page 38: toddlers, infants, newborns

A-

Lab Values Notes Calculator. Item: 1 of 31 l 'Mark

Previous Next

Feedback, End Block

• 3

• 4

• 5

• 6

• 7

• 3

• 9

• 10

• 11

• 12

• 13

• 14

• 15

• 16

• 17

• 18

• 19

• 20

• 21

• 22

• 23

• 24

• 25

• 25

• 27

• 28

• 29

• 30

• 31

uvrierld corn

Right ventricular hypertrophy

Tetralogy of Fallot

Overriding Aorta

Pulmonary stenosis

Ventricular

septa' defect

Page 39: toddlers, infants, newborns

Item: 1 of 31 lark Previous Next

Lab Values. Notes Calculator.

p

Right ventricular hypertrophy

iDwanrld corn

Tetralogy of Fallot is the most common cyanotic congenital heart defect and has 4 characteristic anomalies:

1. Right ventricular outflow tract (RVOT) obstruction (pulmonary stenosis or atresia) 2. Right ventricular hypertrophy 3. Overriding aorta 4. Ventricular septal defect (VSD)

Blood flow across the large, unrestrictive VSD is determined by the relative resistance between the systemic and pulmonary circulations_ If the resistance of the systemic circulation is higher than the resistance of the pulmonary circulation, the blood will shunt from the ventricles to the pulmonary artery (eg, left-to-right shunt). However, if the pulmonary vascular resistance is greater than that of the systemic circulation, the blood will shunt from the ventricles into the aorta (eg, right-to-left shunt), resulting in cyanosis.

The clinical presentation depends primarily on the degree of RVOT obstruction, ranging from minimal symptoms to profound cyanosis and hypoxemia_ Patients with even mild stenosis can develop dramatic infundibular spasm precipitated by agitation or exertion (eg, feeding, crying, hyperventilation), resulting in near-complete occlusion of the RVOT_ The increase in pulmonary vascular resistance (Choice C) causes shunting of deoxygenated blood from the right ventricle across the VSD into the aorta The sudden cyanosis is known as a "hypercyanotic," "hypoxic," or "tet" spell.

Hypercyanotic spells are dangerous and should be managed by immediate placement in a knee-chest position. This maneuver increases systemic vascular resistance, which in turn reduces the degree of right-to-left shunting by promoting right ventricular blood flow into the pulmonary circulation instead of the aorta. Inhaled oxygen should be provided to stimulate pulmonary vasodilation and systemic vasoconstriction. Morphine can also be given to relax the patient, and intravenous fluids can be given to improve right ventricle filling and pulmonary flow.

(Choice A) Inhaled oxygen should be provided to stimulate pulmonary vasodilation (eg, decrease pulmonary vascular resistance). se

▪ 3

▪ 4-

5 ▪ 6

▪ 7

▪ 8 ▪ 9

• 10

• 11

• 12

• 13

• 14

• 15

• 16

• 17

• 18

• 19

• 20

• 21

• 22

• 23

• 24

• 25

• 25

• 27

• 28

• 29

• 30

• 31

Feedback End Block

Page 40: toddlers, infants, newborns

Item: 1 of 31 V-Mark

Previous Next

Lab Values. Notes Calculator.

The clinical presentation depends primarily on the degree of RVOT obstruction, ranging from minimal symptoms to profound cyanosis and hypoxemia. Patients with even mild stenosis can develop dramatic infundibular spasm precipitated by agitation or exertion (eg, feeding, crying, hyperventilation), resulting in near-complete occlusion of the RVOT_ The increase in pulmonary vascular resistance (Choice C) causes shunting of deoxygenated blood from the right ventricle across the YID into the aorta The sudden cyanosis is known as a "hypercyanotic," "hypoxic," or "tet" spell_

Hypercyanotic spells are dangerous and should be managed by immediate placement in a knee-chest position_ This maneuver increases systemic vascular resistance, which in turn reduces the degree of right-to-left shunting by promoting right ventricular blood flow into the pulmonary circulation instead of the aorta. Inhaled oxygen should be provided to stimulate pulmonary vasodilation and systemic vasoconstriction. Morphine can also be given to relax the patient, and intravenous fluids can be given to improve right ventricle filling and pulmonary flow.

(Choice A) Inhaled oxygen should be provided to stimulate pulmonary vasodilation (eg, decrease pulmonary vascular resistance}_

(Choice B) The knee-chest and squatting positions increase systemic vascular resistance.

(Choice E) Placement of the patient in a knee-chest position can transiently increase venous return to the right heart However, venous return is increased secondary to increased systemic vascular resistance. The primary mechanism of intravenous fluids is to increase systemic venous return (eg, preload).

Educational objective: The degree of right ventricular outflow tract obstruction can fluctuate in tetralogy of Fallot. Patients with mild obstruction can develop a dramatic spasm resulting in "hypercyanotic" or "tet" spells. Placement of patients in a knee-chest position during a hypercyanotic spell increases systemic vascular resistance, increases pulmonary blood flow, and improves symptoms and cyanosis_

References:

1. When 'blue babies' grow up: what you need to know about tetralogy of Fallot.

2. Tetralogy of Fallot.

Copyright © LIWorld

Last updated: [9/18/2014]

▪ 3

4-

5

6

▪ 7

▪ 8

▪ 9

• 10

• 11

• 12

• 13

• 14

• 15

• 16

• 17

• 18

• 19

• 20

• 21

• 22

• 23

• 24

• 25

• 25

• 27

• 28

• 29

• 30

• 31

Feedback End Block

Page 41: toddlers, infants, newborns

1

2

3

▪ 4-

5 ▪ 6

▪ 7

▪ 8

▪ 9

• 10

• 11

• 12

• 13

• 14

• 15

• 15

• 17

• 18

• 19

• 20

• 21

• 22

• 23

• 24

• 25

• 25

• 27

• 28

• 29

• 30

• 31

Item: 2 of 31

F' Mark Previous

Next

Lab Values. Notes

Calculator.

An 8-year-old girl is brought to the emergency department with fever for the past 6 days. Her parents report that she has been very "cranky" and has developed a new rash_ Six weeks ago, she completed a 1G-day course of amoxicillin for streptococcal pharyngitis. She has no other medical problems and takes no medications. Her temperature is 394° C (103° F), blood pressure is 110/60 mm Hg, pulse is 120/min, and respirations are 24/min. Physical examination shows injected lips and pharynx. Bilateral conjunctivae are also injected, with no exudates. A 1.7-cm mobile lymph node is palpated on the left neck. A blanching erythematous rash is present across her face, trunk, and extremities, including the palms and soles. Her mouth is shown below.

What is the most appropriate next step in management of this patient?

A. Amoxicillin [7%] se

Feedback End Block

Page 42: toddlers, infants, newborns

se

Feedback End Block

Item: 2 of 31

MI 1-Mark Previous

Next

Lab Values. Notes

Calculator.

What is the most appropriate next step in management of this patient?

A. Amoxicillin [1%]

B. Aspirin and intravenous immunoglobulin [74%]

C. Doxycycline [4%]

D_ Lymph node biopsy [1°A]

E Reassurance and close follow-up [11%]

• Skin biopsy [ON

G. Throat culture [4%]

Explanation:

• Kawasaki disease

D iagnostic criteria

Fever 5 days plus 4 of the following findings:

• Bilateral nonexudative conjunctivitis

* Mucositis (injected or fissured lips, injected pharynx, or

strawberry tongue)

• Cervical lymphadenopathy with at least ore lymph node

being >1.5 cm in diameter

• Erythematous polymorphous rash

• Extremity changes (edema 8, erythema)

TnpatimAnt Acnirin Mug intrAvpnrum immiinrinInhillin

3

4-

5

6

7

8

9

• 10

• 11

• 12

• 13

• 14

• 15

• 16

• 17

• 18

• 19

• 20

• 21

• 22

• 23

• 24

• 25

• 26

• 27

• 28

• 29

• 30

• 31

Page 43: toddlers, infants, newborns

Item: 2 of 31

0 Mark -4(1

Previous

Next

Lab Values. Notes

Calculator.

Kawasaki disease

Dia nosh c criteria

Fever 5 days plus ?4 of the following findings:

• Bilateral nonexudative conjunctivitis

• Mucositis (injected or fissured lips, injected pharynx, or

strawberry tongue)

• Cervical Iyrnphadenopathy with at least ore lymph node

being >1.5 cm in diameter

• Erythematous polymorphous rash

• Extremity changes (edema & erythema)

Treatment Aspirin plus intravenous immunoglobulin

Complications • Coronary artery aneurysms

• Myocardial infarction & ischemia

USMLEWorld, LLC

Kawasaki disease (KID), also known as mucocutaneous lymph node syndrome, is typically a clinical diagnosis; this patient meets all of the diagnostic criteria (Table). Irritability, as seen in this patient, is also a common feature_ KID most commonly affects children age <5 years but can occur in late childhood_ Although KID is usually self-limited, treatment with aspirin and intravenous immunoglobulin should be started within 10 days of fever onset to prevent cardiac complications. Reassurance without intervention (choice E) is inappropriate. Coronary artery aneurysms are the most serious potential sequelae and can develop in —20% of untreated patients, leading to complications such as myocardial infarction and death. Baseline echocardiography should be performed in all patients with suspected KID and repeated 6-8 weeks later to monitor for any changes.

se

1 2 3

▪ 4-

▪ 5

▪ S

▪ 7

▪ 8

▪ 9

• 10

• 11

• 12

• 13

• 14

• 15

• 15

• 17

• 18

• 19

• 20

• 21

• 22

• 23

• 24

• 25

• 25

• 27

• 28

• 29

• 30

• 31

Feedback End Block

Page 44: toddlers, infants, newborns

• 1 2

• 3

• 4-

• 5

• 6

• 7

• 8

• 9

• 10

• 11

• 12

• 13

• 14

• 15

• 15

• 17

• 18

• 19

• 20

• 21

• 22

• 23

• 24

• 25

• 25

• 27

• 28

• 29

• 30

• 31

Item: 2 of 31 Mark -4Z1

Previous

Next

Lab Values. Notes

Calculator.

I'L0 I, usually SETT-IIrr11teo, Treatment with aspirin ono IMTFiVCV. L= .4.1 .JU., UL 110 nn 3LCII 1111

10 days of fever onset to prevent cardiac complications. Reassurance without intervention (choice El is inappropriate. Coronary artery aneurysms are the most serious potential sequelae and can develop in —20% of untreated patients, leading to complications such as myocardial infarction and death. Baseline echocardiography should be performed in all patients with suspected KID and repeated 6-8 weeks later to monitor for any changes.

KD is one of the few pediatric illnesses in which aspirin therapy is warranted because of its antiplatelet and anti-inflammatory properties_ However, patients should be cautioned about the risk of Reye syndrome. Reye syndrome is a rare but life-threatening hepatic encephalopathy that can develop in children using aspirin during influenza or varicella infections.

(Choices A and G) The differential diagnosis for fever and rash includes scarlet fever. Scarlet fever is a potential complication of untreated streptococcal pharyngitis. Although this patient has pharyngeal injection, she lacks tonsillar exudates that typically are present in streptococcal pharyngitis_ In addition, the rash in scarlet fever has a "sandpaper" texture and spares the palms and soles, making this diagnosis unlikely. Therefore, amoxicillin and throat culture are unnecessary.

(Choice C) Rocky Mountain spotted fever can cause rash on the palms and soles and is best treated with doxycycline for 5-7 days_ However, headache and gastrointestinal symptoms typically are prominent features of this infection, making this diagnosis less likely.

(Choices D and F) Laboratory studies and biopsies are not needed to make the diagnosis when the presentation is classic for KID_

Educational objective: Kawasaki disease is characterized by L5 days of fever, bilateral nonexudative conjunctivitis, mucositis, cervical lymphadenopathy, rash, and extremity changes. Aspirin and intravenous immunoglobulin can reduce the incidence of complications such as coronary artery aneurysms_

References:

1. Diagnosis, treatment, and long-term management of Kawasaki disease: a statement for health professionals from the Committee on Rheumatic Fever, Endocarditis, and Kawasaki Disease, Council on Cardiovascular Disease in the Young, American Heart Association.

Feedback End Block

Page 45: toddlers, infants, newborns

Item: 2 of 31

VI/lark -4Z1

Previous

Next

Lab Values. Notes

Calculator.

1 2

3

▪ 4-

5 ▪ 6

▪ 7

▪ 8

▪ 9

• 10

• 11

• 12

• 13

• 14

• 15

• 15

• 17

• 18

• 19

• 20

• 21

• 22

• 23

• 24

• 25

• 25

• 27

• 28

• 29

• 30

• 31

inappropriate. Coronary artery aneurysms are the most serious potential sequelae and can develop in —20% of untreated patients, leading to complications such as myocardial infarction and death. Baseline echocardiography should be performed in all patients with suspected KID and repeated 6-8 weeks later to monitor for any changes.

KID is one of the few pediatric illnesses in which aspirin therapy is warranted because of its antiplatelet and anti-inflammatory properties_ However, patients should be cautioned about the risk of Reye syndrome_ Reye syndrome is a rare but life-threatening hepatic encephalopathy that can develop in children using aspirin during influenza or varicella infections.

(Choices A and G) The differential diagnosis for fever and rash includes scarlet fever. Scarlet fever is a potential complication of untreated streptococcal pharyngitis. Although this patient has pharyngeal injection, she lacks tonsillar exudates that typically are present in streptococcal pharyngitis_ In addition, the rash in scarlet fever has a "sandpaper" texture and spares the palms and soles, making this diagnosis unlikely. Therefore, amoxicillin and throat culture are unnecessary.

(Choice C) Rocky Mountain spotted fever can cause rash on the palms and soles and is best treated with doxycycline for 5-7 days_ However, headache and gastrointestinal symptoms typically are prominent features of this infection, making this diagnosis less likely.

(Choices D and F) Laboratory studies and biopsies are not needed to make the diagnosis when the presentation is classic for KID.

Educational objective: Kawasaki disease is characterized by L5 days of fever, bilateral nonexudative conjunctivitis, mucositis, cervical lymphadenopathy, rash, and extremity changes. Aspirin and intravenous immunoglobulin can reduce the incidence of complications such as coronary artery aneurysms_

References:

1. Diagnosis, treatment, and long-term management of Kawasaki disease: a statement for health professionals from the Committee on Rheumatic Fever, Endocarditis, and Kawasaki Disease, Council on Cardiovascular Disease in the Young, American Heart Association.

Copyright © UWorld Last updated: [9/20/2014]

Feedback End Block

Page 46: toddlers, infants, newborns

Kawasaki disease

Bilateral nonexudative conjunctivitis

Rash -

Mucositis

Fever for 5 days

Cervical lymph node >1.5crn

V

Item: 2 of 31 Mark -4(1

Previous Next Lab Values Notes Calculator

Media Exhibit

Kawasaki disease

Feedback a

End Block

-1

2

3

▪ 4-

▪ 5

▪ 6

▪ 7

▪ 8

▪ 9

• 10

• -1-1

• 12

• 13

• 14

• 15

• 16

• 17

• -18

• 19

• 20

• 21

• 22

• 23

• 24

• 25

• 26

• 27

• 28

• 29

• 30

• 3-1

A

Page 47: toddlers, infants, newborns

Feedback a

End Block

Mark Next Notes

-4(1

Previous Calculator Lab Values

Item: 2 of 31

A Media Exhibit

Kawasaki disease

Bilateral nonexudative conjunctivitis

Rash

14111

Swelling 8dor \ erythema of pal m sisoles

Mucositis

Coronary artery aneurysms

0 USIALEWorid, L LC

5 days

Cervical lymph node >1.5crn

-1

2

3

▪ 4-

▪ 5

▪ 6

▪ 7

▪ 8

▪ 9

• 10

• -1-1

• 12

• 13

• 14

• 15

• 16

• 17

• -18

• 19

• 20

• 21

• 22

• 23

• 24

• 25

• 26

• 27

• 28

• 29

• 30

• 3-1

Page 48: toddlers, infants, newborns

-1

2

3

▪ 4-

5 ▪ 6

▪ 7

▪ 8

▪ 9

• 10

• -1-1

• 12

• 13

• 14

• 15

• 16

• 17

• -18

• 19

• 20

• 21

• 22

• 23

• 24

• 25

• 26

• 27

• 28

• 29

• 30

• 3-1

Next Notes Previous .111 Mark Mora

Calculator Lab Values

Item: 2 of 31

Media Exhibit

a Feedback

End Block

Page 49: toddlers, infants, newborns

• 1 2

3

4-

▪ 5

▪ 6

▪ 7

▪ 8 ▪ 9

• 10

• 11

• 12

• 13

• 14

• 15

• 16

• 17

• 18

• 19

• 20

• 21

• 22

• 23

• 24

• 25

• 25

• 27

• 28

• 29

• 30

• 31

Item: 3 of 31 V-• Mark Previous Next

Lab Values, Notes Calculator.

A 16-year-old female runner comes to the physician with excessive facial hair. She has used multiple creams and makeup over the last year but still considers the facial hair embarrassing_ She also complains of irregular menstrual cycles since menarche at age 12. The patient's last menstrual period was 4 weeks ago. Her blood pressure is 122/70 mm Hg and pulse is 87/min. Her body mass index is 31 kg/m 2. Physical examination confirms hirsutism. The patient has normal female external genitalia, and the rest of the examination is unremarkable. Urine pregnancy test is negative. Laboratory results are as follows:

Serum luteinizing hormone

20 IU/L (0.5-15 IU/L) Serum follicle-stimulating hormone

10 IU/L (1-9 IU/L)

17-hydroxyprogesterone

350 ng/mL (Normal <15 ngimL) Serum testosterone

80 ng/mL (Normal 20-75 ng/dL) Serum DHEA-S

390 ng/mL (Normal 75-370 pgldL)

Serum glucose and electrolytes are normal. Which of the following is the most likely diagnosis?

A. Adrenal carcinoma [2%]

B. Congenital adrenal hyperplasia [53%]

C. Cushing's syndrome [19C]

D. Germ cell tumor [2%]

E. Idiopathic hirsutism [1%]

Polycystic ovarian disease [35°4]

G. Use of performance-enhancing agents [5%]

Explanation:

Hyperandrogenernia & ago-ovulation

7 Polycystic ovarian disease is the most common cause_

a Feedback

End Block

MA, ;.• ~Li *k. ire411.0.3.14. se

Page 50: toddlers, infants, newborns

• 1 2

3

4-

5

6

▪ 7

▪ 8 ▪ 9

• 10

• 11

• 12

• 13

• 14

• 15

• 16

• 17

• 18

• 19

• 20

• 21

• 22

• 23

• 24

• 25

• 25

• 27

• 28

• 29

• 30

• 31

Item: 3 of 31

I 1-Mark .4111°. Previous

Next . Lab Values. Notes

Calculator.

Hyperandrogenemia & ago-ovulation

Older age,

rapidly progressive

symptoms &

very high

androgen levels.

(>3 times upper

limit of normal)

Typical phenotypic

features, non-

suppressible

dexamethasone

suppression test

& elevated 24-hour

urinary free cortisol

Polycystic ovarian disease is the most common cause_

ItS d iagnosis is made after the following disorders are ruled out.

Ovarian or

adrenal tumors

Late-onset congenital

adrenal hyperplasia

Hyper-

prolactinemia

Acromegaly Cushing s disease

Family history,

ethnic predisposition,

& high 17-hydroxy

progesterone levels

Amenorrhea,

galactor rhea

& high prolactin

Typical phenotypic

features & elevated

insulin-like growth

factor 1 levels

LISILEY,Iarkl.LLC

Congenital adrenal hyperplasia (CAH) is a group of disorders due to enzyme deficiencies for cortisol synthesis in the adrenal cortex_ The resultant low plasma cortisol stimulates the pituitary to increase adrenocorticotropic hormone (ACTH) production, which leads to adrenal hyperplasia and increased concentration of steroid precursor before the dysfunctional enzyme. Approximately 90% of cases of adrenal hyperplasia are due to deficiency of 21-hydroxylase, which is involved in the cortisol (converts 17-hydroxyprogesterone to 11-daqcortisol) and aldosterone (converts progesterone to deoxycorticosterone) pathways. 21-hydronflase deficiency increases 17-hydroxyprogesterone (steroid precursor) levels, which are diverted toward adrenal androgen synthesis and lead to hyperandrogenism. Concomitant aldosterone deficiency causes salt wasting.

The clinical spectrum of 21 -hydroxylase deficiency varies depending on the degree of enzyme deficiency_ Severe (classic) 21-hydroxylase deficiency presents in the neonate or in early infancy with virilization in girls and salt-losing crisis in boys. Nonclassic (late-onset) CAH has milder enzyme deficiency and presents in se

J.-. .__

a Feedback

End Block

Page 51: toddlers, infants, newborns

Item: 3 of 31 VI/lark Previous Next

Lab Values. Notes Calculator.

1-.1 I •-•

diverted toward adrenal androgen synthesis and lead to hyperandrogenism. Concomitant aldosterone deficiency causes salt wasting.

The clinical spectrum of 21-hydroxylase deficiency varies depending on the degree of enzyme deficiency_ Severe (classic) 21-hydro,qlase deficiency presents in the neonate or in early infancy with virilization in girls and salt-losing crisis in boys. Nonclassic (late-onset) CAH has milder enzyme deficiency and presents in adolescent girls and women in a spectrum ranging from mild symptoms to acne, irregular menses, and mild hirsutism that are difficult to distinguish from polycystic ovarian syndrome. Adolescent boys with 21-hydroxylase deficiency usually present with precocious puberty. In addition, there can be varying degrees of salt wasting in both genders depending on the severity of enzyme deficiency.

This patient presents at age 16 with hirsutism and menstrual irregularities consistent with likely mild 21-hydrox0ase deficiency. Diagnosis is suggested by the elevated 17-hydroxyprogesterone and can be confirmed by an exaggerated 17-hydroxwrogesterone response with the ACTH stimulation test DHEA and androstenedione are weak androgens that may be mildly elevated, but serum testosterone is not always elevated in nonclassic CAH.

(Choice A) Adrenocortical cancers are rare tumors that are usually very aggressive_ Androgen-producing adrenal tumors (androblastoma, arrhenoblastoma, stromal and hilus cell tumor) occur later in life and present with rapidly progressive hirsutism and sometimes virilization. Serum DHEA-S, an adrenal androgen, is markedly elevated with concentrations >700 pgidL.

(Choice C) Cushing's syndrome can also present with hirsutism, usually due to elevated adrenal androgens. Although this patient is obese, she does not have other features of Cushing's syndrome (eg, moon facies, buffalo hump, central obesity, hypertension, hyperglycemia, hypokalemia). In addition, 17-OH progesterone levels are usually normal in Cushing's syndrome.

(Choice ID) Ovarian germ cell tumors typically present with abdominal pain or enlargement. precocious puberty, abnormal vaginal bleeding, or pregnancy symptoms due to increased human chorionic gonadotropin hormone. However, they do not usually cause hirsutism. In addition, ovarian androgen tumors are associated with increased testosterone and suppressed luteinizing hormone (LH) levels.

(Choice E) Idiopathic hirsutism is due to excessive conversion of testosterone to dihydrotestosterone in the hair follicles. There is usually a positive family history, and 17-OH progesterone levels are usually normal without evidence of virilization. Androgen levels are also normal in patients with idiopathic hirsutism.

(Choice F) Polycystic ovarian disease (PCOD) typically presents with oligo-ovulation, clinical or biochemical se J _I.

• 1 2

3

4-

5

6

▪ 7

▪ 8

▪ 9

• 10

• 11

• 12

• 13

• 14

• 15

• 16

• 17

• 18

• 19

• 20

• 21

• 22

• 23

• 24

• 25

• 25

• 27

• 28

• 29

• 30

• 31

Feedback End Block

Page 52: toddlers, infants, newborns

• 1 2

3 4-

▪ 5

▪ 6

▪ 7

▪ 8 ▪ 9

• 10

• 11

• 12

• 13

• 14

• 15

• 16

• 17

• 18

• 19

• 20

• 21

• 22

• 23

• 24

• 25

• 25

• 27

• 28

• 29

• 30

• 31

Item: 3 of 31

F' Mark -4Z1

Previous

Next

Lab Values. Notes

Calculator.

(Choice C) Cushing's syndrome can also present with hirsutism, usually due to elevated adrenal androgens. Although this patient is obese, she does not have other features of Cushing's syndrome (eg. moon facies. buffalo hump, central obesity, hypertension, hyperglycemia, hypokalemia). In addition. 17-OH progesterone levels are usually normal in Cushing's syndrome.

(Choice ID) Ovarian germ cell tumors typically present with abdominal pain or enlargement, precocious puberty, abnormal vaginal bleeding, or pregnancy symptoms due to increased human chorionic gonadotropin hormone. However, they do not usually cause hirsutism. In addition, ovarian androgen tumors are associated with increased testosterone and suppressed luteinizing hormone (LH) levels.

(Choice E) Idiopathic hirsutism is due to excessive conversion of testosterone to dihydrotestosterone in the hair follicles. There is usually a positive family history, and 17-OH progesterone levels are usually normal without evidence of virilization. Androgen levels are also normal in patients with idiopathic hirsutism.

(Choice F) Polycystic ovarian disease (PCOD) typically presents with oligo-ovulation, clinical or biochemical hyperandrogenemia, and polycystic ovaries on imaging. The diagnosis requires 2 of these 3 features and exclusion of other disorders (eg, CAH, Cushing's syndrome, acromegaly). However, elevated 17-hydroxprogesterone is not usually seen in PCOD and makes CAH more likely in this patient High LH to follicle-stimulating hormone (FSH) ratio is a nonspecific finding in PCOD.

(Choice G) Use of androgens and anabolic steroids for performance enhancement is less common in women than men. In addition, androgen or anabolic steroid use suppresses LH and FSH levels via the negative feedback mechanism.

Educational objective: 21-hydroxylase deficiency is the most common cause of congenital adrenal hyperplasia. Partial deficiency of 21-hydroxylasein female patients typically presents in adolescence or adulthood as hyperandrogenism (eg, hirsutism, virilism), elevated 17-hydroxyprogesterone, and varying degrees of salt wasting (depending on the degree of enzyme deficiency).

References:

1. Non-classic congenital adrenal hyperplasia.

2. Congenital adrenal hyperplasia due to 21 hydroxylase deficiency: from birth to adulthood.

Copyright © UWorld Last updated: [9/3/2014]

Feedback End Block

Page 53: toddlers, infants, newborns

W. 4. Lab Values Notes Calculator

• 1

• 2 NI 1'2 ilark Item: 3 of 31

Peclia Exhibit

Previous Next

Congenital adrenal hyperplasia

cleavage enzyme

De hyd roe pia n drosteron e

Progesterone 17-0i-1 progesterone 1 11110. 11.

lill21-Hydroxylaw

1 1 -Deoxycorticosterone (weak mineratocorticoid)

110-Hydroxylase

Corticosterone

(weak glucocorticoid)

Aldosterone

Mineral ocorticokl s

11-Deoxycortisol

1 1 13-HydrOXylaSe

Cortisol

GI ucocortico ids

V

Testosterone

I Androgens

0 4.151ALEwarid, U.(

21 -Hydroxylase

17a-Hyd roxylase

A n d rostened io ne

Peripheral tissues

14.

• 5

• 6

. 7

• 8

. 9

• 10

• 11

• 12

• 13

• 14

• 15

• 16

• 17

• 18

• 19

• 20

• 21

• 22

• 23

• 24

• 25

• 26

• 27

• 28

• 29

• 30

• 31

Page 54: toddlers, infants, newborns

se

Item: 4 of 31

.11V-• Mark Previous

Next

Lab Values. Notes

Calculator. 2

3 4

5

A 5-year-old boy is brought to the emergency department for a worsening cough. Two weeks ago, he developed rhinorrhea, congestion, and a mild cough_ Since then, the cough has worsened to the point that he often vomits after coughing. The cough is worse at night, during exercise, and with showering. He has taken over-the-counter antitussive medications with no relief. The boy is on a delayed immunization schedule per parental preference and has received the measles-mumps-rubella vaccination. His temperature is 37.2 C (99 F), blood pressure is 90/6G mm Hg, pulse is 98/min, and respirations are 20/min. Pulse oximetry shows an oxygen saturation of 98%_ Physical examination shows an alert and well-appearing child. He has bilateral subconjunctival hemorrhages and periorbital petechiae. The lungs are clear to auscultation. Which organism is the most likely cause of this child's illness?

A. Adenovirus [6%]

B. Bordetetla pertussis [75%]

C. Chiamydophila pireumorliae [196]

D. Mycobacterium tuberculosis [0%]

E Mycoplasma pneurnoniae [2%]

• Nontypeable Haemophilus influenzae [5°S6]

G. Parainfluenza virus [4%]

H. Respiratory syncytial virus [5%]

I. Streptococcus pneumoniae [2%]

Explanation:

Pertussis in infants & children

5

7

8 ▪ 9

• 10

• 11

• 12

• 13

• 14

• 15

• 16

• 17

• 18

• 19

• 20

• 21

• 22

• 23

• 24

• 25

• 25

• 27

• 28

• 29

• 30

• 31

Paroxysmal phase Convalescent

phase Catarrhal

phase

20- to 30-minute coughing paroxysms

Feedback End Block

Page 55: toddlers, infants, newborns

Item: 4 of 31

F-Mark Previous

Next

Lab Values. Notes

Calculator.

Pertussis in infants & children

Clinical manifestations

Catarrhal phase

Paroxysmal phase Convalescent

phase

• Mild cough

• Rhinitis

• 20- to 30-minute coughing paroxysms

• Inspiratory 'whoop"

• 'Staccato' cough

• Posttussive emesis

• Apnea, cyanosis (infants)

• Cough &

ernesis resolve

Duration of symptoms 1-2 weeks 2-6 weeks Weeks to months

Diagnosis

....._

• ljerlussis culture or polyrnerase chain reaction

• Lymphocyte-predominant leukocytosis

Treatment - I

• Macrolides (eg, azithromycin, erythromycin, clarithromycin) —

Prevention • Acellular pertussis vaccine

1

Complications

• Pneumonia

• Weight loss

• Subconjunctival hemorrhages

• Pneumothorax

• Respiratory failure, death (infants)

©ILISMLEWorld, LLC

Feedback End Block

2

a 4

5

5

7

8

▪ 9

• 10

• 1-1

• 12

• 13

• 14

• 15

• 16

• 17

• -18

• 19

• 20

• 21

• 22

• 23

• 24

• 25

• 26

• 27

• 28

• 29

• 30

• 31

Page 56: toddlers, infants, newborns

Item: 4 of 31 F'Mark Previous Next Lab Values. Notes Calculator.

A 15...e.J...3

• Subconjunctival hemorrhages

• Pneurnothorax

• Respiratory failure, death (infants)

Complications

©USMLEWorld, LLC

Bordetella pertussis is a Gram-negative coccobacillus that causes "whooping cough" ("100-day cough"), a highly contagious illnesses characterized by paroxsyms of coughing_ The illness begins with a mild catarrhal phase that resembles the common cold. This is followed by the paroxysmal phase in which severe, periodic coughing bouts can last up to 30 minutes. Each coughing episode often starts with a "whoop" caused by forced inspiration. Forceful coughing often triggers posttussive emesis, which may result in weight loss. Symptoms tend to worsen at night or with laughing, exercise, or exposure to steam or smoke_ Patients can appear remarkably well between coughing spells. Infants age <6 months are at substantial risk for life-threatening apnea and death.

Initial laboratory and x-ray findings may be normal, but patients can develop marked lymphocytosis. Macrolides reduce symptom severity and contagiousness and are the first-line treatment Over-the-counter antitussive medications should be avoided due to lack of proven efficacy and risk of toxicity, especially in children age <6 years_

(Choice A) Adenovirus is a common upper respiratory infection in children and is often accompanied by fever. pharyngitis. rhinitis, conjunctivitis, and diarrhea Coughing paroxysms and posttussive emesis are not typically seen.

(Choices C and E) Chiamydophila prieumorriae and Mycoplasma paeurnorliae cause atypical pneumonias, which present with a gradual onset of symptoms, mild cough, and low-grade fevers_

(Choice 0) Mycobacterium tuberculosis causes tuberculosis, a serious illness that presents with prolonged fever, night sweats, chronic cough. and weight loss. This child does not have systemic symptoms.

(Choice F) Nontypeable Haemophilus influertzae is a common cause of community-acquired pneumonia H influenzae does not cause paroxysmal coughing.

(Choice G) Parainfluenza is a very common cause of laryngotracheobronchitis (croup) in children. Croup presents with fever, a "barky" cough, and inspiratory stridor, none of which are present in this patient.

iChoice Hi Respiratory svncvtial virus is the most common cause of bronchiolitis in children ape <2 years. It se

• 1

• 2

3 4

5

▪ 5

• 7

▪ 8

• 10

• 11

• 12

• 13

• 14

• 15

• 15

• 17

• 18

• 19

• 20

• 21

• 22

• 23

• 24

• 25

• 25

• 27

• 28

• 29

• 30

• 31

Feedback End Block

Page 57: toddlers, infants, newborns

• 1

• 2

3 4

5

• 5

• 7

• 8

• 9

• 10

• 11

• 12

• 13

• 14

• 15

• 15

• 17

• 18

• 19

• 20

• 21

• 22

• 23

• 24

• 25

• 25

• 27

• 28

• 29

• 30

• 31

Item: 4 of 31

V-Mark

UP. Previous

Next

Lab Values, Notes

Calculator .

fever, pharyngitis, rhinitis, conjunctivitis, and diarrhea Coughing paroxysms and posttussive emesis are not typically seen.

(Choices C and E) Chlamydophila prieumorriae and Mycoplasma pneurnorliae cause atypical pneumonias, which present with a gradual onset of symptoms, mild cough, and low-grade fevers_

(Choice 0) Mycobacterium tuberculosis causes tuberculosis, a serious illness that presents with prolonged fever, night sweats, chronic cough, and weight loss. This child does not have systemic symptoms.

(Choice F) Nontypeable Haemophilus influerrzae is a common cause of community-acquired pneumonia H influerrzae does not cause paroxysmal coughing.

(Choice G) Parainfluenza is a very common cause of laryngotracheobronchitis (croup) in children. Croup presents with fever, a "barky" cough, and inspiratory stridor, none of which are present in this patient

(Choice H) Respiratory syncytial virus is the most common cause of bronchiolitis in children age <2 years_ It presents with fever, rhinorrhea, cough, and mild respiratory distress.

(Choice I) Streptococcus prreurnomae is the most common cause of community-acquired pneumonia in children and presents with fever. cough. and tachvpnea. Paroxysms of coughing are typically not seen.

Educational objective: Bordetella pertussis causes "whooping cough," which has 3 phases: catarrhal, paroxysmal, and convalescent The paroxysmal phase is characterized by severe paroxysms of coughing and posttussive emesis_ Macrolide antibiotics are the first-line treatment

References:

1. Clinical presentation of pertussis in unvaccinated and vaccinated children in the first six years of life.

2. Whooping cough in school age children with persistent cough: prospective cohort study in primary care.

3. Recommended antimicrobial agents for the treatment and postexposure prophylaxis of pertussis: 2005 CDC guidelines.

Copyright @ UWorld

Last updated: [10/5/2014]

Feedback End Block

Page 58: toddlers, infants, newborns

2

3

4

5

6

7

8

▪ 9

• 10

• 11

• 12

• 13

• 14

• 15

• 16

• 17

• 18

• 19

• 20

• 21

• 22

• 23

• 24

• 25

• 26

• 27

• 28

• 29

• 30

• 31

Item: 5 of 31 V-• Mark Previous Next

Lab Values. Notes Calculator.

A 5-year-old girl is brought to the physician for 'wetting the bed." Since age 3, she has been completely toilet-trained during the day and at night However, for the past 2 weeks, the girl has wet the bed nightly and has slept poorly from trying to make it to the bathroom multiple times each night She is also more thirsty than usual, urinating more often during the day, and having ''accidents" at school. Review of systems is positive for fatigue and "crankiness" and negative for dysuria, hesitancy, and urgency_ She has had no major illnesses and has met all developmental milestones. One month ago, the family moved to a new house after her brother was born. Vital signs are normal. Weight and height are at the 25th percentile. Physical examination shows a tired-appearing girl. Mucous membranes are dry_ The rest of the examination is unremarkable. Which of the following is the most likely explanation for this patient's symptoms?

A. Autoimmune destruction of the pancreatic beta cells [57%]

B. Bacterial infection of the bladder [1 %]

C. Behavior regression from changes in the home environment [31%]

D. Delayed maturation of sphincter control [0%]

E Impaired arousal during sleep [0%]

F. Impaired renal tubule response to antidiuretic hormone [10%]

Explanation:

Causes of secondary enuresis

Etio[ogy Associated symptoms

Psychological stress • Behavior regression, mood lability

Urinary tract infection • Dysuria, hesitancy, urgency, abdominal pair

Diabete mellitus s * Polyuria, polydipsia, polyphagia„ weight loss.

lethargy, candidiasis se

Feedback End Block

Emaad
Highlight
Page 59: toddlers, infants, newborns

• 1

• 2

▪ 3

4

5

6

▪ 7

▪ 8

• 10

• 11

• 12

• 13

• 14

• 15

• 16

• 17

• 18

• 19

• 20

• 21

• 22

• 23

• 24

• 25

• 26

• 27

• 28

• 29

• 30

• 31

Lab Values. Calculator. Notes Previous Next ii IMark Item: 5 of 31

se

Causes of secondary enuresis

Etiology Associated symptoms

Psychological stress • Behavior regression, mood lability

Urinary tract infection • Dysuria, hesitancy, urgency, abdominal pair

s Diabete mellitus • Polyuria, polydipsia, polyphagia„ weight loss.

lethargy, canclidiasis

Diabetes insipidLls • Polyuria, polydipsia

Obstructive sleep apnea • Snoring, dry mouth, fatigue, hyperactivity.

irritability

cp SNILEWor I d, LLC

Enuresis is defined as urinary incontinence in children age 5 and further characterized as primary or secondary. Children who have never achieved dryness have primary enuresis. Secondary enuresis is the return of incontinence after L.6 months of dryness and results from psychological stress or a medical problem that requires further evaluation_

This patient's enuresis is occurring in the setting of polyuria and polydipsia, which are red flags for type .1 diabetes mellitus (DM). Type 1 DM has a bimodal onset, with patients typically presenting at age 4-6 years or at early puberty. When hyperglycemia exceeds the renal threshold for glucose, the resultant glucosuria leads to osmotic diuresis, polyuria, and dehydration.

The next steps in evaluation include urinalysis_ serum chemistry, hemoglobin Al c, and blood gas analysis. When euglycemia is achieved with insulin therapy, glucosuria and polyuria should resolve. curing the enuresis.

(Choice B) Although occult urinary tract infections can cause enuresis, most patients experience dysuria. hesitancy, and urgency, making this diagnosis less likely.

Feedback End Block

Page 60: toddlers, infants, newborns

• 1

• 2

▪ 3

4

5

6

▪ 7

▪ 8

▪ 9

• 10

• 11

• 12

• 13

• 14

• 15

• 16

• 17

• 18

• 19

• 20

• 21

• 22

• 23

• 24

• 25

• 26

• 27

• 28

• 29

• 30

• 31

Item: 5 of 31 V-Mark Previous

Next

Lab Values. Notes

Calculator.

This patient's enuresis is occurring in the setting of polyuria and polydipsia, which are red flags for type 1 diabetes mellitus (DM). Type 1 DM has a bimodal onset, with patients typically presenting at age 4-6 years or at early puberty. When hyperglycemia exceeds the renal threshold for glucose, the resultant glucosuria leads to osmotic diuresis, polyuria, and dehydration.

The next steps in evaluation include urinalysis. serum chemistry, hemoglobin Al c, and blood gas analysis. When euglycemia is achieved with insulin therapy. glucosuria and polyuria should resolve, curing the enuresis.

(Choice B) Although occult urinary tract infections can cause enuresis. most patients experience dysuria, hesitancy. and urgency, making this diagnosis less likely.

(Choice C) Psychological stress (eg, moving to a new home, birth of a sibling, parental divorce) can cause behavioral regression and enuresis. However, it would not cause polyuria and polydipsia.

(Choice 0) Maturational delay of sphincter control may contribute to primary enuresis. This patient already developed sphincter control because she was previously continent

(Choice El Obstructive sleep apnea impairs sleep arousal and can therefore result in nocturnal enuresis. Associated symptoms include fatigue and irritability_ However, this patient was appropriately waking and attempting to use the toilet The fatigue and irritability are results of disrupted sleep and the underlying illness.

(Choice F) Nephrogenic diabetes insipidus presents with copious excretion of dilute urine and polydipsia. In contrast to type 1 DM, it is uncommon in children and therefore less likely to cause this patient's symptoms.

Educational objective: Polyuria and polydipsia are classic features of new-onset type 1 diabetes mellitus. Nocturnal enuresis can be a presenting symptom in toddlers.

References:

1. Characteristics at diagnosis of type 1 diabetes in children younger than 6 years.

2. Clinical presentation of type 1 diabetes.

Copyright @ LIWorld Last updated: [12/29/2014]

Feedback End Block

Page 61: toddlers, infants, newborns

Item: 6 of 31

V-Mark Previous Next . Lab Values. Notes Calculator.

• 1 • 2

• a 4

5

A 1-day-old full-term boy is in the neonatal intensive care unit with cyanosis. His blood pressure is 80/40 mm Hg in all 4 extremities, heart rate is 140/min, and respirations are 55/min_ Pulse oximetry shows 80% and does not improve with 100% inspired oxygen by face mask. He is breathing comfortably, but his fingertips and oral mucosa are blue. A continuous machine-like murmur is heard on auscultation. Chest x-ray shows clear lung fields bilaterally. Which of the following is the best next step in management of this patient?

A. Furosemide [0%]

B. Intubation with 100% Fi0.2 [4%]

C. Indomethacin [43%]

D. Propranolol [0%]

E Prostaglandin El [53%]

F. Red blood cell transfusion [CM]

Explanation:

This patient's hypoxia that fails to improve with inhaled oxygenation is concerning for a congenital heart defect An oxygen saturation of 70%-90% and central cyanosis is expected in patients with cyanotic heart disease due to mixing of deoxygenated blood through the cardiac defect. Many types of cyanotic heart disease depend on a patent ductus arteriosus (PDA) to provide vital blood flow between the pulmonary and systemic circulations (Table). Normally, the ductus arteriosus constricts and is nonfunctional around day 3 of life_ Closure of the ductus arteriosus in patients with FDA-dependent defects could cause severe hypoxia, shock, and severe metabolic (lactic) acidosis. Maintaining patency can be potentially life-saving; prostaglandin El is a vasodilator used to prevent ductus arteriosus closure in such infants until definitive surgery can be performed.

(Choice A) Loop diuretics are indicated in congenital heart disease complicated by volume overload (eg, pulmonary edema, sacral edema). This patient has no signs of volume overload.

(Choice B) Positive-pressure ventilation and oxygen supplementation can cause acute decompensation in • I I-% I

6

7

▪ 8 ▪ 9

• 10

• 11

• 12

• 13

• 14

• 15

• 16

• 17

• 18

• 19

• 20

• 21

• 22

• 23

• 24

• 25

• 26

• 27

• 28

• 29

• 30

• 31

se

Feedback End Block

Page 62: toddlers, infants, newborns

se

Previous Next Lab Values. Notes Calculator.

Item: 6 of 31 M 1- Mark

PD -dependent congenital heart disease

• Coarctation of the aorta • D-transposition of the great arteries • Hypoplastic left heart syndrome • Total anomalous pulmonary venous connection • Tricuspid atresia

O usrALEWorfd, L LC

This patient's hypoxia that fails to improve with inhaled oxygenation is concerning for a congenital heart defect An oxygen saturation of 70%-90% and central cyanosis is expected in patients with cyanotic heart disease due to mixing of deoxygenated blood through the cardiac defect Many types of cyanotic heart disease depend on a patent ductus arteriosus (FDA) to provide vital blood flow between the pulmonary and systemic circulations (Table). Normally, the ductus arteriosus constricts and is nonfunctional around day 3 of life_ Closure of the ductus arteriosus in patients with FDA-dependent defects could cause severe hypoxia, shock, and severe metabolic (lactic) acidosis. Maintaining patency can be potentially life-saving; prostaglandin El is a vasodilator used to prevent ductus arteriosus closure in such infants until definitive surgery can be performed.

(Choice A) Loop diuretics are indicated in congenital heart disease complicated by volume overload (eg, pulmonary edema, sacral edema). This patient has no signs of volume overload.

(Choice B) Positive-pressure ventilation and oxygen supplementation can cause acute decompensation in newborns with FDA-dependent defects by changing the pressure and physiology of the cardiopulmonary system. Inspired oxygen stimulates FDA constriction, which can precipitate clinical deterioration. Other than hypoxemia and cyanosis, this patient's respiratory status is stable and does not warrant intubation.

(Choice C) Indomethacin should be avoided as it is a potent prostaglandin inhibitor that would close the ductus arteriosus.

(Choice 0) Beta blockers are used to treat arrhy-thmias and heart failure associated with congenital heart disease. This patient has no evidence of heart failure or arrhythmia.

(Choice F) Anemia must be very severe before it causes systemic hypoxemia, and it never causes •-•

• 1 • 2

• a 4

5

6

7

▪ 8 ▪ 9

• 10

• 11

• 12

• 13

• 14

• 15

• 16

• 17

• 18

• 19

• 20

• 21

• 22

• 23

• 24

• 25

• 26

• 27

• 28

• 29

• 30

• 31

Feedback End Block

Page 63: toddlers, infants, newborns

Item: 6 of 31

F' Mark -<=1

.4111°. Previous

Next

Lab Values. Notes

Calculator.

ulosure OT tne auctus arteriosus in patients \vim rum-dependent aerects could cause severe nypoxia, shock, and severe metabolic (lactic) acidosis. Maintaining patency can be potentially life-saving; prostaglandin El is a vasodilator used to prevent ductus arteriosus closure in such infants until definitive surgery can be performed.

(Choice A) Loop diuretics are indicated in congenital heart disease complicated by volume overload (eg, pulmonary edema, sacral edema). This patient has no signs of volume overload.

(Choice B) Positive-pressure ventilation and oxygen supplementation can cause acute decompensation in newborns with FDA-dependent defects by changing the pressure and physiology of the cardiopulmonary system. Inspired oxygen stimulates FDA constriction, which can precipitate clinical deterioration. Other than hypoxemia and cyanosis, this patient's respiratory status is stable and does not warrant intubation.

(Choice C) Indomethacin should be avoided as it is a potent prostaglandin inhibitor that would close the ductus arteriosus.

(Choice 13) Beta blockers are used to treat arrhythmias and heart failure associated with congenital heart disease_ This patient has no evidence of heart failure or arrhythmia.

(Choice F) Anemia must be very severe before it causes systemic hypoxemia, and it never causes cyanosis_ Red blood cell transfusions should not be given preemptively without evidence of severe anemia

Educational objective: Prostaglandin El infusion maintains patency of the ductus arteriosus. It is potentially life-saving in patients with ductus-dependent congenital heart disease until definitive surgery can be performed. Excessive inspired oxygen and indomethacin should be avoided as these interventions constrict the ductus arteriosus.

References:

1. Reappraisal of the prostaglandin El dose for early newborns with patent ductus arteriosus-dependent pulmonary circulation.

2. Prostaglandin El treatment in patent ductus arteriosus dependent congenital heart defects.

3. To intubate or not to intubate? Transporting infants on prostaglandin El.

Copyright @ LIWorld Last updated: [11/26/2014]

• 1

• 2

• a • 4

• 5

6

7

▪ 8

▪ 9

• 10

• 11

• 12

• 13

• 14

• 15

• 16

• 17

• 18

• 19

• 20

• 21

• 22

• 23

• 24

• 25

• 26

• 27

• 28

• 29

• 30

• 31

Feedback End Block

Page 64: toddlers, infants, newborns

8

▪ 9

• 10

• -1 -1

• 12

• 13

• 14

• 15

• 16

• 17

• -18

• 19

• 20

• 21

• 22

• 23

• 24

• 25

• 26

• 27

• 28

• 29

• 30

• 31

Item: 6 of 31 Mark Next Notes Previous Calculator Lab Values.

Patent ductus arteriosus

Media Exhibit

Patent ductus arteriosus

Left common carotid

Left subclavian artery

Brachiocephalic artery

Patent

ductuS arteriosus

se

Feedback End Block

Page 65: toddlers, infants, newborns

2

a 4

8 ▪ 9

• 10

• -1-1

• 12

• 13

• 14

• 15

• 16

• 17

• -18

• 19

• 20

• 21

• 22

• 23

• 24

• 25

• 26

• 27

• 28

• 29

• 30

• 31

RI I-Mark Item: 6 of 31 Next Notes Previous Calculator Lab Values.

Media Exhibit

Patent ductus arteriosus

Brachiocephalic artery

Patent ductus arteriosus

Pulmonary artery

COUSIALEVitixIckLIC V

a End Block

Feedback

Page 66: toddlers, infants, newborns

se

• 1

• 2

3

• 4

▪ 5

6

7

8

▪ 9

• 10

• 11

• 12

• 13

• 14

• 15

• 15

• 17

• 18

• 19

• 20

• 21

• 22

• 23

• 24

• 25

• 25

• 27

• 28

• 29

• 30

• 31

Item: 7 of 31 F' Mark Previous

Next

Lab Values. Notes

Calculator.

A 17-year-old female is brought to the primary care physician by her mother, who is concerned because her daughter has lost 10 lbs (4_5 kg) in the last two months. The mother says that her daughter has suddenly become a "picky eater" since starting college and that she often refuses to eat now, insisting that she is "not hungry.' Friends of the girl have confided that she is behaving strangely at school. When questioned about these concerns, the girl denies having any problems and says that she feels just fine_ Her height is 5'2" (157.5 cm) and her weight is 100 lbs (45 kg). Physical examination reveals erythema of her turbinates and nasal septum. Which of the following is the most likely diagnosis?

A. Anorexia nervosa [6°A]

B. Bulimia nervosa [4%]

C. Normal adolescence [1%]

ID. Cocaine abuse [82%]

E Eating disorder, not otherwise specified [4%]

F. Marijuana abuse [1%]

Explanation:

Individuals who abuse stimulants such as cocaine often present with strange behavior (eg, euphoria) and weight loss secondary to decreased appetite. The diagnostic hallmark in this scenario is erythema of the turbinates and nasal septum, which is a common finding in individuals who snort cocaine.

(Choices A, B, C, and E) Erythema of the turbinates and nasal septum is not associated with normal adolescent development or with any of the eating disorders. Moreover, an eating disorder diagnosis would require the presence of other features, such as amenorrhea, excessive use of diuretics or laxatives, vigorous exercising, a history of binging and purging, or signs of forceful induced vomiting.

(Choice F) Marijuana abuse is usually associated with an increase in appetite, not a decrease. Moreover, while injected conjunctivae are commonly seen in those who use marijuana, erythema of the turbinates and nasal septum would not be expected_

Educational Objective: Cocaine abuse should be suspected in an individual presenting with weight loss, behavioral changes, and

Feedback End Block

Page 67: toddlers, infants, newborns

• 1

• 2

3

• 4

▪ 5

6

7

8

▪ 9

• 10

• 11

• 12

• 13

• 14

• 15

• 15

• 17

• 18

• 19

• 20

• 21

• 22

• 23

• 24

• 25

• 25

• 27

• 28

• 29

• 30

• 31

daughter has lost 10 lbs (4_5 kg) in the last two months. The mother says that her daughter has suddenly become a "picky eater" since starting college and that she often refuses to eat now, insisting that she is "not hungry." Friends of the girl have confided that she is behaving strangely at school. When questioned about these concerns, the girl denies having any problems and says that she feels just fine_ Her height is 5'2" (157.5 cm) and her weight is 100 lbs (45 kg). Physical examination reveals erythema of her turbinates and nasal septum. Which of the following is the most likely diagnosis?

A. Anorexia nervosa [6%]

B. Bulimia nervosa [4%]

C. Normal adolescence [1%]

▪ ID. Cocaine abuse [82%]

E Eating disorder, not otherwise specified [4%]

• Marijuana abuse [1%]

Explanation:

Individuals who abuse stimulants such as cocaine often present with strange behavior (eg, euphoria) and weight loss secondary to decreased appetite. The diagnostic hallmark in this scenario is erythema of the turbinates and nasal septum, which is a common finding in individuals who snort cocaine.

(Choices A, B, C, and E) Erythema of the turbinates and nasal septum is not associated with normal adolescent development or with any of the eating disorders. Moreover, an eating disorder diagnosis would require the presence of other features, such as amenorrhea, excessive use of diuretics or laxatives, vigorous exercising, a history of binging and purging, or signs of forceful induced vomiting.

(Choice F) Marijuana abuse is usually associated with an increase in appetite, not a decrease. Moreover, while injected conjunctivae are commonly seen in those who use marijuana. erythema of the turbinates and nasal septum would not be expected_

Item: 7 of 31 F' Mark Previous

Next

Lab Values. Notes

Calculator.

Educational Objective: Cocaine abuse should be suspected in an individual presenting with weight loss, behavioral changes, and erythema of the turbinates and nasal septum.

Copyright © UWorld Last updated: [8/14/2014]

Feedback End Block

Page 68: toddlers, infants, newborns

A 2-year-old child is brought by an anxious mother to the emergency department because of difficulty in breathing for the past 1 hours. The child has a history of runny nose that started 3 days ago. He suddenly developed a progressively increasing difficulty in breathing. The mother reports that the child "sounds like a barking dog when he coughs." The child's temperature is 373' C (99.9° F), respirations are 30/min, blood pressure is 901&Imm Hg, and pulse rate is 104/min. He is coughing and using his accessory muscles of respiration. His oxygen saturation is 98% on 2L of oxygen. He is diagnosed with laryngotracheobronchitis, started on humidified oxygen, and kept in an upright position_ After 20 minutes, he becomes more irritable, his oxygen saturation drops to 92%, respirations increase to 40/min, and pulse increases to 120/min. His blood pressure and temperature are unchanged_ What is the most appropriate next step in the management of this patient?

A. Continue the same therapy [4%]

B. Take the patient to the operating room [6%]

C. Consult the anesthesiologist [4%]

ID_ Intubate the patient in the emergency room [36%]

▪ E Start racemic epinephrine [50%]

Explanation:

Always give a trial of epinephrine in cases of croup before intubation. Controlled trials have shown that a trial of racemic epinephrine should precede any invasive procedure in cases of croup, as this decreases the need for intubation_ Epinephrine acts by its alpha-adrenergic and beta-adrenergic effects_ In asthma, the alpha-adrenergic effect is beneficial by reducing bronchial secretions and mucosal edema. This similar process is thought to help patients with croup. In addition, the beta-adrenergic effect helps croup patients who are also wheezing by promoting smooth muscle relaxation.

Educational Objective: Racemic epinephrine decreases the need for intubation in patients with croup and should always be tried before any invasive procedure.

Copyright @ UWorld Last updated: [812212014]

1 2

▪ 3

▪ 4-

▪ 5

▪ 6

7

a

9

• 10

• 11

• 12

• 13

• 14

• 15

• 15

• 17

• 18

• 19

• 20

• 21

• 22

• 23

• 24

• 25

• 25

• 27

• 28

• 29

• 30

• 31

Item: 8 of 31 A V-Mark Previous

Next

Lab Values. Notes

Calculator.

Feedback End Block

Page 69: toddlers, infants, newborns

1 2

▪ 3

▪ 4-

▪ 5

▪ 6

▪ 7

• 8

9

• 10

• 11

• 12

• 13

• 14

• 15

• 15

• 17

• 18

• 19

• 20

• 21

• 22

• 23

• 24

• 25

• 25

• 27

• 28

• 29

• 30

• 31

Item: 9 of 31 F' Mark Previous Next

Lab Values. Notes Calculator.

A 6-year-old African-American child is brought in by his father for complaints of easy fatigability and pallor. These symptoms occurred after the son was treated with "some medication" for a recent diarrhea. Physical examination is normal except for pallor and multiple petechiae_ Laboratory values are as follows:

Hb 8.0 gicIL WBC 12,000/cmm Platelets 500001cl-rim Blood glucose 118 mgldL Serum Na 135 mEq/L Serum K 5.3 mEq!L Chloride 110 mEq/L Bicarbonate 18 mEq!L BUN 38 mgldL Serum creatinine 2.5 mgldL Total bilirubin 3 mgldL Direct bilirubin 0.5 mgldL PT 12 seconds APTT 30 seconds LDH 900 IU/L Reticulocyte count .6%

A peripheral blood smear reveals giant platelets and multiple schistocytes. What is the most likely underlying pathophysiology for this boy's pallor?

A. Sickle cell anemia [8%]

B. Thalassemia [3%]

C. Vitamin 812 deficiency [1%]

ID. Folate deficiency [1%]

▪ E Microangiopathic hemolytic anemia [63%]

F. Lead poisoning [1%]

G. Disseminated intravascular coagulation [3%]

H. Idiopathic thrombocytopenic purpura [18%] se

Feedback End Block

Page 70: toddlers, infants, newborns

1 2

▪ 3

▪ 4-

▪ 5

▪ 6

▪ 7

• 8

9

• 10

• 11

• 12

• 13

• 14

• 15

• 15

• 17

• 18

• 19

• 20

• 21

• 22

• 23

• 24

• 25

• 25

• 27

• 28

• 29

• 30

• 31

Item: 9 of 31 VI/lark Previous Next

Lab Values. Notes Calculator.

Reticulocyte count .6%

se

A peripheral blood smear reveals giant platelets and multiple schistocytes. What is the most likely underlying pathophysiology for this boy's pallor?

A. Sickle cell anemia [8%]

B. Thalassemia [3%]

C. Vitamin B12 deficiency [1%]

D. Folate deficiency [1%]

E Microangiopathic hemolytic anemia [63%]

• Lead poisoning [1%]

G. Disseminated intravascular coagulation [3%]

H. Idiopathic thrombocytopenic purpura [18%]

I. Renal failure [2%]

Explanation:

Hemolytic uremic syndrome is typically a disease of young children. It is usually preceded by an acute diarrheal illness due to the pathogens Escherichia cob. serotype 0157:H7, Strigella, Satmonelia, Yersinia, and Campylobacter species_ It is less commonly preceded by an upper respiratory infection_ GI bleeding is a common symptom_ Physical examination frequently reveals purpura and hypertension. The hallmark finding is microangiopathic hemolytic anemia Other typical features are acute renal failure, fever, oliguria (or anuria) and thrombocytopenia. The peripheral smear reveal schistocytes (which represent fragmented RBCs) and giant platelets_ Intravascular hemolysis results in elevated levels of lactate dehydrogenase (LDH) and indirect bilirubin, as well as reticulocyte count BUN and creatinine levels are markedly elevated. Moderate leukocytosis may be present The urine contains hemoglobin, hemosiderin, albumin, RBCs, WE s, and casts_

(Choice A) Sickle cell anemia is another type of hemolytic anemia. which can be easily ruled out in this case. due to the absence of sickle cells on the peripheral smear.

(Choice B) Patients with thalassemia present with hepatosplenomegaly (absent in this patient). Peripheral blood smear may reveal target cells, microcytosis, hypochromia_ and anisopoikilocytosis, but not

Feedback End Block

Page 71: toddlers, infants, newborns

2

▪ 3

▪ 4-

▪ 5

▪ 6

▪ 7

• 8

9 • 10

• 11

• 12

• 13

• 14

• 15

• 16

• 17

• 18

• 19

• 20

• 21

• 22

• 23

• 24

• 25

• 25

• 27

• 28

• 29

• 30

• 31

se

Hemolytic uremic syndrome is typically a disease of young children. It is usually preceded by an acute diarrheal illness due to the pathogens Escherichia col' serotype G157:H7, Sliigella, Salmonella, Yersiriia, and Gampylobacter species_ It is less commonly preceded by an upper respiratory infection_ GI bleeding is a common symptom_ Physical examination frequently reveals purpura and hypertension. The hallmark finding is microangiopathic hemolytic anemia Other typical features are acute renal failure, fever, oliguria (or anuria) and thrombocytopenia. The peripheral smear reveal schistocytes (which represent fragmented RE Cs) and giant platelets. Intravascular hemolysis results in elevated levels of lactate dehydrogenase (LDH) and indirect bilirubin, as well as reticulocyte count BUN and creatinine levels are markedly elevated. Moderate leukocytosis may be present The urine contains hemoglobin, hemosiderin, albumin, RBCs, WBCs, and casts_

(Choice A) Sickle cell anemia is another type of hemolytic anemia which can be easily ruled out in this case, due to the absence of sickle cells on the peripheral smear.

(Choice B) Patients with thalassemia present with hepatosplenomegaly (absent in this patient). Peripheral blood smear may reveal target cells, microcytosis, hypochromia, and anisopoikilocytosis, but not schistocytes_ BUN and creatinine levels are normal_

(Choice C) Vitamin B12 deficiency is characterized by macrocytosis and hypersegmented neutrophils on peripheral smear.

(Choice 0) Folate deficiency also presents with macrocytosis and hypersegmented polymorphonuclear leucocytes (PMNs).

(Choice F) Lead poisoning causes a microcytic anemia, with basophilic stippling of RECs on peripheral smear_ Furthermore, patients typically present with some GI symptoms_

(Choice G) Prothrombin time (PT) and activated partial thromboplastin time (aPTT) are prolonged in patients with DIC.

(Choice H) ITP is characterized by an isoiatedthrombocytopenia. On peripheral smear. the morphology of red cells is normal.

(Choice I) Bleeding in renal failure patients is usually due to dysfunctional platelets. It is very unusual to see schistocytes and a hemolytic picture from renal failure.

Educational Objective: i iQnpri- NI I in a rhilri inshn higQ rinrcinthf riarnwprinri frnm a rligrrhingl illnpcc qnri nrinQinnfQ Ifirith ari itA renal

Item: 9 of 31 V-Mark Previous

Next

Lab Values. Notes

Calculator.

Feedback End Block

Page 72: toddlers, infants, newborns

Item: 9 of 31 V-Mark Previous Next

Lab Values. Notes Calculator.

RBCs) and giant platelets_ Intravascular hemolysis results in elevated levels of lactate dehydrogenase (LDH) and indirect bilirubin, as well as reticulocyte count. BUN and creatinine levels are markedly elevated. Moderate leukocytosis may be present The urine contains hemoglobin, hemosiderin. albumin. RECs, WBCs, and casts_

(Choice A) Sickle cell anemia is another type of hemolytic anemia, which can be easily ruled out in this case, due to the absence of sickle cells on the peripheral smear.

(Choice B) Patients with thalassemia present with hepatosplenomegaly (absent in this patient). Peripheral blood smear may reveal target cells, microcytosis, hypochromia, and anisopoikilocytosis, but not schistocytes_ BUN and creatinine levels are normal_

(Choice C) Vitamin B12 deficiency is characterized by macrocytosis and hypersegmented neutrophils on peripheral smear.

(Choice 0) Folate deficiency also presents with macrocytosis and hypersegmented polymorphonuclear leucocytes (PMNs).

(Choice F) Lead poisoning causes a microcytic anemia, with basophilic stippling of RI3Cs on peripheral smear. Furthermore, patients typically present with some GI symptoms.

(Choice G) Prothrombin time (PT) and activated partial thromboplastin time (aPTT) are prolonged in patients with DIC.

(Choice H) ITP is characterized by an isolated thrombocytopenia. On peripheral smear. the morphology of red cells is normal.

(Choice I) Bleeding in renal failure patients is usually due to dysfunctional platelets. It is very unusual to see schistocytes and a hemolytic picture from renal failure.

Educational Objective: Suspect HUS in a child who has recently recovered from a diarrheal illness and presents with acute renal failure, microangiopathic hemolytic anemia, fever, thrombocytopenia and characteristic peripheral smear finding of schistocytes.

*Extremely high-yield question for the USMLEIll

Copyright © UWorld

Last updated: [8/22/2014]

1 2

▪ 3

▪ 4-

▪ 5

▪ 6

▪ 7

▪ 8

9

• 10

• 11

• 12

• 13

• 14

• 15

• 15

• 17

• 18

• 19

• 20

• 21

• 22

• 23

• 24

• 25

• 25

• 27

• 28

• 29

• 30

• 31

1

Feedback End Block

Page 73: toddlers, infants, newborns

An 8-year-old boy is brought to the emergency department by his mother. Two weeks ago, he developed a low-grade fever followed by a persistent cough. He occasionally has severe paroxysms of cough that are precipitated by eating and do not resolve with antitussive medications. On physical examination, extensive subcutaneous emphysema over the anterior chest is noted. What is the most appropriate next step in the management of this patient?

A. Chest x-ray [71%]

B. Throat culture [12%]

C. Ear examination [2%]

D. CT scan of head [2%]

E Blood cultures [5%]

Explanation: Severe coughing paroxysms may result in subcutaneous emphysema, wherein air leaks from the chest wall into the subcutaneous tissues due to the high intraalveolar pressure provoked by the cough. By a similar process, pneumothorax can occur; therefore, in such patients (i.e_, those with apparent subcutaneous emphysema secondary to severe coughing paroxysms), chest x-rays must be obtained emergently to rule out pneumothorax.

(Choice B) Cultures of the throat may be relevant in this case for diagnosis, but pneumothorax should still be ruled out first.

(Choice C) An ear infection could have caused this child's condition via orophar-yngeal transmission. but this information is still not as important as the absence/presence of a pneumothorax.

(Choice 0) In this child, there is no reason for a CT scan of the head since there are no neurologic findings.

(Choice E) Blood cultures are not warranted in this child. Throat cultures should be done first before blood cultures.

Educational Objective: In patients with apparent subcutaneous emphysema secondary to severe coughing paroxysms, chest x-rays must be obtained first to rule out pneumothorax.

1 2

▪ 3

▪ 4-

▪ 5

▪ 6

7

8

9

* 10

• 11

• 12

• 13

• 14

• 15

• 15

• 17

• 18

• 19

• 20

• 21

• 22

• 23

• 24

• 25

• 25

• 27

• 28

• 29

• 30

• 31

Item: 10 of 31

F' Mark Previous

Next

Lab Values. Notes

Calculator.

Feedback End Block

Page 74: toddlers, infants, newborns

Item: 10 of 31 F' Mark Previous Next

Lab Values. Notes Calculator.

An 8-year-old boy is brought to the emergency department by his mother. Two weeks ago, he developed a low-grade fever followed by a persistent cough. He occasionally has severe paroxysms of cough that are precipitated by eating and do not resolve with antitussive medications. On physical examination, extensive subcutaneous emphysema over the anterior chest is noted. What is the most appropriate next step in the management of this patient?

• A. Chest x-ray [77%]

B. Throat culture [12%]

C. Ear examination [2%]

D. CT scan of head [2%]

E Blood cultures [5%]

Explanation: Severe coughing paroxysms may result in subcutaneous emphysema, wherein air leaks from the chest wall into the subcutaneous tissues due to the high intraalveolar pressure provoked by the cough. By a similar process, pneumothorax can occur; therefore, in such patients (i.e., those with apparent subcutaneous emphysema secondary to severe coughing paroxysms), chest x-rays must be obtained emergently to rule out pneumothorax.

(Choice B) Cultures of the throat may be relevant in this case for diagnosis. but pneumothorax should still be ruled out first.

(Choice C) An ear infection could have caused this child's condition via oropharyngeal transmission, but this information is still not as important as the absence/presence of a pneumothorax.

(Choice 0) In this child, there is no reason for a CT scan of the head since there are no neurologic findings.

(Choice E) Blood cultures are not warranted in this child. Throat cultures should be done first before blood cultures.

Educational Objective: In patients with apparent subcutaneous emphysema secondary to severe coughing paroxysms, chest x-rays must be obtained first to rule out pneumothorax.

Copyright © UWorld Last updated: [8/22/2014]

1 2

▪ 3

▪ 4-

▪ 5

▪ 6

7

8

9

* 10

• 11

• 12

• 13

• 14

• 15

• 15

• 17

• 18

• 19

• 20

• 21

• 22

• 23

• 24

• 25

• 25

• 27

• 28

• 29

• 30

• 31

Feedback End Block

Page 75: toddlers, infants, newborns

1 2

▪ 3

▪ 4-

▪ 5

▪ 6

▪ 7

• 8

• 9

• 10

• 11

• 12

13

14

15

15

17

18

19

20

21

22

23

24

25

25

27

28

29

30

31

A 6-year-old girl is brought to the physician with a rash and joint pains. She had previously been healthy except for a sore throat a few weeks ago that resolved on its own. Last week, the patient developed pain in her knees. The pain resolved after a few days, but now her ankles and wrists are tender_ She has also developed a non-pruritic pink rash on her back. Temperature is 38.3 C (101 F), pulse is 85/min, and respirations are 20/min. On examination, there is pain and stiffness with manipulation of the wrists and ankles. A faint, erythematous, centrifugal rash is present on her trunk and proximal limbs. Lungs are clear and no murmur is heard_ Serum laboratory results are as follows:

Leukocytes

6,500/pL Neutrophils

56% Lymphocytes

33% Hemoglobin

12_5 gfdL Platelets

200,000/p L C-reactive protein

35 mg/dL Erythrocyte sedimentation rate

38 mmihr

Which of the following is the most likely diagnosis?

A. Acute lymphoblastic leukemia [1%]

• B. Acute rheumatic fever [64%]

C. Juvenile idiopathic arthritis [22%]

D. Henoch-SchOnlein purpura [10%]

E. Systemic lupus erythematosus [3%]

Explanation:

Acute rheumatic fever

Item: 11 of 31

'MY-Mark Previous Next

Lab Values, Notes Calculator.

Epidemiology • Age 5-15

• Twice as common in girls

• .hints (liiiriratriry arthritic) se

a Feedback

End Block

Page 76: toddlers, infants, newborns

1

2

▪ 3

▪ 4-

▪ 5

▪ 6

▪ 7

▪ 8

• 9

• 10

11

• 12

• 13

• 14

• 15

• 16

• 17

• -18

• 19

• 20

• 21

• 22

• 23

• 24

• 25

• 26

• 27

• 28

• 29

• 30

• 31

Previous Next . Lab Values. Notes Calculator.

Item: 11 of 31 F'Mark

Acute rheumatic fever - ,

Epidemiology • Age 5-15

• Twice as common in girls

Clinical features

.

Major

. • Joints (migratory arthritis)

• v (Carditis)

• Nodules (subcutaneous)

• Erythema marginatum

• Sydenham chorea

-

Minor

- • Fever

• Arthralgias

• Elevated erythrocyte sedimentation rate/C-reactive protein

• Prolonged PR interval

Late sequelae . .

Mitral regurgitationistenosis

Prevention Penicillin for group A streptococcal (Streptococcus pyogenes) pharyngitis

1 0 morld.corn

Acute rheumatic fever (ARF) is a complication of untreated group A streptococcal pharyngitis, which usually precedes the onset of rheumatic fever by 2-4 weeks. ARF is diagnosed clinically using the Jones criteria, which are split into major and minor criteria (Table). The diagnosis is based on evidence of a preceding group A streptococcal infection along with 2 major criteria, or 1 major plus 2 minor criteria

Thic ngtipntiQ nnIu2r-Fhritic artitharnm mmrninmilim fan priagninQr rant ninle rash inrith Qhqrn Prirl fpnapr gnri v

Feedback End Block

Page 77: toddlers, infants, newborns

a- 1

• 2 ▪ 3

• 4-

• 5

• 6

• 7

• 8

• 9

• 10

• 11

• 12

• 13

• 14

• 15

• 15

• 17

• 18

• 19

• 20

• 21

• 22

• 23

• 24

• 25

• 25

• 27

• 28

• 29

• 30

• 31

Item: 11 of 31

F' Mark Previous

Next

Lab Values. Notes

Calculator.

Acute rheumatic fever (ARF) is a complication of untreated group A streptococcal pharyngitis, which usually precedes the onset of rheumatic fever by 2-4 weeks. ARF is diagnosed clinically using the Jones criteria, which are split into major and minor criteria (Table). The diagnosis is based on evidence of a preceding group A streptococcal infection along with 2 major criteria, or 1 major plus 2 minor criteria.

This patient's polyarthritis, erythema marginatum (an evanescent pink rash with sharp edges), fever, and elevated acute phase reactants (C-reactive protein [CRP] and erythrocyte sedimentation rate [ESR]) satisfy the diagnostic criteria for ARF. Supportive laboratory findings include a positive streptococcal antigen test or elevated antistreptolysin 0 titer. Rheumatic fever can be prevented with penicillin treatment of streptococcal pharyngitis.

(Choice A) Acute lymphoblastic leukemia is a malignancy of white blood cells that causes lymphocytosis or lymphopenia along with anemia or thrombocytopenia. This patient has a normal complete blood count

(Choice Cl Juvenile idiopathic arthritis is diagnosed when arthritis is present for >6 weeks_ Systemic symptoms, including a rash, can be seen in children with juvenile idiopathic arthritis, but the arthritis is usually not migratory, as in this patient

(Choice 0) Henoch-SchOnlein purpura is a vasculitis that can cause a transient, migratory arthritis and rash. However, the rash is purpuric and the arthritis typically involves the lower-extremity joints (hip/knee/ankle).

(Choice El Systemic lupus erythematosus is an autoimmune disease that affects multiple organ systems. Arthritis and elevated CRP and ESR can be seen, but the rash is usually present across the cheeks (malar rash)_ Multiple organ system involvement (eg, hematologic, neurologic, renal) is needed for the diagnosis.

Educational objective: Rheumatic fever is a preventable complication of streptococcal pharyngitis. It is diagnosed clinically using the Jones criteria The major criteria include carditis, migratory polyarthritis, Sydenham chorea, subcutaneous nodules, and erythema marginatum.

References:

1. Acute rheumatic fever and streptococci: the quintessential pathogenic trigger of autoimmunity.

2. Diagnostic criteria of acute rheumatic fever.

3. Clinical characteristics and cardiac outcome of acute rheumatic fever in Italy in the last 15

Feedback End Block

Page 78: toddlers, infants, newborns

1 2

▪ 3

4

▪ 5

• 6

• 7

• 8

• 9

• 10

• 11

• 12

• 13

• 14

• 15

• 15

• 17

• 18

• 19

• 20

• 21

• 22

• 23

• 24

• 25

• 25

• 27

• 28

• 29

• 30

• 31

Item: 11 of 31

F'Mark Previous

Next

Lab Values. Notes

Calculator.

i streptococcal inrection along witn z major criteria, or major plus z minor criteria.

This patient's polyarthritis, erythema marginatum (an evanescent pink rash with sharp edges), fever, and elevated acute phase reactants (C-reactive protein [CRP] and erythrocyte sedimentation rate [ESR]) satisfy the diagnostic criteria for ARE. Supportive laboratory findings include a positive streptococcal antigen test or elevated antistreptolysin 0 titer. Rheumatic fever can be prevented with penicillin treatment of streptococcal pharyngitis.

(Choice A) Acute lymphoblastic leukemia is a malignancy of white blood cells that causes lymphocytosis or lymphopenia along with anemia or thrombocytopenia. This patient has a normal complete blood count

(Choice C) Juvenile idiopathic arthritis is diagnosed when arthritis is present for >6 weeks_ Systemic symptoms, including a rash, can be seen in children with juvenile idiopathic arthritis, but the arthritis is usually not migratory, as in this patient

(Choice 0) Henoch-Schbnlein purpura is a vasculitis that can cause a transient, migratory arthritis and rash. However, the rash is purpuric and the arthritis typically involves the lower-extremity joints (hip/knee/ankle).

(Choice E) Systemic lupus erythematosus is an autoimmune disease that affects multiple organ systems. Arthritis and elevated CRP and ER can be seen, but the rash is usually present across the cheeks (malar rash). Multiple organ system involvement (eg, hematologic, neurologic, renal) is needed for the diagnosis.

Educational objective: Rheumatic fever is a preventable complication of streptococcal pharyngitis. It is diagnosed clinically using the Jones criteria The major criteria include carditis, migratory polyarthritis. Sydenham chorea, subcutaneous nodules, and erythema marginatum.

References:

1. Acute rheumatic fever and streptococci: the quintessential pathogenic trigger of autoimmunity.

2. Diagnostic criteria of acute rheumatic fever.

3. Clinical characteristics and cardiac outcome of acute rheumatic fever in Italy in the last 15 years.

Copyright © LIWorld Last updated: [7/22/2014]

Feedback End Block

Page 79: toddlers, infants, newborns

Item: 12 of 31 .11 V- • Mark Previous Next

Lab Values. Notes Calculator.

An 8-month-old child is referred by a nurse practitioner due to "pale mucous membranes, irritability, and listlessness.' The stool examination is negative for occult blood, ova and parasites. Laboratory studies reveal:

Hemoglobin 6.0 MCHC 25% MCH 16.5 pg MCV 68 fl Reticulocytes 0.6% Platelets 230,000/mm3 Leukocyte count 5,5G8imm3

Neutrophils 56% Eosinophils 1% Lymphocytes 33% Monocytes 1G%

40 mcg/dL 460 mcg/dL (normal 300-350 mcgidL)

Percent saturation of transferrin Total serum bilirubin

The peripheral blood smear shows marked anisocytosis, microcytosis, hypochromia, and poikilocytosis. Which of the following is the most likely diagnosis?

A. Iron deficiency anemia [81%]

B. Sideroblastic anemia [12%]

C. Dimorphic anemia [3%]

D. Megaloblastic anemia [1%]

E Anemia of chronic disease [3%]

Explanation: se

I L r

1 2

▪ 3

▪ 4-

▪ 5

▪ 6

▪ 7

▪ 8

▪ 9

• 10

• 11

• 12

• 13

• 14

• 15

• 16

• 17

• 18

• 19

• 20

• 21

• 22

• 23

• 24

• 25

• 26

• 27

• 28

• 29

• 30

• 31

Serum Iron

TIBC

83%

0.9 mg/I:IL

Feedback End Block

Page 80: toddlers, infants, newborns

Item: 12 of 31

V-Mark Previous

Next

Lab Values. Notes

Calculator.

The peripheral blood smear shows marked anisocytosis, microcytosis, hypochromia, and poikilocytosis. Which of the following is the most likely diagnosis?

A. Iron deficiency anemia [81%]

Sideroblastic anemia [12%]

C. Dimorphic anemia [3%]

ID. Megaloblastic anemia [1%]

E Anemia of chronic disease [3%]

Explanation:

This patient has a microcytic anemia (RBC indices: decreased MCV and MCH), and the findings on the peripheral blood film (Le., hypochromic, microcytic erythrocytes) are suggestive of iron deficiency anemia. The characteristic laboratory findings of iron deficiency anemia are decreased serum iron level, decreased percent saturation (serum iron ITIBC), and increased TIBC. The most common cause in infants is inadequate dietary intake of iron whereas in older children and adults it is chronic blood loss. The typical setting involves small children with a diet consisting largely of milk, and without supplementation of iron-fortified food products during the early years of development Other causes of iron deficiency anemia are bleeding and parasitic infections. Hemolysis can also cause microcytic anemia (Le_, PNH); however, the serum bilirubin level is typically elevated in such patients.

(Choice B) Sideroblastic anemia is characterized by increased serum iron levels and normal TIBC.

(Choice C) In dimorphic anemia, two distinct forms of red cells are circulating.

(Choice ID) Megaloblastic anemia is characterized by an elevated MCV, elevated MCH, and normal MCHC.

(Choice El Anemia of chronic disease is associated with decreased TIBC_

Educational objective: The characteristic laboratory findings of iron deficiency anemia are decreased serum iron level, decreased percent saturation (serum iron ITIBC), and increased TIBC.

Copyright @ UWorld Last updated: [8/17/2014]

1 2

▪ 3

▪ 4-

▪ 5

▪ 6

▪ 7

▪ 8

▪ 9

• 10

• 11

• 12

• 13

• 14

• 15

• 16

• 17

• 18

• 19

• 20

• 21

• 22

• 23

• 24

• 25

• 26

• 27

• 28

• 29

• 30

• 31

Feedback End Block

Page 81: toddlers, infants, newborns

2

3

4

• 5

• 6

• 7

• 8

• 9

• 10

• 11

• 12

13

• 14

• 15

• 16

• 17

• 18

• 19

• 20

• 21

• 22

• 23

• 24

• 26

• 26

▪ Z. '7

• 28

• 29

• 30

Feedback End Block

Item: 13 of 31

F'Mark Previous

Next . Lab Values. Notes

Calculator.

A 14-year-old African American girl comes to the emergency department with a low-grade fever and severe right-sided thigh pain. Her pain started yesterday and is worsening despite ibuprofen and oxycodone. She is agitated and requests quick pain relief. The patient has had similar episodes every summer that required hospitalization for intravenous pain management. but she has not received blood transfusions. Her vaccinations are up to date and she takes an iron-free multivitamin. Her brother had similar pain episodes and died of a bloodstream infection at age 4. Her temperature is 38 C (100_4 F), blood pressure is 120/80 mm Hg, pulse is 120/min. and respirations are 18/min. Which of the following would most likely be seen on this patient's peripheral blood smear?

A. Blast cells [2%]

B. Burr cells [3°,'0]

C. Eosinophilia [1°/0]

D. Howell-Jolly bodies [88%]

E. Pappenheimer bodies [3%]

F. Thrombocytopenia [29.6]

Explanation:

Sickle-cell anemia

Inheritance •

Autosomal recessive

Clinical

presentation

. ..

• Hemolytic anemia

• Acute vasoocclusive pain crises

Laboratory findings

• 4, Hematocrit, IN reticulocyles, t lactate dehydrogenase,

1h unconjugated bilirubin

• Peripheral smear: Sickled red cells_ Howell-Jolly bodies

Page 82: toddlers, infants, newborns

▪ 2

▪ 3

▪ 4-

▪ 5

▪ 6

▪ 7

▪ 8

• 0

• -10

• -1-1

• 12

13.

• 14

• 15

• 16

• 17

• 18

• 19

• 20

• 21

• 22

23

• 24

25

• 26

▪ Z. '7

• 28

29

• 30

31

.4111°. Lab Values. Notes Calculator.

Feedback

End Block,

v

: 13 of 31 r Flilark Previous , Next

Sickle-cell anemia

Inheritance Autosomal recessive

Clinical presentation

• Hemolytic anemia

• Acute vasoocclusive pain crises

Laboratory findings

• 4, Hematocrit, t reticulocytes, t lactate dehydrogenase, t unconjugated bilirubin

• Peripheral smear: Sickled red cells, Howell-Jolly bodies

Management

Maintenance

• Vaccination

• Penicillin (until age 5)

• Folic acid supplementation

• Hydroxyurea (for patients with recurrent vasoocclusive events)

— ,

Acute pain crises

• Hydration

• Analgesia

• Transfusion

Complications

• Infection with encapsulated organisms

• Streptococcal pneumoniae bacteremia & pneumonia • Salmonella osteomyelitis

• Osteonecrosis

• Acute chest syndrome

Page 83: toddlers, infants, newborns

1

2

▪ 3

▪ 4-

▪ 5 ▪ S

▪ 7

▪ 8 ▪ 9

• 10

• 11

• 12

13

- 14

• 15

• 16

- 17

• 18

• 19

• 20

• 21

• 22

• 23

• 24

• 25

• 26

• 27

• 28

• 29

• 30

• 31

Complications

• intection witn encapsuiateo organisms

• Streptococcal pneumoniae bacteremia & pneumonia

• Salmonella osteomyelitis

• Osteonecrosis

• Acute chest syndrome

• Pulmonary hypertension

• Pigment gallstonesicholecystitis

• Priapism

• Stroke

OUSIMLEWorldr U_C

This patient most likely has sickle cell anemia (SCA) based on her ethnicity, family history, and characteristic presentation. SCA is a common hemoglobinopathy that manifests as recurrent pain crises. The pain is caused by intermittent vascular occlusion and ischemia and can be precipitated by weather changes and dehydration (eg, hot summer weather).

Sickled red cells, polychromasia, and Howell-Jolly bodies are typically seen on peripheral smear_ Polychromasia reflects reticulocytosis. Howell-Jolly bodies are nuclear remnants that are usually removed by the spleen_ Patients with SCA have markedly decreased splenic function due to damage from recurrent splenic sequestration (pooling and congestion of sickled red cells in the spleen) and infarction_ The result is Howell-Jolly bodies in the peripheral blood.

(Choice A) Circulating blasts are rare in normal patients but are elevated in those with acute leukemia.

(Choice B) Burr cells are spiculated red blood cells of similar size and with regularly spaced projections. They are seen most commonly in uremia or as an artifact of preparation_

(Choice C) Allergies, parasitic infections, adrenal insufficiency, and several other conditions increase the number of circulating eosinophils, but SCA does not

(Choice E) Pappenheimer bodies are iron-containing inclusion bodies found in peripheral red blood cells. They result from phagosomes that engulf excessive amounts of iron and are typically seen in sideroblastic anemia.

Item: 13 of 31 IMIY- Mark Previous Next

Lab Values. Notes Calculator.

Feedback End Block

Page 84: toddlers, infants, newborns

1 2

▪ 3

▪ 4-

▪ 5

• 6

• 7

• 8

• 9

• 10

• 11

• 12

13

- 14

Item: 13 of 31

F'Mark Previous

Next

Lab Values. Notes

Calculator.

presentation. SCA is a common hemoglobinopathy that manifests as recurrent pain crises. The pain is caused by intermittent vascular occlusion and ischemia and can be precipitated by weather changes and dehydration (eg, hot summer weather).

Sickled red cells, polychromasia, and Howell-Jolly bodies are typically seen on peripheral smear_ Polychromasia reflects reticulocytosis. Howell-Jolly bodies are nuclear remnants that are usually removed by the spleen_ Patients with SCA have markedly decreased splenic function due to damage from recurrent splenic sequestration (pooling and congestion of sickled red cells in the spleen) and infarction_ The result is Howell-Jolly bodies in the peripheral blood.

(Choice A) Circulating blasts are rare in normal patients but are elevated in those with acute leukemia.

• 15

• 16

- 17

• 18

• 19

• 20

• 21

• 22

• 23

• 24

• 25

• 26

• 27

• 28

• 29

• 30

• 31

(Choice B) Burr cells are spiculated red blood cells of similar size and with regularly spaced projections. They are seen most commonly in uremia or as an artifact of preparation_

(Choice C) Allergies, parasitic infections, adrenal insufficiency, and several other conditions increase the number of circulating eosinophils, but SCA does not

(Choice El Pappenheimer bodies are iron-containing inclusion bodies found in peripheral red blood cells. They result from phagosomes that engulf excessive amounts of iron and are typically seen in sideroblastic anemia

(Choice F) Thrombocytopenia may present with petechiae, ecchymosis, purpura, and cutaneous or mucosal bleeding. It is not a feature of SCA.

Educational objective: Acute pain crises due to vasoocclusion and ischemia are the most common manifestation of sickle cell anemia Howell-Jolly bodies are typically seen on peripheral smear and reflect splenic dysfunction_

References:

1. Health supervision for children with sickle cell disease.

2. Hyposplenism: comparison of different methods for determining splenic function.

3. Sickle-cell disease.

Copyright @ LIWorld Last updated: [10/19/2014]

Feedback End Block

Page 85: toddlers, infants, newborns

2

▪ 3

▪ 4-

▪ 5

▪ 6

▪ 7

▪ 8 ▪ 9

• 10

• 11

• 12

• 13

A 4-week-old full-term boy is brought to the emergency department with persistent, forceful vomiting immediately after feeds for the past several days. The vomitus is always nonbloody and nonbilious. Vital signs show tachycardia, hypotension, and bradypnea. Examination shows a thin infant with a sunken anterior fontanel and dry mucous membranes. His abdomen is soft, nontender, and nondistended. An olive-shaped mass is palpated in the right upper quadrant Which of the following laboratory findings would be expected in this patient?

pH PaCO2 HCO3 Ft I

Item: 14 of 31

V/lark Previous

Next

Lab Values. Notes

Calculator.

• 14

• 15

• 16

• 17

• 18

• 19

• 20

• 21

• 22

• 23

• 24

• 25

• 26

• 27

• 28

• 29

• 30

• 31

Explanation:

Laboratory derangements in pyloric stenosis

A. 4. 1 1 4. 1 [6%]

B. i t I Normal Normal [116]

C. Normal Normal Normal Normal Normal [0%]

D. .- i [90%]

E. i [2%]

Vomiting

Loss of gastric HCI Loss of Naa r H2O

se

Feedback End Block

Page 86: toddlers, infants, newborns

Item: 14 of 31 .41°111.. Notes Next

1 1- Mark Lab Values. Calculator. Previous

Aldosterone

1

HCO3 reabsorption

f K# secretion

Hypokalennia H* secretion

Laboratory derangements in pyloric sten osis

Vomiting

Loss of NaCl, H2O

Tir Renal perfusion pressure

Angiotensin II

Maintenance of metabolic alkalosis

@ LPSMLEWOdd, LLC

Thic hictrInf nIntrcir r etncictant inrith 1-7arnarFrrInhir rarInrir c-i-anncic Thic se

Initiation of metabolic alkalosis,

hypochioremia

Hypovoiernia

Loss of gastric HCI

1

2

▪ 3

▪ 4-

▪ 5

▪ 6

▪ 7

▪ 8

• 10

• -1 -1

• 12

• 13

14

• 15

• 16

• 17

• -18

• 19

• 20

• 21

• 22

• 23

• 24

• 25

• 26

• 27

• 28

• 29

• 30

• 31

Feedback End Block

Page 87: toddlers, infants, newborns

Item: 14 of 31

V/lark Previous

Next

Lab Values. Notes

Calculator.

I metabolic aiKainsm

LEHLEVIlo ad, LLC

This infant's history and physical examination are consistent with infantile hypertrophic pyloric stenosis. This condition typically presents in first-born boys age 3-6 weeks. The hypertrophied pylorus muscle obstructs the gastric outlet, resulting in projectile, nonbilious emesis and an olive-shaped abdominal mass. Diagnosis is confirmed by abdominal ultrasonography. %,,.hich shows a thick, elongated pylorus_

If diagnosis is delayed, protracted vomiting can result in hypochioremic, hypokaremic metabolic alkalosis (diagram). Normally, gastric acid enters the duodenum and stimulates pancreatic secretion of bicarbonate. In pyloric stenosis, hydrochloric acid is lost in the emesis and cannot enter the duodenum to stimulate pancreatic bicarbonate secretion_ Dehydration and hypovolemia also cause contraction alkalosis_ Hypovolemia activates the renin-angiotensin-aldosterone system in attempt to retain water at the expense of hydrogen ions_ Some potassium is also lost in the emesis, and hypokalemia is exacerbated as the kidneys secrete potassium in response to aldosterone. The respiratory system responds with compensatory hypoventilation, resulting in secondary respiratory acidosis.

Pyloromyotomy is the treatment of choice. However, prior to surgery_ laboratory derangements should be normalized with intravenous rehydration as alkalosis increases risk of postoperative apnea.

(Choice A) Primary metabolic acidosis is characterized by decreased pH, PaCO2, and bicarbonate.

Metabolic acidosis can occur when vomiting is accompanied by diarrhea as significant amounts of bicarbonate are lost in the stool. Potassium and chloride are also lost in diarrhea.

(Choice B) Primary respiratory acidosis is characterized by decreased pH with elevated PaC0.2 and

bicarbonate. This occurs in respiratory depression (eg, narcotic overdose) and hypoventilation syndromes (eg, neuromuscular disease). However, the PaCO.i. retention in pyloric stenosis is secondary to the primary

metabolic alkalosis disturbance.

(Choice C) A normal acid-base status is characterized by pH of T35-7_45, PaCO.i. of 35-45 mm Hg, and

bicarbonate of 22-2.6 mEdi_ Normal laboratory values are unlikely in this patient with prolonged vomiting, dehydration, and abnormal vital signs.

(Choice E) Hyperkalemia and hyperchloremia are not seen in pyloric stenosis.

Educational objective: Pyloric stenosis presents at age 3-5 weeks with projectile, nonbilious vomiting that occurs after each

se

1 2

▪ 3 ▪ 4- ▪ 5 ▪ 6 ▪ 7 ▪ 8 ▪ 9

• 10 • 11 • 12 • 13 • 14 • 15 • 16 • 17 • 18 • 19 • 20 • 21 • 22 • 23 • 24 • 25 • 26 • 27 • 28 • 29 • 30 • 31

Feedback End Block

Page 88: toddlers, infants, newborns

Item: 14 of 31 Al II -Mark Previous Next

Lab Values. Notes Calculator.

pancreatic bicarbonate secretion_ Dehydration and hypovolemia also cause contraction alkalosis_ Hypovolemia activates the renin-angiotensin-aldosterone system in attempt to retain water at the expense of hydrogen ions_ Some potassium is also lost in the emesis, and hypokalemia is exacerbated as the kidneys secrete potassium in response to aldosterone. The respiratory system responds with compensatory hypoventilation, resulting in secondary respiratory acidosis.

Pyloromyotomy is the treatment of choice. However, prior to surgery, laboratory derangements should be normalized with intravenous rehydration as alkalosis increases risk of postoperative apnea.

(Choice A) Primary metabolic acidosis is characterized by decreased pH, PaCO3.. and bicarbonate.

Metabolic acidosis can occur when vomiting is accompanied by diarrhea as significant amounts of bicarbonate are lost in the stool. Potassium and chloride are also lost in diarrhea.

(Choice B) Primary respiratory acidosis is characterized by decreased pH with elevated PaCO2 and

bicarbonate. This occurs in respiratory depression (eg, narcotic overdose) and hypoventilation syndromes (eg, neuromuscular disease). However, the PaCO2 retention in pyloric stenosis is secondary to the primary

metabolic alkalosis disturbance.

(Choice C) A normal acid-base status is characterized by pH of 7.35-7.45. PaCO2 of 35-45 mm Hg, and

bicarbonate of 22-2€ mEcilL_ Normal laboratory values are unlikely in this patient with prolonged vomiting, dehydration, and abnormal vital signs.

(Choice El Hyperkalemia and hyperchloremia are not seen in pyloric stenosis.

Educational objective: Pyloric stenosis presents at age 3-5 weeks with projectile, nonbilious vomiting that occurs after each feed. Protracted vomiting produces a hypochloremic, hypokalemic metabolic alkalosis that should be corrected prior to pyloromyotomy.

References:

1. Infantile hypertrophic pyloric stenosis.

2. Electrolyte profile of pediatric patients with hypertrophic pyloric stenosis.

Copyright © LIWorld Last updated: [1D/27/2014]

1

2

▪ 3

▪ 4-

▪ 5

▪ 6

▪ 7

• 8

• 9

• 10

• 11

• 12

• 13

• 14 • 15

• 16

• 17

• 18

• 19

• 20

• 21

• 22

• 23

• 24

• 25

• 26

• 27

• 28

• 29

• 30

• 31

Feedback End Block

Page 89: toddlers, infants, newborns

Feedback End Block

• -1

▪ 2

▪ 3

▪ 4-

▪ 5

▪ 6

▪ 7

▪ 8

▪ 9

• 10

• -1 -1

• 12

• 13

14

• 15

• 16

• 17

• -18

• 19

• 20

• 21

• 22

• 23

• 24

• 25

• 26

• 27

• 28

• 29

• 30

• 31

Item: 14 of 31 .111M Mark Previous Next Lab Values

FF. Notes Calculator

Media Exhibit

Hypertrophic pyloric stenosis Hypertrophic pyloric stenosis II Hypertrophic pyloric stenosis Hypertrophic pyloric stenosis

Page 90: toddlers, infants, newborns

Next FF. Notes Previous

.111M Mark Calculator Lab Values

Item: 14 of 31

Media Exhibit 4-

▪ 5

▪ 6

▪ 7

▪ 8

▪ 9

• 10

• 11

• 12

• 13

14

• 15

• 16

• 17

• -18

• 19

• 20

• 21

• 22

• 23

• 24

• 25

• 26

• 27

• 28

• 29

• 30

• 31

ypertrophic pyloric stenosis Hypertrophic pyloric stenosis Hypertrophic pyloric stenosis Hypertrophic pyloric stenosis

• -1 1

• ')

▪ 3

Feedback End Block

Page 91: toddlers, infants, newborns

Feedback End Block

Item: 14 of 31 .111M Mark Previous Next Lab Values

FF. Notes Calculator

Media Exhibit

I Hypertrophic pyloric stenosis II Hypertrophic pyloric stenosis I Hypertrophic pyloric stenosis I Hypertrophic pyloric stenosis I

• -1

▪ 2

▪ 3

▪ 4-

▪ 5

▪ 6

▪ 7

▪ 8

• 10

• -1-1

• 12

• 13

• 14

• 15

• 16

• 17

• -18

• 19

• 20

• 21

• 22

• 23

• 24

• 25

• 26

• 27

• 28

• 29

• 30

• 31

Page 92: toddlers, infants, newborns

Item: 14 of 31 .111M Mark Previous Next Lab Values Notes Calculator

uyva u up IL pyloric 3LC 6313 II rpm uLy IL pyloric stenosisII Hyperirophic pyloric 3LC L3.13 1 Hurwr L vT i- pylons stenosis

Feedback End Block

• -1

• ')

▪ 3

4- Media Exhibit

V

▪ 5

• 6

• 7

▪ 8

• 10

• 1-1

• 12

• 13

• 14

• 15

• 16

• 17

• -18

• 19

• 20

• 21

• 22

• 23

• 24

• 25

• 26

• 27

• 28

• 29

• 30

• 31

Page 93: toddlers, infants, newborns

se

2

▪ 3

▪ 4-

▪ 5

▪ 6

▪ 7

▪ 8 ▪ 9

• 10

• 11

• 12

• 13

• 14

15

• 16

• 17

• 18

• 19

• 20

• 21

• 22

• 23

• 24

• 25

• 25

• 27

• 28

• 29

• 30

• 31

1

An 11-year-old boy is brought to the physician for evaluation of scoliosis. His mother noticed that he always seems to be leaning even when he tries to sit or stand upright The boy has a history of myopia and upward lens dislocation for which he wears corrective glasses_ He otherwise has been healthy and doing well in school_ His father had scoliosis and vision problems and died from 'heart problems" last year. Physical examination shows a boy with a long face, high arched palate with crowded teeth, and upward dislocation of the lens. He has a tall stature for his age; long arms and legs with minimal subcutaneous fat; long, thin fingers; and a prominent sternum. Joint hypermobility, skin hyperelasticity, and 15 degrees of thoracic scoliosis are seen. A diastolic murmur is heard in the aortic area. Which of the following is the most likely etiology of this patient's condition?

A. Cystathionine synthase deficiency [416]

B. Defective collagen production [1094]

• C. Mutation of the fibrillin-1 gene [81%]

D. Mutation of the fibrillin-2 gene [4136]

E Nondisjunction resulting in an extra X chromosome [1%]

Explanation:

Marian syndrome

Item: 15 of 31

F' Mark -4(1

UP. Previous

Next

Lab Values, Notes

Calculator.

Ectopia lentis

t Arm-to-height ratio

vigor-- -

A Am. ■ Feedback End Block

Page 94: toddlers, infants, newborns

Joint hypermobility

Arachnodactyly

2

▪ 3

▪ 4-

▪ 5

▪ 6

▪ 7

▪ 8

▪ 9

• 10

• -11

• 12

• 13

• 14

15

• 16

• 17

• -18

• 19

• 20

• 21

• 22

• 23

• 24

• 25

• 26

• 27

• 28

• 29

• 30

• 31

Item: 15 of 31

r. Mark -4(1

Previous

Next

Lab Values, Notes

Calculator.

Marfan syndrome

f-,illil— Ectopia lentis

1160041,

t Arm-to-height ratio

Scoliosis or kyphosis

Breastbone that dips inward or

protrudes outward

Upper-to-lower-body-segment ratio

Taller & slender build

a Feedback

End Block

Page 95: toddlers, infants, newborns

Item: 15 of 31 Mark Previous Next Lab Values. Notes Calculator.

Taller & slender build

Arachnodactyly

Flat feet

1 2

▪ 3

▪ 4-

▪ 5

▪ 6

▪ 7

▪ 8

▪ 9

• 10

• 11

• 12

• 13

• 14

15

• 16

• 17

• 18

• 19

• 20

• 21

• 22

• 23

• 24

• 25

• 25

• 27

• 28

• 29

• 30

• 31

0415MLEWorld, L LC

This patient's family history and physical appearance are very characteristic of Marian syndrome. Marian syndrome is an autosomal dominant disorder of the fibrillin-1 gene that results in systemic weakening of connective tissue. Classic skeletal manifestations include joint hypermobility, skin hyperelasticity, long fingers (arachnodactyly ["thumb sign"]), pectus excavatum, and scoliosisikyphosis_ The face is long, the palate has a high arch, and the teeth are crowded. Lens dislocation (ectopia lentil), iridodonesis (a rapid contraction and dilation of the iris), and myopia (from elongation of the globe) are also typical_

The most life-threatening finding in Marian syndrome is aortic root dilation_ The diastolic murmur in this patient reflects aortic regurgitation. The syndrome requires close monitoring with echocardiography for the development of aneurysms and aortic arch dissection. Mitral valve prolapse is also common and manifests as a mid-systolic click and late systolic murmur. First-degree relatives should undergo genetic testing_

(Choice Al Homocystinuria is an autosomal recessive disorder that results from deficiency of cystathionine synthase, an enzyme involved in the metabolism of methionine. These patients share many features of Marian syndrome (eg, pectus deformity, tall stature, arachnodactyly}_ However, they usually have a fair complexion, thromboembolic events, and intellectual disability. The other main differentiating feature is lens dislocation in homocystinuria that is downward rather than upward.

se

Feedback End Block

Page 96: toddlers, infants, newborns

1 2

▪ 3

▪ 4-

▪ 5

▪ 6

▪ 7

• 8

• 9

• 10

• 11

• 12

• 13

• 14

15

• 16

• 17

• 18

• 19

• 20

• 21

• 22

• 23

• 24

• 25

• 25

• 27

• 28

• 29

• 30

• 31

Item: 15 of 31 V-Mark Previous Next

Lab Values. Notes Calculator.

4-..4-01 II 14-.4-..1.14.. .J11.4-.14-.1-L.11 I I IL.11 III 4--JLEALIL/I i..a II 14,ILAL.14-. 111.4141111.1, I i iy [......1..-.1[A.2LIL,ILy,

fingers (arachnodactyly ["thumb sign"]), pectus excavatum, and scoliosisikyphosis_ The face is long, the palate has a high arch, and the teeth are crowded. Lens dislocation (ectopia lentil), iridodonesis (a rapid contraction and dilation of the iris), and myopia (from elongation of the globe) are also typical_

The most life-threatening finding in Marfan syndrome is aortic root dilation_ The diastolic murmur in this patient reflects aortic regurgitation_ The syndrome requires close monitoring with echocardiography for the development of aneurysms and aortic arch dissection_ Mitral valve prolapse is also common and manifests as a mid-systolic click and late systolic murmur. First-degree relatives should undergo genetic testing_

(Choice A) Homocystinuria is an autosomal recessive disorder that results from deficiency of cystathionine synthase, an enzyme involved in the metabolism of methionine. These patients share many features of Marfan syndrome (eg, pectus deformity, tall stature, arachnodactyly}_ However, they usually have a fair complexion, thromboembolic events, and intellectual disability. The other main differentiating feature is lens dislocation in homocystinuria that is downward rather than upward.

(Choice B) Ehlers-Danlos syndrome is a collagen disorder characterized by scoliosis, joint laxity, and aortic dilation. Patients with this disorder do not have the disproportionately tall stature, lens dislocation, or pectus carinatum seen in Marfan syndrome.

(Choice ID) Congenital contractural arachnodactyly is an autosomal dominant condition resulting from mutations of the fibrillin-2 gene_ These patients have tall stature, arachnodactyly, and multiple contractures involving large joints. Ocular and cardiovascular symptoms are not present in congenital contractural arachnodactyly. In addition. patients with Marfan syndrome do not have joint contractures_

(Choice E) A tall and slender stature, but not connective tissue problems, is seen in Klinefelter syndrome_

Educational objective: Marfan syndrome is an autosomal dominant disorder that results from mutations of the fibrillin-1 gene. Affected patients have tall stature; long, thin extremities; arachnodactyly; joint hypermobility; upward lens dislocation; and aortic root dilation.

References:

1. Health supervision for children with Madan syndrome.

Copyright @ UWorld Last updated: [7/23/2014]

Feedback End Block

Page 97: toddlers, infants, newborns

.111 Mark Lab Values

Item: 15 of 31 EF. 11.

Notes Calculator Previous Next

Media Exhibit

Feedback End Block

V

V

Marfan syndrome

Thumb sign

▪ 2

▪ 3

▪ 4-

▪ 5

▪ 6

▪ 7

▪ 8

• 10

• 11

• 12

• 13

• 14

• 16

• 17

• -18

• 19

• 20

• 21

• 22

• 23

• 24

• 25

• 26

• 27

• 28

• 29

• 30

• 31

Page 98: toddlers, infants, newborns

1 .11M Mark Lab Values

Item: 15 of 31 Previous Next

rg.

Notes Calculator

V

Media Exhibit

Feedback End Block

V

Marfan syndrome

Arachnodactyly and loose joints allow the distal phalanx to protrude beyond the ulnar side of a clenched fist

<DUSK E LLC

▪ 2

▪ 3

▪ 4-

▪ 5

▪ 6

▪ 7

▪ 8

• 10

• 11

• 12

• 13

• 14

• 16

• 17

• -18

• 19

• 20

• 21

• 22

• 23

• 24

• 25

• 26

• 27

• 28

• 29

• 30

• 31

Page 99: toddlers, infants, newborns

Item: 15 of 31 .11M Mark Previous Next Lab Values Notes Calculator

Various aortic aneurysms I

Ascending aortic aneurysm

▪ 2

▪ 3

▪ 4-

▪ 5

▪ 6

▪ 7

▪ 8

▪ 9

• 10

• 1-1

• 12

• 13

• 14

• 16

• 17

• -18

• 19

• 20

• 21

• 22

• 23

• 24

• 25

• 26

• 27

• 28

• 29

• 30

• 31

Media Exhibit

a End Block

Feedback

V

Page 100: toddlers, infants, newborns

Previous Next Lab Values Notes Calculator

Item: 15 of 31 Mark

Various aortic aneurysms

OUSIALewofid..1.1.0

▪ 2

▪ 3

▪ 4-

▪ 5

▪ 6

▪ 7

▪ 8

• 10

• 1-1

• 12

• 13

• 14

• 16,

• 17

• -18

• 19

• 20

• 21

• 22

• 23

• 24

• 25

• 26

• 27

• 28

• 29

• 30

• 31

Media Exhibit

a End Block

Feedback

Page 101: toddlers, infants, newborns

1

A 6-year-old boy is brought to the physician by his mother with abdominal pain that began 4 days earlier_ The pain is diffuse and associated with nausea The boy has had no fever, vomiting, or changes in bowel habits_ He had a rash that was initially erythematous and macular but has now become confluent Physical examination shows a diffusely tender abdomen without rebound or guarding_ A nonblanching rash is noted on his lower extremities_ The patient's right knee is swollen and he complains of pain with passive range of motion. Laboratory results are as follows:

Complete blood count Hemoglobin 13.5 WI:IL Platelets 350,000/L Leukocytes 9,100/pL

Neutrophils 68% Eosinophils 1% Lymphocytes 25% Monocytes 6%

Serum chemistry Sodium 140 mEq!L Potassium 3_6 mEq/L Chloride 98 mEq!L Bicarbonate 23 mEq!L Blood urea nitrogen 18 mg/I:IL Creatinine 1.3 mg/dL Calcium E2mgldL Glucose 118 mg/dL

Urinalysis Specific gravity 1.022 Protein +1 Blood Moderate Glucose Negative Ketones Negative Leukocyte esterase Negative Nitrites Negative

Item: 16 of 31 F' Mark Previous Next

Lab Values. Notes Calculator.

se

2

▪ 3

▪ 4-

▪ 5

▪ 6

▪ 7

▪ 8 ▪ 9

• 10

• 11

• 12

• 13

• 14

• 15

I 16

• 17

• 18

• 19

• 20

• 21

• 22

• 23

• 24

• 25

• 26

• 27

• 28

• 29

• 30

• 31

Feedback End Block

Page 102: toddlers, infants, newborns

Item: 16 of 31

1 Mark Previous

Next

Lab Values. Notes

Calculator.

Which of the following glomerular abnormalities is most likely present in this patient?

A. Glomerular basement membrane thickening [5%]

B. Linear deposition of immunoglobulin G on the basement membrane [11°A]

C. Localized areas of mesangial sclerosis and collapse [3%]

D. Mesangial deposition of immunoglobulin A [72%]

E. Podocyte fusion [89.6]

Explanation:

Pathogenesis

Clinical

manifestations

Henoch-Schonlein purpura

• Immune-mediated leukocytoclastic vasculitis

• Associated with IgA deposition in affected organs

• Palpable purpura with normal platelet count &

coagulation studies

• Arthritis/arthralgia

• Abdominal pain

• Gastrointestinal bleeding

• Intussusception

• Renal disease similar to IgA nephropathy

• Scrotal pain & swelling

Laboratory

findings

• Normal platelet count & coagulation studies

• Normal to elevated creatinine

• Urinalysis: Hematuria, red cell casts & mild proteinuria

2

▪ 3

▪ 4-

▪ 5

▪ 6

▪ 7

▪ 8

▪ 9

• 10

• 11

• 12

• 13

• 14

• 15

I 16

• 17

• -18

• 19

• 20

• 21

• 22

• 23

• 24

• 25

• 26

• 27

• 28

• 29

• 30

• 31

Feedback End Block

Page 103: toddlers, infants, newborns

1

2

▪ 3

▪ 4-

5 ▪ 6

▪ 7

▪ 8

▪ 9

• 10

• -1 -1

• 12

• 13

• 14

• 15

I 16

• 17

• -18

• 19

• 20

• 21

• 22

• 23

• 24

• 25

• 26

• 27

• 28

• 29

• 30

• 31

Item: 16 of 31 MN/ark Previous

Next

Lab Values. Notes

Calculator.

Henoch-Schonlein purpura

Pathogenesis • Immune-mediated leukocytoclastic vasculitis

• Associated with IgA deposition in affected organs

Clinical

manifestations

- Palpable purpura with normal platelet count &

coagulation studies

• Arthritisiarthralgia

- Abdominal pain

' Gastrointestinal bleeding

• Intussusception

• Renal disease similar to IgA nephropathy

- Scrotal pain & swelling

Laboratory

findings

i Normal platelet count & coagulation studies

• Normal to elevated creatinine

• Urinalysis: Hematuria, red cell casts & mild proteinuria

Treatment

• Supportive management (hydration & pain control

with nonsteroidal anti-inflammatory drugs)

- Hospitalization indicated for severe abdominar pain,

renal insufficiency, inability to tolerate oral intake, or

altered mental status

- Systemic glucoccrticoids in patients with severe

abdominal pain unresponsive to nonsteroidal anti-

inflammatory drugs

USIVIEWAd, LI2C

Feedback End Block

Page 104: toddlers, infants, newborns

se

Item: 16 of 31 ,'Mark -<1

Previous Next

Lab Values. Notes Calculator.

et usrALEmad,E.Lc

This child's clinical presentation of abdominal pain, lower-extremity purpura, arthritis, and hematuria is most consistent with Henoch-SchOnlein purpura (HSP), an immunoglobulin A (IgA)-mediated vasculitis of the small vessels_ The rash typically consists of palpable purpura and is distributed symmetrically over the lower legs, buttocks, and arms. Arthralgiasiarthritis most commonly affect the knees and ankles. These symptoms are usually transient, and there is no permanent damage to the joints. Colicky abdominal pain (presumably due to local vasculitis) is present in the majority of patients.

Renal involvement occurs in approximately 2G%-50% of children and may occur as late as 4- weeks after onset of the illness. Most patients have relatively mild disease characterized by microscopic or macroscopic hematuria, red cell casts, and mild-to-moderate proteinuria (typically non-nephrotic range) with a normal or only slightly elevated serum creatinine_ However, more severe complications including nephrotic syndrome, hypertension, and acute renal failure, may occur. The diagnosis of HSP is made clinically in pediatric patients with a classic presentation_ However, in children with atypical presentations, a renal biopsy may be helpful to confirm the diagnosis and will demonstrate deposition of IgA in the mesangium.

(Choice A) Membranous nephropathy is characterized by thickening of the basement membrane that affects all glomeruli. Membranous nephropathy is far more common in adults than children.

(Choice B) Goodpasture syndrome, which is caused by antibodies directed against the basement membrane, typically presents with pulmonary hemorrhage and glomerulonephritis. It is seen in both adolescents and adults. Linear deposition of immunoglobulin G on the basement membrane is seen in Goodpasture syndrome.

(Choice C) Focal segmental glomerulosclerosis is the most common cause of nephrotic syndrome in adults in the United States. It is characterized by localized regions of mesangial sclerosis and basement membrane collapse. As suggested by the name, not all glomeruli are affected in this disease.

(Choice E) Minimal change disease is the most common form of idiopathic nephrotic syndrome in children and often presents with edema and hematuria. Electron microscopy findings include fusion or flattening of the podocytes ("foot processes").

Educational objective: Henoch-SchOnlein purpura is an immunoglobulin A (IgA)-mediated vasculitis of the small vessels that is most common in children. Classic manifestations include palpable purpura on the lower extremities, arthralgias. abdominal pain. and renal disease_ Immunofluorescence microscopy shows IgA deposition in the kidney.

2

▪ 3

▪ 4-

▪ 5

▪ 6

▪ 7

▪ 8 ▪ 9

• 10

• 11

• 12

• 13

• 14

• 15

I 16

• 17

• 18

• 19

• 20

• 21

• 22

• 23

• 24

• 25

• 26

• 27

• 28

• 29

• 30

• 31

Feedback End Block

Page 105: toddlers, infants, newborns

Item: 16 of 31 Mark Previous Next

Lab Values. Notes Calculator.

onset of the illness_ Most patients have relatively mild disease characterized by microscopic or macroscopic hematuria, red cell casts, and mild-to-moderate proteinuria (typically non-nephrotic range) with a normal or only slightly elevated serum creatinine. However, more severe complications including nephrotic syndrome, hypertension, and acute renal failure, may occur. The diagnosis of HP is made clinically in pediatric patients with a classic presentation_ However, in children with atypical presentations, a renal biopsy may be helpful to confirm the diagnosis and will demonstrate deposition of IgA in the mesangium_

(Choice A) Membranous nephropathy is characterized by thickening of the basement membrane that affects all glomeruli. Membranous nephropathy is far more common in adults than children.

(Choice B) Goodpasture syndrome, which is caused by antibodies directed against the basement membrane, typically presents with pulmonary hemorrhage and glomerulonephritis. It is seen in both adolescents and adults_ Linear deposition of immunoglobulin G on the basement membrane is seen in Goodpasture syndrome.

(Choice C) Focal segmental glomerulosclerosis is the most common cause of nephrotic syndrome in adults in the United States_ It is characterized by localized regions of mesangial sclerosis and basement membrane collapse. As suggested by the name, not all glomeruli are affected in this disease.

(Choice El Minimal change disease is the most common form of idiopathic nephrotic syndrome in children and often presents with edema and hematuria. Electron microscopy findings include fusion or flattening of the podocytes ("foot processes").

Educational objective: Henoch-SchOnlein purpura is an immunoglobulin A (IgA)-mediated vasculitis of the small vessels that is most common in children. Classic manifestations include palpable purpura on the lower extremities, arthralgias, abdominal pain, and renal disease_ Immunofluorescence microscopy shows IgA deposition in the kidney_

References:

1. Henoch Sch?nlein purpura in childhood: epidemiological and clinical analysis of 150 cases over a 5-year period and review of literature.

2. Clinical course of extrarenal symptoms in Henoch-Sch?nlein purpura: a 6-month prospective study.

Copyright © UWorld Last updated: [12/30/2014]

Feedback End Block

1 2

▪ 3

▪ 4-

▪ 5

▪ 6

▪ 7

▪ 8

• 9

• 10

• 11

• 12

• 13

• 14

• 15

I 16

• 17

• 18

• 19

• 20

• 21

• 22

• 23

• 24

• 25

• 26

• 27

• 28

• 29

• 30

• 31

Page 106: toddlers, infants, newborns

Item: 16 of 31 .111 II Mark Previous Next Lab Values Notes Calculator

Henoch-Schonlein purpura

2

▪ 3

▪ 4-

▪ 5

▪ 6

▪ 7

▪ 8

▪ 9

• 10

• -1-1

• 12

• 13

• 14

• 15

Media Exhibit

• 17

• -18

• 19

• 20

• 21

• 22

• 23

• 24

• 25

• 26

• 27

• 28

• 29

• 30

• 31

Page 107: toddlers, infants, newborns

1 2

▪ 3

▪ 4-

▪ 5

▪ 6

▪ 7

▪ 8 ▪ 9

• 10

• 11

• 12

• 13

• 14

• 15

• 15

a 17

• 18

• 19

• 20

• 21

• 22

• 23

• 24

• 25

• 25

• 27

• 28

• 29

• 30

• 31

se

A 2 and a half-year-old child is brought to the office for the evaluation of easy bruising, nosebleeds, and decreased activity over the past week. He had an upper respiratory infection that was treated with an antibiotic 2 weeks ago. On examination, he is well-developed, seems well-nourished, anicteric, and pale. Pertinent findings include some small palpable posterior cervical lymph nodes, sinus tachycardia, a grade INI systolic ejection murmur, ecchymoses on his left shoulder and both lower extremities, and petechiae over his extremities and groin. There is no hepatosplenomegaly_ The laboratory findings are as follows:

Hemoglobin 7_9 g/dL Hematocrit 24% Platelet count 12,000/mm3 WEC 3,000/mm3 Reticulocyte count 15%

A bone marrow biopsy reveal a markedly hypocellular marrow with decreased megakaryocytes and precursors of the erythroid and myeloid cell lines. What is the most likely diagnosis?

A. Acquired aplastic anemia [71%]

B. Fanconi's anemia [12%]

C. Diamond-Black-fan anemia [€%]

Transient erythroblastopenia [4%]

E Acute myeloid leukemia [7*]

Explanation:

Acquired aplastic anemia results from an injury to the bone marrow by radiation, drugs (chemotherapy or antibiotics such as chloramphenicol), insecticides, toxins (benzene, carbon tetrachloride), or infections. Signs and symptoms include pallor, fatigue, weakness, loss of appetite, easy bruising, petechiae, mucosal hemorrhage, and fever_ Laboratory evaluation demonstrates a normocytic or macrocytic anemia, leukopenia, reticulocytopenia, and thrombocytopenia_ A bone marrow biopsy is essential to make the diagnosis; it typically shows profound hypocellularity with a decrease in all cell lines and fatty infiltration of the marrow.

(Choice B) Classically, patients with Fanconi's anemia have pancytopenia and characteristic congenital anomalies_ such as hvoeroiomentation on the trunk_ neck and intertrioinous areas and/or cafe-au-fait soots.

Item: 17 of 31 .11M Mark Previous Next

Lab Values. Notes Calculator.

Feedback End Block

Page 108: toddlers, infants, newborns

2

▪ 3

▪ 4-

▪ 5

▪ 6

▪ 7

▪ 8

▪ 9

• 10

• 11

• 12

• 13

• 14

• 15

• 15

a 17

• 18

• 19

• 20

• 21

• 22

• 23

• 24

• 25

• 25

• 27

• 28

• 29

• 30

• 31

1

Acquired aplastic anemia results from an injury to the bone marrow by radiation, drugs (chemotherapy or antibiotics such as chloramphenicol), insecticides, toxins (benzene, carbon tetrachloride), or infections. Signs and symptoms include pallor, fatigue, weakness, loss of appetite, easy bruising, petechiae, mucosal hemorrhage, and fever. Laboratory evaluation demonstrates a normocytic or macrocytic anemia, leukopenia, reticulocytopenia, and thrombocytopenia_ A bone marrow biopsy is essential to make the diagnosis; it typically shows profound hypocellularity with a decrease in all cell lines and fatty infiltration of the marrow.

(Choice B) Classically, patients with Fanconi's anemia have pancytopenia and characteristic congenital anomalies, such as hyperpigmentation on the trunk, neck and intertriginous areas and/or cafe-au-fait spots, short stature, upper limb abnormalities, hypogonadism, skeletal anomalies, eye or eyelid changes, and renal malformations. Blood counts start to decrease between 4 and 12 years of age, and the initial manifestation is usually thrombocytopenia, followed by neutropenia, then anemia.

(Choice C) Diamond-Black-fan anemia (IDEA), or congenital pure red cell aplasia, presents in the first 3 months of life with pallor and poor feeding. CBC reveals a normocytic or macrocytic anemia with reticulocytopenia. WBC and platelet counts are normal.

(Choice ID) Transient erythroblastopenia of childhood (TEC) is an acquired red cell aplasia which occurs in healthy children between 6 months and 5 years old. There is a gradual onset of symptoms such as pallor and decreased activity. The physical examination is unremarkable except for pallor and tachycardia The typical laboratory findings are normocytic normochromic anemia, with hemoglobin levels ranging from 3 to 8 g/dL, and an extremely low reticulocyte count

(Choice E) Bone marrow infiltration due to leukemia results in pancytopenia by crowding out the normal bone marrow elements. Patients present with lethargy (from anemia), bruising and bleeding (from thrombocytopenia), and unexplained fever_ They may also complain of bone pain or present with a limp_ CBC shows pancytopenia or anemia, thrombocytopenia and leukocytosis. Acute myeloid leukemia is more common in adults_ In this case, the classic presentation after an upper respiratory infection, as well as the absence of other features of leukemia (normal bone marrow), makes acquired aplastic anemia the best answer.

Educational Objective: Aplastic anemia should be suspected in any patient with pancytopenia following drug intake, exposure to toxins or viral infections.

Copyright © LIWorld

Last updated: [1G/23/2014]

Item: 17 of 31 V-Mark Previous Next

Lab Values. Notes Calculator.

1

Feedback End Block

Page 109: toddlers, infants, newborns

se

An 11-year-old boy is brought to the office by his mother because "he is sick.' He has had headaches for the past several weeks, and has vomited 4 times in the past 5 days. He drinks large amounts of water and goes to the bathroom all the time. He is no longer interested in playing football and going out with his friends. His temperature is 3L0° C (98.6° F), blood pressure is 118/78 mm Hg, pulse rate is 84/min, and respirations are 16/min. On examination, there is loss of peripheral visual fields. His laboratory findings are as follows:

WE C 7,800/mm3 Hemoglobin 12.6 g/dL Hematocrit 35% Platelets 199,000/mm3 Sodium 145 mEq/L Potassium 3.6 mEq/L Bicarbonate 24 mEq/L Blood urea nitrogen 18 mg/dL Serum creatinine 1.0 mg/dL Blood glucose 88 mg/dL

X-rays of the head reveal a calcified lesion above the sella. What is the most likely diagnosis?

A. Pituitary adenoma [20%]

B. Meningioma [1%]

C. Empty sella syndrome [4%]

• ID. Craniopharyngioma [73%]

E Ependymoma [1%]

Explanation:

A young boy with symptoms of increased intracranial pressure (e_g_, headaches, vomiting), bitemporal hemianopsia, and a calcified lesion above the sella has a craniopharyngioma until proven otherwise. Craniopharyngiomas are derived from epithelial remnants of Rathke's pouch. It is not a true pituitary tumor, and is characterized by nests of squamous cells in a loose stroma, resembling the appearance of embryonic tooth bud enamel. Its location is suprasellar and inferior to the optic chiasm, which is why it can cause hi-temporal hemianooia by pressina the optic chiasm). Associated endocrine symptoms such as diabetes

1 2

▪ 3

▪ 4-

▪ 5

▪ 6

▪ 7

▪ 8

▪ 9

• 10

• 11

• 12

• 13

• 14

• 15

• 16

• 17

I -18

• 19

• 20

• 21

• 22

• 23

• 24

• 25

• 26

• 27

• 28

• 29

• 30

• 31

Item: 18 of 31 F' Mark Previous Next

Lab Values. Notes Calculator.

Feedback End Block

Page 110: toddlers, infants, newborns

Explanation:

A young boy with symptoms of increased intracranial pressure (e_g_, headaches, vomiting), bitemporal hemianopsia, and a calcified lesion above the sella has a craniopharyngioma until proven otherwise. Craniopharyngiomas are derived from epithelial remnants of Rathke's pouch. It is not a true pituitary tumor, and is characterized by nests of squamous cells in a loose stroma, resembling the appearance of embryonic tooth bud enamel. Its location is suprasellar and inferior to the optic chiasm, which is why it can cause bi-temporal hemianopia (by pressing the optic chiasm). Associated endocrine symptoms such as diabetes insipidus (as in this patient), and growth failure associated with either hypothyroidism or growth hormone deficiency may occur. The diagnosis is usually made by computed tomographic (CT) or magnetic resonance (MR) imaging. Presence of a cystic calcified parasellar lesion on MRI is almost diagnostic of craniopharyngioma_ Surgical removal is the treatment of choice.

(Choice A) Patients with pituitary adenoma may present with similar symptoms; however, the condition is more frequent in women. Furthermore, prolactinoma is an important part of the syndrome, and calcification of the gland does not occur.

(Choice B) Meningiomas are non-invasive tumors derived from arachnoid cap cells. and consist of concentric whorls and calcified psammoma bodies. These may indent the brain and cause hvperostosis. These usually occur on the convex surfaces of the brain.

(Choice C) In empty sella syndrome, the sella turcica is often enlarged and contains no discernible pituitary gland_ This syndrome may be primarily due to an incompetent cellar diaphragm with compression of the pituitary gland by the herniating arachnoid, or secondary to surgery or radiotherapy.

(Choice E) Ependymomas are gliomas and are benign. These do not cause bi-temporal hemianopia.

Educational Objective: A young boy with symptoms of increased intracranial pressure (e_g_, headaches, vomiting), bitemporal hemianopsia, and a calcified lesion above the sella has a craniopharyngioma until proven otherwise. Presence of a cystic calcified parasellar lesion on MRI is almost diagnostic of craniopharyngioma.

Copyright © UWorld Last updated: [8/22/2014]

1 2

▪ 3

▪ 4-

▪ 5

▪ 6

▪ 7

▪ 8

▪ 9

• 10

• 11

• 12

• 13

• 14

• 15

• 15

• 17

I -18

• 19

• 20

• 21

• 22

• 23

• 24

• 25

• 25

• 27

• 28

• 29

• 30

• 31

Item: 18 of 31 Mark Previous

Next

Lab Values. Notes

Calculator.

L). I dI1114,11 Id! yI iyiui I Id Li an

E Ependymoma [1'36]

Feedback End Block

Emaad
Highlight
Emaad
Highlight
Emaad
Highlight
Emaad
Highlight
Emaad
Highlight
Emaad
Highlight
Emaad
Highlight
Page 111: toddlers, infants, newborns

1 2

▪ 3

▪ 4-

▪ 5

▪ 6

▪ 7

▪ 8 ▪ 9

• 10

• 11

• 12

• 13

• 14

• 15

• 15

• 17

• 18

19

• 20

• 21

• 22

• 23

• 24

• 25

• 25

• 27

• 28

• 29

• 30

• 31

se

The parents of a 5-year-old girl bring her to clinic for evaluation. They are frightened because their daughter has started to talk to herself, especially when she is alone in her room. When the parents ask who she is talking to, she says, "Nobody!" and refuses to discuss the matter. She becomes angry if they persist. Recently, after spilling milk during breakfast, the girl denied having done it, insisting that "Mindy" was responsible_ The parents state they have no idea who Mindy is. The girl had some trouble adjusting to kindergarten at the start of the school year, but is now doing well_ She is an only child, has no medical problems, and has reached all of her milestones on time. What should the parents be advised?

▪ A. 'Your daughter's behavior is normal for her age, and there is nothing to worry about" [91 °,70]

B. 'Your daughter should start taking risperidone to decrease the likelihood of her symptoms worsening." [1%]

C. 'Your daughter may be experiencing mistreatment and abuse in her kindergarten class." [296]

ID. "Your daughter should begin weekly counseling sessions with the school psychologist" [4%]

E "Your daughter has oppositional defiant disorder and needs more limit setting before it worsens.' [1%]

Explanation:

This 5-year-old girl has an imaginary friend, which is quite common for her age. Parents often become concerned and puzzled when their children acquire imaginary friends, but this phenomenon is generally considered an indication that the child has found creative ways to deal with being alone. Children between the ages of two and six years are most likely to develop imaginary friends, typically in response to times of change or stress_ Most children abandon this behavior within the first few years of elementary school_

(Choices B, C, and 13) Imaginary friends are considered a completely normal phenomenon in children. They are not indicative of psychosis, abuse, or the need for counseling.

(Choice E) Although children with oppositional defiant disorder may blame others for their mistakes or misdeeds, they also tend to have a pattern of angry, argumentative, and negative behavior towards their parents and teachers.

Educational objective: Imaninary fripnri arp a nnrml rnmnnnpnt of riplipInnmpnt fnr rrinv vni inn rhilrirpn anri Arp i ici isltv ni itnrniArn

Item: 19 of 31

V-• Mark Previous

Next

Lab Values. Notes

Calculator.

Feedback End Block

Page 112: toddlers, infants, newborns

1 2

▪ 3

▪ 4-

▪ 5

▪ 6

▪ 7

▪ 8 ▪ 9

• 10

• 11

• 12

• 13

• 14

• 15

• 15

• 17

• 18

19

• 20

• 21

• 22

• 23

• 24

• 25

• 25

• 27

• 28

• 29

• 30

• 31

A I IC13 3LCII LCU LU LOIN LU I ICI 3CII, C3pcuic1iiy vvi ici I I lc 13 CHU! IC II 1 I ICI I UUI WWI ICI I LI IC IJCII CI IL3 Cl3rx WI IU 3I 1C IJ

talking to, she says, "Nobody!" and refuses to discuss the matter. She becomes angry if they persist. Recently, after spilling milk during breakfast, the girl denied having done it, insisting that "Mindy" was responsible_ The parents state they have no idea who Mindy is. The girl had some trouble adjusting to kindergarten at the start of the school year, but is now doing well_ She is an only child, has no medical problems, and has reached all of her milestones on time. What should the parents be advised?

▪ A. 'Your daughter's behavior is normal for her age, and there is nothing to worry about" [91%]

B. 'Your daughter should start taking risperidone to decrease the likelihood of her symptoms worsening." [1%]

C. 'Your daughter may be experiencing mistreatment and abuse in her kindergarten class." [2%]

ID. 'Your daughter should begin weekly counseling sessions with the school psychologist" [4*]

E "Your daughter has oppositional defiant disorder and needs more limit setting before it worsens.' [1%]

Explanation:

This 5-year-old girl has an imaginary friend, which is quite common for her age. Parents often become concerned and puzzled when their children acquire imaginary friends, but this phenomenon is generally considered an indication that the child has found creative ways to deal with being alone. Children between the ages of two and six years are most likely to develop imaginary friends, typically in response to times of change or stress. Most children abandon this behavior within the first few years of elementary school.

(Choices B, C, and D) Imaginary friends are considered a completely normal phenomenon in children. They are not indicative of psychosis, abuse, or the need for counseling.

(Choice E) Although children with oppositional defiant disorder may blame others for their mistakes or misdeeds. they also tend to have a pattern of angry, argumentative, and negative behavior towards their parents and teachers.

Educational objective: Imaginary friends are a normal component of development for many young children. and are usually outgrown by the early elementary school years_

Item: 19 of 31

V- Mark Previous

Next

Lab Values. Notes

Calculator.

Copyright © UWorld Last updated: [1/1G/2015]

Feedback End Block

Page 113: toddlers, infants, newborns

1 2

▪ 3

▪ 4-

5 ▪ 6

▪ 7

▪ 8 ▪ 9

• 10

• 11

• 12

• 13

• 14

• 15

• 15

• 17

• 18

• 19

• 20

• 21

• 22

• 23

• 24

• 25

• 25

• 27

• 28

• 29

• 30

• 31

se

A 15-year-old boy is brought to the emergency department due to the sudden onset of difficulty breathing for the past 45 minutes. He also complains of nausea, colicky abdominal pain, and a swollen face. The patient has suffered from bronchitis for the past 4 days, and his condition had been improving. His mother says that he had a similar episode when he had a tooth extracted 2 years ago. On examination, there is an edematous swelling of his face including the lips, hands, arms, legs, and genitals without a rash. Temperature is 36.8 C (98.4 F), blood pressure is 120/80 mm Hg, pulse is 82/min, and respirations are 18/min. Which of the following best explains the pathological process of this patient's condition?

A. Antibody-mediated hypersensitivity [15%]

E Cl inhibitor deficiency [62%]

C. Cell-mediated hypersensitivity [9%]

D. Depressed Cl q [3%]

E. Immune complex-mediated hypersensitivity [12%]

Explanation:

The patient described is suffering from hereditary angioedema. which is characterized by the following:

1. Rapid onset of:

• Noninflammatory edema of the face, limbs, and genitalia • Laryngeal edema — can be life-threatening • Edema of the intestines resulting in colicky abdominal pain

2. No evidence of urticaria

Angioedema can be hereditary or acquired. The pathology in both forms can involve Cl inhibitor deficiency, dysfunction, or destruction. A defect or deficiency of Cl inhibitor leads to elevated levels of the edema-producing factors C2b and bradykinin. The most common cause of acquired isolated angioedema is due to angiotensin-converting-enzyme inhibitor use, which results in elevated levels of bradykinin.

Hereditary angioedema typically presents in late childhood_ Episodes usually follow an infection, dental procedure, or trauma Cl q levels are normal in hereditary angioedema and depressed in acquired forms,

Item: 20 of 31 V-Mark Previous Next

Lab Values. Notes Calculator.

Feedback End Block

Page 114: toddlers, infants, newborns

1 2

▪ 3

▪ 4-

5 ▪ 6

▪ 7

▪ 8 ▪ 9

• 10

• 11

• 12

• 13

• 14

• 15

• 15

• 17

• 18

• 19

• 20

• 21

• 22

• 23

• 24

• 25

• 25

• 27

• 28

• 29

• 30

• 31

Item: 20 of 31 V-Mark Previous Next

Lab Values. Notes Calculator.

dysfunction, or destruction. A defect or deficiency of Cl inhibitor leads to elevated levels of the edema-producing factors C2b and bradykinin. The most common cause of acquired isolated angioedema is due to angiotensin-converting-enzyme inhibitor use, which results in elevated levels of bradykinin.

Hereditary angioedema typically presents in late childhood_ Episodes usually follow an infection, dental procedure, or trauma Cl q levels are normal in hereditary angioedema and depressed in acquired forms, which usually present much later in life. C4 levels are depressed in all forms of angioedema.

(Choice A) Antibody-mediated (type II) hypersensitivity can be seen in immune hemolytic anemia and Rh hemolytic disease of the newborn. It involves specific reactions of IgG or IV antibodies with cell-bound antigens leading to complement activation and cell destruction. Type I reactions (immediate hypersensitivity) also involve the antibody IgE but result in mast cell and basophil degranulation with associated urticaria and pruritus; this patient has neither.

(Choice C) The most common examples of cell-mediated (type IV) hypersensitivity are the tuberculin skin test and allergic contact dermatitis. In allergic contact dermatitis, the allergen causes dermal inflammation following direct contact with the skin after a latent period of 1-2 days_

(Choice ID) Low Cl q levels are associated with acquired angioedema, which usually presents in patients age 313.

(Choice E) Immune complex-mediated (type III) hypersensitivity can be seen in serum sickness. Antibodies form complexes with allergens that are present in tissues, leading to activation of the complement cascade wherever the immune complexes deposit

Educational objective: Hereditary angioedema is characterized by rapid-onset edema of the face, acral extremities, genitals. trachea. and abdominal organs without urticaria. It is due to a deficiency or dysfunction in Cl inhibitor, which results in elevated levels of the edema-producing factors C2b and bradykinin.

References:

1. Hereditary angia-oedema.

2. Recognizing and managing hereditary angicedema.

Copyright @ UWorld

Last updated: [9/15/2014]

Feedback End Block

Emaad
Highlight
Page 115: toddlers, infants, newborns

1 2

▪ 3

▪ 4-

5 ▪ 6

▪ 7

▪ 8 ▪ 9

• 10

• 11

• 12

• 13

• 14

• 15

• 15

• 17

• -18

• 19

• 20

• 21

• 22

• 23

• 24

• 25

• 25

• 27

• 28

• 29

• 30

• 31

A 3-year-old boy from a refugee camp is brought to the physician for evaluation of a rash_ He recently came to the United States and his medical history is unknown. The rash has been present for a few weeks and seems to be spreading. He is hungry "all the time" and his family had limited access to food in their home country. Review of systems is negative for nausea, vomiting, diarrhea, headaches, numbness, or tingling. The boy takes no medications and has no known allergies_ His weight and length are <5th percentile for age and sex. Examination shows a malnourished boy with minimal subcutaneous fat. Scaling and fissures are present at the mouth corners and his lips are cracked and inflamed. The patient's tongue and oropharyngeal mucous membranes are swollen and hyperemic. There are erythematous scaly patches on his eyebrows, cheeks, and nose_ The rash is also present on the scrotal skin and extends to the medial aspect of both thighs. The skin and conjunctivae are pale and his fingers and toenails are brittle_ Laboratory results are as follows:

Complete blood count Hemoglobin 9 g/dL Mean corpuscular volume 82 fL Platelets 1 80,000/pL Leukocytes 7,500/pL

Serum chemistry Sodium 136 mEq!L Potassium 3.8 mEq!L Chloride 1DD mEcilL Bicarbonate 26 mEq!L Blood urea nitrogen 9 mg/dL Creatinine 0.2mgldL Glucose 73 mg/I:IL

Which of the following is the most likely cause of this patient's condition?

A. Vitamin A toxicity [3%]

B. Vitamin B, (thiamine) deficiency [4%]

• C. Vitamin B2 (riboflavin) deficiency [441C]

ID. Vitamin 13, (niacin) deficiency [27%]

Item: 21 of 31 V-• Mark Previous Next

Lab Values. Notes Calculator.

se

Feedback End Block

Page 116: toddlers, infants, newborns

2 ▪ 3 ▪ 4-

▪ 5

▪ 6 ▪ 7 ▪ 8

• 10 • 11 • 12 • 13 • 14 • 15 • 16 • 17 • 18 • 19 • 20 • 21 • 22 • 23 • 24 • 25 • 26 • 27 • 28 • 29 • 30 • 31

I Water-soluble vitamins

. , ,

Vitamin Source Deficien cy

B1 (thiamine) Whole grains, meat, fortified cereal, nuts, legumes

• Beriberi (peripheral neuropathy, heart failure)

• Wernicke-Korsakoff syndrome

171 (riboflavin) Dairy, eggs, meat, green vegetables

• Angular cheilosis, stomatitis, glossitis

• Normocytic anemia • Seborrheic dermatitis

B (niacin 3 ) Meat, whole grains, legumes

• Pellagra (dermatitis, diarrhea, delusions/dementia, glossitis)

. ,

B€ (pyridoxine) Meat, whole grains: legumes, nuts

• Cheilosis, stomatitis, glossitis, • irritability, confusion,

depression

B9 (folate, folic acid) Green leafy vegetables, fruit, meat, fortified cereal/grains

• IVIegaroblastic anemia • Neural tube defects (fetus)

B12 (cobalamin) Meat, dairy • Megaloblastic anemia • Neurologic deficits (confusion,

paresthesias, ataxia)

I .

Item: 21 of 31 Mark Previous

Next

Lab Values, Notes

Calculator.

Feedback a

End Block

Emaad
Sticky Note
try again aliyah
Page 117: toddlers, infants, newborns

11P,. Lab Values, Notes Calculator.

: 21 of 31 1- V'Mark Previous Next .

B12 (cobalamin) Meat, dairy

.

• Wlegaloblastic anemia

• Neurologic deficits (confusionr paresthesias, ataxia)

,

C (ascorbic acid) Citrus fruits, strawberries, tomatoes,

s, potatoe broccoli

• Scurvy {pun tate hemorrhage,

gingivitis, corkscrew hair)

S LEW° r1d„ LLC

Vitamin 62 (riboflavin) is a precursor to important coenzymes that participate in oxidation-reduction reactions

for energy production. Riboflavin is present in meat, eggs, yeast, dairy products, green vegetables, and enriched foods. Deficiency of this vitamin is common in underdeveloped countries with severe food shortages. This is unusual in industrialized nations but can be seen in patients with anorexia nervosa, low-dairy diet (eg, lactose intolerance), and malabsorptive syndromes (eg, celiac sprue).

Riboflavin deficiency is characterized by angular cheilitis (fissures at corners of lips), glossitis (hyperemic tongue), stomatitis (hyperemic/edematous oropharvngeal mucous membranes, sore throat), normocytic-normochromic anemia, and seborrheic dermatitis. Riboflavin has no known toxic effects as excess amounts of this water-soluble vitamin are excreted in urine.

(Choice A) Hypervitaminosis A can cause neuropsychiatric symptoms and cerebral edema. However, toxicity of any vitamin is unlikely in this severely malnourished child.

(Choice B) Thiamine deficiency can cause dry beriberi (peripheral neuropathy). wet beriberi (dilated cardiomyopathy), and Wernicke-Korsakoff syndrome.

(Choice 0) Niacin deficiency causes pellagra, which is characterized by a symmetric reddish rash on exposed skin areas, glossitis, diarrhea/vomiting, and a variety of neurologic disturbances (insomnia, anxiety, disorientation, delusions, dementia, encephalopathy). This diagnosis is less likely as niacin deficiency does not cause anemia_ In addition, this patient has no neurologic or gastrointestinal symptoms.

(Choice E) Although toxicity from water-soluble vitamins is uncommon, excess pyridoxine can cause peripheral neuropathy_

se

1 2

▪ 3 ▪ 4-

▪ 5 ▪ S ▪ 7 ▪ 8 ▪ 9

• 10 • 11 • 12 • 13 • 14 • 15 • 15 • 17 • -18 • 19 • 20 • 21 • 22 • 23 • 24

• 25 • 25 • 27 • 28 • 29 • 30 • 31

Feedback End Block

Page 118: toddlers, infants, newborns

1 2

▪ 3

▪ 4-

5 ▪ 6

▪ 7

▪ 8 ▪ 9

• 10

• 11

• 12

• 13

• 14

• 15

• 15

• 17

• -18

• 19

• 20

• 21

• 22

• 23

• 24

• 25

• 25

• 27

• 28

• 29

• 30

• 31

Item: 21 of 31

1 lark Previous

Next

Lab Values. Notes

Calculator.

excess amounts of this water-soluble vitamin are excreted in urine.

(Choice A) Hypervitaminosis A can cause neuropsychiatric symptoms and cerebral edema. However, toxicity of any vitamin is unlikely in this severely malnourished child.

(Choice B) Thiamine deficiency can cause dry beriberi (peripheral neuropathy), wet beriberi (dilated cardiomyopathy), and Wernicke-Korsakoff syndrome_

(Choice 0) Niacin deficiency causes pellagra, which is characterized by a symmetric reddish rash on exposed skin areas, glossitis, diarrhea/vomiting, and a variety of neurologic disturbances (insomnia, anxiety, disorientation, delusions, dementia, encephalopathy). This diagnosis is less likely as niacin deficiency does not cause anemia_ In addition, this patient has no neurologic or gastrointestinal symptoms.

(Choice E) Although toxicity from water-soluble vitamins is uncommon. excess pyridoxine can cause peripheral neuropathy_

(Choice F) Vitamin C deficiency (scurvy) impairs collagen synthesis. Mucocutaneous symptoms include ecchymoses, petechiae, bleeding gums, hyperkeratosis, and coiled hair_

(Choice G) Vitamin K deficiency manifests as impaired coagulation (eg, easy bruisability. mucosal bleeding, melena. hematuria). It is generally rare, but neonates and patients on long-term antibiotics are at risk due to lack of bowel flora.

Educational objective: Vitamin B2 (riboflavin) is a water-soluble vitamin that is present in meat, eggs, yeast, dairy products, green

vegetables, and enriched foods. Riboflavin deficiency should be suspected in a malnourished patient with angular cheilitis, stomatitis, glossitis, normocytic-normochromic anemia, and seborrheic dermatitis.

References:

1. Suspected outbreak of riboflavin deficiency among populations reliant on food assistance: a case study of drought-stricken Karamoja, Uganda, 2009-2010.

2. Chapter 30: historical aspects of the major neurological vitamin deficiency disorders: the water-soluble B vitamins.

Copyright © UWorld Last updated: [9/11/2014]

Feedback End Block

Page 119: toddlers, infants, newborns

Item: 21 of 31 .111 Mark -•:::1

Previous Next Lab Values Notes

ligrno Lox.

Calculator

Media Exhibit

Scrvy

Coiled hairs

Perifollicular I hemorrhages

Feedback a

End Block

• 1

▪ 2

▪ 3

▪ 4-

▪ 5

▪ 6

▪ 7

▪ 8

• 10

• 11

• 12

• 13

• 14

• 15

• 16

• 17

• 18

- 19

- 20

22

23

24

25

26

27

28

29

30

31

it

Page 120: toddlers, infants, newborns

1 2

▪ 3

▪ 4-

5 ▪ 6

▪ 7

▪ 8 ▪ 9

• 10

• 11

• 12

• 13

• 14

• 15

• 15

• 17

• 18

• 19

• 20

• 21

• 22

▪ 23

• 24

▪ 25

• 25

• 27

• 28

• 29

• 30

• 31

se

A 12-year-old African American male with known sickle cell disease presents with a 2 hour history of right-sided arm weakness and slurred speech. He has been hospitalized before for pain crises and pneumonia He takes hydroryurea, oxycodone as needed, and folic acid. His temperature is 98.0°F (36.6°C), blood pressure is 153/83 mmHg and heart rate is 11 /min with regular rhythm_ an physical examination, he has right arm weakness and mild dysarthria. His laboratory values are the following:

Hemoglobin 8.2 mg/dL WBC count 14,000/me Platelet count 210,000/mm Creatinine mg/dL

CT of the head shows no evidence of intracranial bleeding_ Which of the following is the best initial management for this patient?

A. Beta-blockers and aspirin [7%]

13. Exchange transfusion [43%]

C. Fibrinolytic therapy [36%]

ID. Heparin and warfarin [6%]

E Plasmapheresis [7%]

Explanation:

This patient's slurred speech may represent a Broca's aphasia In combination with his right-sided arm weakness, these findings are suggestive of a focal abnormality in the left frontal lobe, most likely an acute stroke given his history of sickle cell disease. There are a variety of factors which can lead to stroke in a young sickle cell patient, most prominently sludging and occlusion of the cerebral arterial vasculature by the malformed red blood cells. While the head CT is reportedly negative, this should not dissuade one from considering stroke as it can be difficult to detect changes of stroke in the first few hours on a CT_

In the acute setting, one of the primary treatments for stroke in a sickle cell patient is exchange transfusion. While this may not reverse changes from the initial vascular event, it helps to decrease the percentage of sickle cells in the bloodstream and makes an additional stroke less likely. Continuing the hydroxyurea should also help to decrease the percentage of sickle cells by increasing the proportion of Hqb

Item: 22 of 31 V-Mark Previous Next

Lab Values. Notes Calculator.

Feedback End Block

Page 121: toddlers, infants, newborns

1 2

▪ 3

▪ 4-

5 ▪ 6

▪ 7

▪ 8 ▪ 9

• 10

• 11

• 12

• 13

• 14

• 15

• 15

• 17

• 18

• 19

• 20

• 21

• 22

▪ 23

• 24

▪ 25

• 25

• 27

• 28

• 29

• 30

• 31

Item: 22 of 31

V-Mark Previous

Next

Lab Values. Notes

Calculator.

axpianation;

This patient's slurred speech may represent a Broca's aphasia_ In combination with his right-sided arm weakness, these findings are suggestive of a focal abnormality in the left frontal lobe, most likely an acute stroke given his history of sickle cell disease_ There are a variety of factors which can lead to stroke in a young sickle cell patient, most prominently sludging and occlusion of the cerebral arterial vasculature by the malformed red blood cells. While the head CT is reportedly negative, this should not dissuade one from considering stroke as it can be difficult to detect changes of stroke in the first few hours on a CT_

In the acute setting, one of the primary treatments for stroke in a sickle cell patient is exchange transfusion. While this may not reverse changes from the initial vascular event, it helps to decrease the percentage of sickle cells in the bloodstream and makes an additional stroke less likely. Continuing the hydroxyurea should also help to decrease the percentage of sickle cells by increasing the proportion of Hgb F.

(Choice A) It is unlikely that this patient has true underlying vascular disease given his young age_ Treatment should be directed towards the sickle cell disease itself

(Choice C) Fibrinolytic therapy is unlikely to be effective since this patient's symptoms are likely secondary to sludging of sickle cells as opposed to a true thrombus.

(Choice 13) Heparin and warfarin will likely not be effective as this patient probably does not have a true thrombus.

(Choice E) Plasmapharesis is not a common treatment used in sickle cell disease.

Educational objective: Stroke is a common complication of sickle cell disease secondary to sludging and occlusion in the cerebral vasculature_ Exchange transfusion is the recommended treatment acutely since it helps to decrease the percentage of sickle cells and prevent a second infarct from occurring.

References:

1. Exchange blood transfusion compared with simple transfusion for first overt stroke is associated with a lower risk of subsequent stroke: a retrospective cohort study of 137 children with sickle cell anemia.

Copyright © UWorld

Last updated: [8/15/2014]

Feedback End Block

Page 122: toddlers, infants, newborns

1 2

▪ 3

▪ 4-

5 ▪ 6

▪ 7

▪ 8 ▪ 9

• 10

• 11

• 12

• 13

• 14

• 15

• 15

• 17

• 18

• 19

• 20

• 21

• 22

23

• 24

• 25

• 25

• 27

• 28

• 29

• 30

• 31

se

A 5-month-old girl is brought to the physician for a routine visit The patient has been evaluated several times for poor weight gain. The infant takes .6 ounces of regular formula every 4 hours. Increasing the caloric density of her formula has not improved her growth. The patient has no abdominal pain, nausea, diarrhea, or vomiting. There is a family history of nephrolithiasis_ She was born full term without complications. The patient's birth weight was 3f kg (8 Ib, 50th percentile). Weight is <5th percentile; length and head circumference have been tracking along the 25th percentile. The infant appears thin, but the remainder of the physical examination is unremarkable. Newborn screening results were normal. Laboratory results are as follows:

Serum chemistry Sodium 140 mEq/L Potassium 3 mEq!L Chloride 121 mEq!L Blood urea nitrogen 10 mg/dL Creatinine 0.5 mg/I:IL Calcium 9 mg/dL Glucose 98 mg/dL

Arterial blood gases pH 721 PaCO2. 31 mm Hg Bicarbonate 14 mEq!L

Urinalysis pH 7.9 Potassium Normal Sodium Normal

Which of the following is the most likely cause of this patient's failure to thrive?

A. Cystic fibrosis [11%]

B. Gastroesophageal reflux [1'36]

C. Insufficient caloric intake [4'36]

n I artir aririn¢ia f1f?°,1

Item: 23 of 31 F' Mark Previous Next

Lab Values. Notes Calculator.

Feedback End Block

Page 123: toddlers, infants, newborns

2

▪ 3

▪ 4-

5 ▪ 6

▪ 7

▪ 8

• 10

• 11

• 12

• 13

• 14

• 15

• 16

• 17

• 18

• 19

• 20

• 21

• 22

23.

• 24

• 25

• 26

• 27

• 28

• 29

• 30

• 31

Item: 23 of 31 V-• Mark -4Z1 Previous Next

a VI .... ... Lab Values Notes Calculator

WWI III-I I UI LI le IUIIUY III IJ LI le I I IUJL IIISCI L dlJ7C UI LI IIJ pauei IL J IdIILII C LU LI II IVe

A. Cystic fibrosis [11%]

B. Gastroesophageal reflux [1%]

C. Insufficient caloric intake [4%]

ID. Lactic acidosis [10%]

E Renal tubular acidosis [13%]

Explanation:

Renal tubular acidosis

Type ,

1 (Distal) 2 (Proximal) 4

Primary defect

Poor hydrogen ion secretion into urine

Poor bicarbonate resorption

Aldosterone resistance

Urine pH X5.5 <5_5 <5.5

. Serum

potassium Low-normal Low-normal

• High

Causes

• Genetic disorders • Medication toxicity • Autoimmune

disorders (eg, Sjogren syndrome, rheumatoid arthritis)

Fanconi syndrome (glucosuria, phosphaturia, aminoaciduria)

• Obstructive uropathy

• Congenital adrenal hyperplasia

IP% I 11.111,riall J. ,,... v

Page 124: toddlers, infants, newborns

2

▪ 3

▪ 4-

5 ▪ 6

▪ 7

▪ 8 ▪ 9

• 10

• 11

• 12

• 13

• 14

• 15

• 15

• 17

• 18

• 19

• 20

• 21

• 22

23

• 24

• 25

• 25

• 27

• 28

• 29

• 30

• 31

Item: 23 of 31 i'Mark Previous Next

Lab Values. Notes Calculator.

iii uiiia~uiu LI II I

1

Ouworld.00m

This infant has evidence of a normal anion gap acidosis and failure to thrive. In the absence of an anion gap, a renal or gastrointestinal etiology of the acidosis is most likely. This child has no diarrhea but does have markedly alkalotic urine. These findings are suggestive of renal tubular acidosis (RTA). RTA is caused by a defect in the ability of the renal tubules to reabsorb bicarbonate (type 2 RTA) or excrete hydrogen (type 1 RTA)_ Type 1 RTA is often a genetic disorder and is commonly associated with nephrolithiasis. Type 2 RTA may be isolated but is more commonly a component of Fanconi syndrome (glucosuria, aminoaciduria, and phosphaturia are also present). Type 4 RTA is caused by a defect in the sodium/potassium exchange in the distal tubule, which results in hyperkalemic, hyperchloremic metabolic acidosis. In children, obstructive uropathy and aldosterone insufficiency are common causes.

All types of RTA can present as growth failure (due to poor cellular growth and division in acidic conditions)_ Screening laboratory results will show a low serum bicarbonate level and hyperchloremia, which lead to a normal anion gap metabolic acidosis_ Evaluation of urine pH and urine electrolytes can help distinguish between the types of RTA. Given the markedly alkalotic urine in this patient, type 1 RTA is the most likely diagnosis. Treatment consists of oral sodium bicarbonate to normalize the serum bicarbonate levels.

(Choice A) Cystic fibrosis can present with failure to thrive due to malabsorption, chronic diarrhea, and frequent sinopulmonary infections. This diagnosis is unlikely given the lack of gastrointestinal and pulmonary symptoms.

(Choice B) Gastroesophageal reflux is common in infants and can cause failure to thrive if severe. However, this infant does not have a history of spitting up or vomiting.

(Choice C) This infant is taking 36 ounces of formula a day, which is above the normal caloric intake for an infant (up to 32 ounces a day).

(Choice 0) Lactic acidosis causes a high anion gap metabolic acidosis.

Educational objective: Renal tubular acidosis is caused by a defect in either hydrogen excretion or bicarbonate resorption in the kidney. In infancy, it most commonly presents with failure to thrive due to a chronic, normal anion gap metabolic acidosis. Treatment consists of oral bicarbonate replacement

Copyright © LIWorld

Last updated: [1G/31/2014] 1

Feedback End Block

Page 125: toddlers, infants, newborns

An 18-month-old boy is brought to the emergency department an hour after drinking liquid oven cleaner from an unlocked kitchen cabinet His parents tried to give him water and milk, but he has difficulty swallowing. The boy also has blood-tinged oral secretions. His vital signs are stable. Examination shows an anxious child who is crying and drooling. His lips and chin are swollen and erythematous. He has no stridor and his breathing pattern appears normal. Lungs are clear to auscultation. His shirt is covered in oven cleaner. Which of the following is the best next step in management of this patient?

A. Barium swallow study [1%]

▪ B. Clothing removal [62%]

C. Intravenous corticosteroids [2%]

D. Nasogastric feeding tube [11.6]

E. Nasogastric lavage [17%]

Neutralization with vinegar [1%]

G. Upper gastrointestinal endoscopy [15%]

Item: 24 of 31

F' Mark -4(1

Previous

Next

Lab Values. Notes

Calculator.

Explanation:

Clinical features

Caustic ingestion

Chemical burn or liquefaction necrosis resulting in:

• Laryngeal damage: Hoarseness, stridor • Esophageal damage: Dysphagia, odynophagia • Gastric damage: Epigastric pain, bleeding

Management

• Secure airway, breathing, circulation

• Decontamination: Remove contaminated clothing & visible chemicals; irrigate exposed skin

se

2

▪ 3

▪ 4-

▪ 5

▪ 6

▪ 7

▪ 8 ▪ 9

• 10

• 11

• 12

• 13

• 14

• 15

• 16

• 17

• 18

• 19

• 20

• 21

• 22

• 23

& 24

• 25

• 25

• 27

• 28

• 29

• 30

• 31

Feedback End Block

Page 126: toddlers, infants, newborns

Lab Values. Calculator. Notes Previous . Next r.Mark Item: 24 of 31

Caustic ingestion

Clinical features

Chemical burn or liquefaction necrosis resulting in:

• Laryngeal damage: Hoarseness, stridor • Esophageal damage: Dysphagia, odynophagia • Gastric damage: Epigastric pain, bleeding

Management

• Secure airway, breathing, circulation

• Decontamination: Remove contaminated clothing & visible chemicals; irrigate exposed skin

• Chest x-ray if respiratory symptoms

• Endoscopy within 24 hours

Complications

• Upper airway compromise • Perforation • Stricturesistenosis (2-3 weeks) • Ulcers • Cancer

©LISMLEVirorld,LLC

Infants and toddlers are at high risk for foreign body ingestion as much of their normal behavior involves putting objects in their mouths. Parents should be vigilant about locking up hazardous substances (eg, cleaning supplies) as most accidental ingestions occur at home.

This patient is experiencing orofacial inflammation and dysphagia after ingesting a caustic alkaline substance. After assessing his airway, breathing, and circulation, the best next step is to remove contaminated clothing, as persistent exposure to the caustic agent can cause ongoing damage_ Patients should be hospitalized under close supervision for any developing airway compromise_ Upper gastrointestinal endoscopy (Choice G) is recommended within 24 hours to assess the extent of the injury. The extent of the injury may not be apparent if performed immediately, and delayed endoscopy increases perforation risk_

1 2

▪ 3

▪ 4-

▪ 5

▪ 6

▪ 7

▪ 8

▪ 9

• 10

• 11

• 12

• 13

• 14

• 15

• 15

• 17

• 18

• 19

• 20

• 21

• 22

• 23

& 24

• 25

• 25

• 27

• 28

• 29

• 30

• 31

se

Feedback End Block

Page 127: toddlers, infants, newborns

1

2

▪ 3

▪ 4-

5 ▪ 6

▪ 7

▪ 8

▪ 9

• 10

• 11

• 12

• 13

• 14

• 15

• 15

• 17

• 18

• 19

• 20

• 21

• 22

• 23

& 24

• 25

• 25

• 27

• 28

• 29

• 30

• 31

A

Item: 24 of 31 Al II - Mark Previous

Next

Lab Values. Notes

Calculator. puLui UUJCUL3 II I LI III 1 1 IUULI 13. I 01 CI IL3 31 ILIUIU UC Vly11011L CIUUUL IU4.R1114y, up I UUU3 3UU31.0111.C3 1,C y,

cleaning supplies) as most accidental ingestions occur at home.

This patient is experiencing orofacial inflammation and dysphagia after ingesting a caustic alkaline substance. After assessing his airway, breathing, and circulation, the best next step is to remove contaminated clothing, as persistent exposure to the caustic agent can cause ongoing damage_ Patients should be hospitalized under close supervision for any developing airway compromise_ Upper gastrointestinal endoscopy (Choice G) is recommended within 24 hours to assess the extent of the injury. The extent of the injury may not be apparent if performed immediately, and delayed endoscopy increases perforation risk_

(Choice A) Barium swallow studies are usually not helpful in determining the extent of injury in the acute setting_ All patients with persistent dysphagia or significant esophageal burns on endoscopy should undergo barium contrast studies 2-3 weeks after ingestion to assess for esophageal strictures or pyloric stenosis.

(Choice C) Steroids are not recommended given the lack of proven efficacy in preventing strictures and potential increased chance of perforation.

(Choices ID, E, and F) Any intervention that could provoke vomiting should be avoided. This includes administration of milk, water, activated charcoal, vinegar, or nasogastric lavage, as vomiting can increase the extent of injury. In addition, vinegar combined with an alkaline substance can cause an exothermic reaction and burn the mucosa, exacerbating the existing injury. During endoscopy, a nasogastric feeding tube can be placed under direct visualization; it should not be placed blindly due to the risk of perforation.

Educational objective: The first step in managing caustic ingestions is assessing airway, breathing, and circulation. Contaminated clothing should be removed promptly. Upper gastrointestinal endoscopy is the diagnostic study of choice to evaluate the extent of injury. Attempting to neutralize the alkali with vinegar or lavage is dangerous as these interventions may trigger vomiting, which may cause further mucosal damage.

References:

1. Caustic injury of the upper gastrointestinal tract: a comprehensive review.

2. Caustic ingestion in children.

3. Updates in pediatric gastrointestinal foreign bodies.

Copyright @ UWorld Last updated: [11/24/2014]

Feedback End Block

Page 128: toddlers, infants, newborns

2

▪ 3

▪ 4-

▪ 5

▪ 6

▪ 7

▪ 8 ▪ 9

• 10

• 11

• 12

• 13

• 14

• 15

• 15

• 17

• 18

• 19

• 20

• 21

• 22

• 23

• 24

25

• 26

• 27

• 28

• 29

• 30

• 31

A 1-year-old African American girl is brought to the office for routine evaluation. She is learning how to walk and says "mama" and "dada." She recently transitioned from breast milk to cow's milk. Review of systems is negative. Both parents have sickle cell trait and her sister has sickle cell disease. Her height and weight are appropriate for her age_ Physical examination shows a well-appearing, well-nourished child with no abnormalities. Laboratory results are as follows:

Complete blood count Hemoglobin 14 g/dL Hematocrit 42% Mean corpuscular volume 88 fL Reticulocytes 2%

Hemoglobin electrophoresis Hemoglobin A 60% Hemoglobin S 40% Hemoglobin F 0%

What is the most common complication of her condition?

Item: 25 of 31 V-Mark Previous Next

Lab Values. Notes Calculator.

A. Acute chest syndrome [5%]

B. IDactylitis [14%]

C. Hematuria [46%]

ID_ lschemic stroke [1%]

E Osteomyelitis [2%]

F. Splenic infarction [27%]

G. Urinary tract infection [5%]

Explanation:

Hemoglobin electrophoresis patterns

Feedback End Block

Page 129: toddlers, infants, newborns

▪ 1

• 2

• 3

• 4

. 5

• 6

. 7

• 8

• 9

• 10

• 11

• 12

• 13

• 14

• 15

• 16

• 17

• 18

• 19

• 20

• 21

• 22

• 23

• 24

25 • 26

• 27

• 28

• 29

• 30

• 31

Item: 25 of 31

r. Mark Previous

Next

Lab Values. Notes

Calculator.

Hemoglobin electrophoresis patterns

Diagnosis I Hemoglobin A

,

Hemoglobin S Hemoglobin F

Normal —99%

.

0%

i

<1%

Sickle cell disease

.

0%

.

85-95% 5-15%

Sickle cell trait 50-60%

i.

35-45% <2%

USMLEWorkl, LLC

Sickle cell disease is a hemoglobinopathy with an autosomal recessive inheritance. When both parents carry the trait, their children are at risk for inheriting the disease_ This patient's hemoglobin electrophoresis pattern and normal complete blood count are consistent with being a carrier for the sickle cell trait

Individuals with sickle cell trait are generally asymptomatic and can lead a healthy life_ Although patients are at increased risk for renal issues, the most common of these is painless microscopic or gross hematuria that results from sickling in the renal medulla_ Isosthenuria (impairment in concentrating ability) is also common and can present as nocturia and polyuria. Less commonly, there may be an increased risk of urinary tract infections, particularly during pregnancy (Choice G). Splenic infarctions (Choice F) are uncommon but can occur at high altitudes.

(Choices A, B, D, and E) Other complications of sickle cell disease are unlikely to occur with sickle cell trait.

Educational objective: Most patients with sickle cell trait lead normal, healthy lives. Painless hematuria is the most common complication_

References:

1 Comnlications associated with sickle cell trait: a brief narrative review.

se

Feedback End Block

Page 130: toddlers, infants, newborns

▪ 1

• 2 ▪ 3

• 4-

• 5

• 6

. 7

• 8

. 9

• 10

• 11

• 12

• 13

• 14

• 15

• 16

• 17

• 18

• 19

• 20

• 21

• 22

• 23

• 24

25

• 26

• 27

• 28

• 29

• 30

• 31

of 31 r rldlark Previous . Next

Normal —99%

.

0% .

<1%

Sickle cell disease 0% 85-95% 5-15%

Sic_kle cell trait 50-60% 35-45% <2%

UP. Lab Values, Notes

Calculator.

LEMLEWorld, LLC

Sickle cell disease is a hemoglobinopathy with an autosomal recessive inheritance. When both parents carry the trait, their children are at risk for inheriting the disease_ This patient's hemoglobin electrophoresis pattern and normal complete blood count are consistent with being a carrier for the sickle cell trait

Individuals with sickle cell trait are generally asymptomatic and can lead a healthy life. Although patients are at increased risk for renal issues, the most common of these is painless microscopic or gross hematuria that results from sickling in the renal medulla_ Isosthenuria (impairment in concentrating ability) is also common and can present as nocturia and polyuria. Less commonly, there may be an increased risk of urinary tract infections, particularly during pregnancy (Choice G). Splenic infarctions (Choice F) are uncommon but can occur at high altitudes.

(Choices A, B, D, and E) Other complications of sickle cell disease are unlikely to occur with sickle cell trait.

Educational objective: Most patients with sickle cell trait lead normal, healthy lives. Painless hematuria is the most common complication_

References:

1. Complications associated with sickle cell trait: a brief narrative review.

2. Renal abnormalities in sickle cell disease.

Copyright © LIWorld

Last updated: [11/17/2014]

Feedback End Block

Page 131: toddlers, infants, newborns

2

▪ 3

▪ 4-

▪ 5

▪ 6

▪ 7

▪ 8

▪ 9

• 10

• 11

• 12

• 13

• 14

• 15

• 16

• 17

• 18

• 19

• 20

• 21

• 22

• 23

• 24

• 25

1

A 3-year-old boy is brought to the physician for help with toilet training. He recently started day care and screams "no" when teachers try to place him on the toilet He has bowel movements every other day and strains when he passes hard, pellet-like stools. The boy is a picky eater but loves milk and drinks up to 30 oz of chocolate milk daily. He has no medical problems and takes no medications. His weight and height have been tracking along the 75th percentile. Examination shows a cooperative, well-nourished boy. He runs well and can climb onto the examination table independently. He speaks in short sentences that are mostly understandable_ The boy's abdomen is soft. nontender, and nondistended_ He has normal Tanner I male genitalia. A small fissure is noted on the anal verge. Which of the following is the best next step in management of this patient?

A. Abdominal x-ray [6%]

B. Anorectal manometry [5%]

C. IDisimpaction with rectal enema [10%]

D. Increase juice intake [17%]

E Oral laxative therapy [60%]

F. Rectal biopsy [2%]

G. Thyroid function testing [1%]

Item: 26 of 31 F' Mark Previous Next

Lab Values. Notes Calculator.

• 26

• 27

• 28

• 29

• 30

• 31

Explanation:

Risk factors

Constipation in children

▪ Initiation of solid food a cow's milk ▪ Toilet training ▪ School entry

Clinical presentation

▪ Straining with passage of hard stools ▪ Cramp} abdominal pain ▪ defecationsiweek se

Feedback End Block

Page 132: toddlers, infants, newborns

of 31 r Flilark Previous Next

Constipation in children

Risk factors Initiation of solid food & cow's milk

- Toilet training School entry

Clinical presentation

- Straining with passage of hard stools Crampy abdominal pain

. defecationsMeek

Complications

Anal fissures Hemorrhoids

- Encopresis Enuresis/urinary tract infections Vomiting

Treatment

Increase dietary fiber - Limit cow's milk intake to <24 oz

Laxative +/- Suppositories, enema

CD 4J 5M LEWol cl, LLC

.4111°. Lab Values. Notes Calculator.

Toddlers are at risk for constipation due to multiple transitional events, including dietary changes, toilet training, and school initiation_ The straining, hard stools, and anal fissures in this patient are characteristic of constipation and likely exacerbated by excessive milk consumption, toilet training, and adjusting to day care_

Although constipation is common, prevention and treatment are important as straining and painful defecation can be very stressful_ Children may avoid defecation, which creates a vicious cycle of further accumulation of hard stool, increased pain, and persistent stool withholding. Severe fecal impaction can mimic intestinal obstruction and cause abdominal Damn and vomitino. If the rectum dilates Drooressivelv_ the internal anal se

1 2

▪ 3

▪ 4-

▪ 5

▪ 6

▪ 7

▪ 8

▪ 9

• 10

• 11

• 12

• 13

• 14

• 15

• 16

• 17

• 18

• 19

• 20

• 21

• 22

• 23

• 24

• 25

• 26

• 27

• 28

• 29

• 30

• 31

Feedback End Block

Page 133: toddlers, infants, newborns

1 2

▪ 3

▪ 4-

5 ▪ 6

▪ 7

▪ 8 ▪ 9

• 10

• 11

• 12

• 13

• 14

• 15

• 16

• 17

• 18

• 19

• 20

• 21

• 22

• 23

• 24

• 25

• 26

• 27

• 28

• 29

• 30

• 31

Item: 26 of 31 VI/lark Previous Next

Lab Values. Notes Calculator.

characteristic of constipation and likely exacerbated by excessive milk consumption, toilet training, and adjusting to day care.

Although constipation is common, prevention and treatment are important as straining and painful defecation can be very stressful. Children may avoid defecation, which creates a vicious cycle of further accumulation of hard stool, increased pain, and persistent stool withholding. Severe fecal impaction can mimic intestinal obstruction and cause abdominal pain and vomiting. If the rectum dilates progressively, the internal anal sphincter may relax in response to the increasing pressure, resulting in encopresis (fecal incontinence). The stool burden also decreases bladder capacity and can contribute to enuresis (urinary incontinence).

Children should increase water intake, drink <24 oz of cow's milk, eat fiber-rich foods, and sit on the toilet after each meal. Oral laxatives (eg, polyethylene glycol, mineral oil) are the preferred treatment and should be administered until the stool is consistently soft.

(Choice A) Constipation is a clinical diagnosis. Abdominal x-rays are not routinely indicated but can be helpful to rule out air-fluid levels and free peritoneal air if constipation is severe enough to cause abdominal pain and vomiting.

(Choices B and F) Hirschsprung disease is diagnosed by rectal biopsy and usually presents in the neonatal period with delayed meconium passage and abdominal distension. Mild disease may manifest later in childhood and is an uncommon cause of chronic constipation. If constipation persists despite standard therapy, screening with anorectal manometry can be considered.

(Choice C) Enemas are helpful in evacuating stool_ However, rectal manipulation may be distressing to children_ An enema can be administered if oral laxatives are not helpful and there has been no bowel movement for several days.

(Choice 0) Sorbitol-containing juices (eg, prune, pear, apple) can increase stool water content Juice is less effective than laxatives, and excessive consumption increases the risk for obesity.

(Choice G) Constipation is rarely the sole manifestation of hypothyroidism. These children usually have other problems (eg, short stature, lethargy, dry skin, brittle hair).

Educational objective: Constipation is a common problem in toddlers due to transition to solid food and cow's milk, toilet training, and school entry. Complications include anal fissures, encopresis, and enuresis. Laxative therapy should be initiated promptly to soften stools_

se

Feedback End Block

Page 134: toddlers, infants, newborns

1 2

▪ 3

▪ 4-

5 ▪ 6

▪ 7

▪ 8

▪ 9

• 10

• 11

• 12

• 13

• 14

• 15

• 16

• 17

• 18

• 19

• 20

• 21

• 22

• 23

• 24

• 25

• 26

• 27

• 28

• 29

• 30

• 31

Item: 26 of 31 V-Mark Previous

Next

Lab Values. Notes

Calculator.

Children should increase water intake, drink <24 oz of cow's milk, eat fiber-rich foods, and sit on the toilet after each meal. Oral laxatives (eg, polyethylene glycol, mineral oil) are the preferred treatment and should be administered until the stool is consistently soft.

(Choice A) Constipation is a clinical diagnosis. Abdominal x-rays are not routinely indicated but can be helpful to rule out air-fluid levels and free peritoneal air if constipation is severe enough to cause abdominal pain and vomiting.

(Choices B and F) Hirschsprung disease is diagnosed by rectal biopsy and usually presents in the neonatal period with delayed meconium passage and abdominal distension. Mild disease may manifest later in childhood and is an uncommon cause of chronic constipation. If constipation persists despite standard therapy, screening with anorectal manometry can be considered.

(Choice C) Enemas are helpful in evacuating stool_ However, rectal manipulation may be distressing to children_ An enema can be administered if oral laxatives are not helpful and there has been no bowel movement for several days.

(Choice ID) Sorbitol-containing juices (eg, prune. pear, apple) can increase stool water content. Juice is less effective than laxatives, and excessive consumption increases the risk for obesity.

(Choice G) Constipation is rarely the sole manifestation of hypothyroidism. These children usually have other problems (eg, short stature, lethargy, dry skin. brittle hairy

Educational objective: Constipation is a common problem in toddlers due to transition to solid food and cow's milk, toilet training, and school entry. Complications include anal fissures, encopresis, and enuresis. Laxative therapy should be initiated promptly to soften stools_

References:

1. Polyethylene glycol: a game-changer laxative for children.

2. Hypothyroidism is a rare cause of isolated constipation.

3. Managing functional constipation in children.

Copyright © LIWorld Last updated: [9/9/2014]

Feedback End Block

Page 135: toddlers, infants, newborns

1 2

▪ 3

▪ 4-

5 ▪ 6

▪ 7

▪ 8

▪ 9

• 10

• 11

• 12

• 13

• 14

• 15

• 16

• 17

• 18

• 19

• 20

• 21

• 22

• 23

• 24

• 25

• 26

• 27

• 28

• 29

• 30

• 31

se

A 12-year-old girl is brought to the clinic due to a 2-month history of headaches. Her headaches last 1 - 2 hours and have no fixed time of occurrence. She denies nausea, vomiting, chills or fevers. She has no other medical problems and takes no medication_ Her family history is significant for hypertension and diabetes. Her blood pressure is 156/90 mm Hg, pulse is 80/min, and respirations are 14/min. Examination shows an alert child in no distress. There is a soft to-and-fro bruit heard at the right costovertebral angle. Which of the following is the most likely cause of her hypertension?

A. Coarctation of aorta [12%]

B. Renal artery atherosclerosis [15%]

C. Pheochromocytoma [10%]

▪ D. Fibromuscular dysplasia [58%]

E Conn's syndrome [4%]

Explanation:

The most common cause of secondary hypertension in children is fibromuscular dysplasia. It is responsible for approximately 20% of all cases of renal hypertension. Aside from children, fibromuscular dysplasia is also generally seen in premenopausal women (or women less than 50 years old). Physical examination reveals a hum or bruit in the costovertebral angle due to well-developed collaterals_ The right renal artery is more affected than the left. Angiography typically shows a "string of beads" pattern to the renal artery.

(Choice A) Coarctation generally presents as upper extremity hypertension with absent or diminished pulses in the lower extremities. The narrowing is usually just distal to the left subclavian artery_ Late systolic murmurs from collaterals may be heard in the back. Chest-x ray may show rib notching from collateral development

(Choice B) Renal artery stenosis due to atherosclerosis is generally seen in older patients (>50 years and common in males). It is often a segmental defect in the proximal renal artery. The stenosis is more common on the left. Bruits may be heard, but are generally unrelated to the renal artery stenosis_

(Choice C) Pheochromocytoma is due to catecholamine-secreting tumors of the adrenals_ It can present with paroxysms of headache, sweating and palpitations. Elevated urinary catecholamines are diagnostic_ Bruits are not heard in patients with this condition_

frhnirtp Fl r,nnn's qvnrirnmp is rii IP to An Alrinstprnnp-sprrptinn AripnnmA nr hilAtprAl ArirpnAl hvnprnlAgio It

Item: 27 of 31

.11V-• Mark Previous

Next

Lab Values. Notes

Calculator.

Feedback End Block

Page 136: toddlers, infants, newborns

1 2

▪ 3

▪ 4-

5 ▪ 6

▪ 7

▪ 8 ▪ 9

• 10

• 11

• 12

• 13

• 14

• 15

• 16

• 17

• 18

• 19

• 20

• 21

• 22

• 23

• 24

• 25

• 26

a 27

• 28

• 29

• 30

• 31

A. Coarctation of aorta [12%]

B. Renal artery atherosclerosis [15%]

C. Pheochromocytoma [10%]

ID. Fibromuscular dysplasia [58%]

E Conn's syndrome [4%]

Explanation:

The most common cause of secondary hypertension in children is fibromuscular dysplasia. It is responsible for approximately 20% of all cases of renal hypertension. Aside from children, fibromuscular dysplasia is also generally seen in premenopausal women (or women less than 50 years old). Physical examination reveals a hum or bruit in the costovertebral angle due to well-developed collaterals_ The right renal artery is more affected than the left. Angiography typically shows a "string of beads" pattern to the renal artery.

(Choice A) Coarctation generally presents as upper extremity hypertension with absent or diminished pulses in the lower extremities. The narrowing is usually just distal to the left subclavian artery_ Late systolic murmurs from collaterals may be heard in the back. Chest-x ray may show rib notching from collateral development.

(Choice B) Renal artery stenosis due to atherosclerosis is generally seen in older patients (>50 years and common in males). It is often a segmental defect in the proximal renal artery_ The stenosis is more common on the left. Bruits may be heard, but are generally unrelated to the renal artery stenosis.

(Choice C) Pheochromocytoma is due to catecholamine-secreting tumors of the adrenals. It can present with paroxysms of headache, sweating and palpitations. Elevated urinary catecholamines are diagnostic_ Bruits are not heard in patients with this condition_

(Choice E) Conn's syndrome is due to an aldosterone-secreting adenoma or bilateral adrenal hyperplasia. It usually presents in older patients as hypokalemia and hypertension_

Educational Objective: Fibromuscular dysplasia can present as new onset hypertension in children_ Bruit or venous hum may be heard at the costovertebral angle_ Angiogram reveals the 'string of beads" sign.

Item: 27 of 31 F' Mark Previous Next

Lab Values. Notes Calculator.

Copyright © UWorld Last updated: [11/9/2014]

Feedback End Block

Page 137: toddlers, infants, newborns

1 2

▪ 3

▪ 4-

5 ▪ 6

▪ 7

▪ 8 ▪ 9

• 10

• 11

• 12

• 13

• 14

• 15

• 16

• 17

• 18

• 19

• 20

• 21

• 22

• 23

• 24

• 25

• 26

• 27

28

• 29

• 30

• 31

se

A 7-year-old boy is brought to the office by his mother because, The is always daydreaming_" Sometimes, he stares for a few seconds and rolls his eyes for unknown reasons. His teacher has noted a recent decline in his school performance. He has no family history of any seizure disorder, and his psychomotor development is normal. His neurological examination is unremarkable. EEG during hyperventilation shows generalized, symmetrical 3-Hz spike-and-wave activity on a normal background. Which of the following medications is the most appropriate to treat the boy's disorder?

A. Phenytoin [11%]

B. Phenobarbital [8%]

C. Valproic acid [73%]

D. Prednisone [2%]

E Gabapentin [5%]

Explanation:

The patient is having absence (petit mal) seizures. His presentation (i_e_, daydreaming episodes, brief staring spells, declining school performance) is classic. Other characteristic features are: absent warning and postictal phases, unresponsiveness during the seizure, and no memory of what just occured after the seizure. When the seizure occurs while the patient is engaged in gross motor activity such as walking, he may stop and stand motionless, or continue to walk. Generally, patients are unaware that a seizure has occured. The classic EEG pattern is generalized, symmetrical 3-Hz spike-and-wave activity on a normal background (this finding is provoked by hyperventilation of the patient during EEG). Absence seizures usually occur in children aged 4-8 years. The preferred treatment is ethosuximide or valproic acid.

(Choices 0) Prednisone can be used for treatment of infantile spasms and (Choice E) gabapentin is effective for partial seizures.

(Choice A) Phenytoin is currently the initial drug of choice for partial seizures_

(Choice B) Phenobarbital is not used for the treatment of absence seizures. It is used as an alternative agent in primary generalized and partial seizures.

Educational objective: Suspect absence seizures in a 4- to 8-year-old child with frequent daydreaming episodes, brief staring spells,

Item: 28 of 31 V-Mark Previous Next

Lab Values. Notes Calculator.

Feedback End Block

Page 138: toddlers, infants, newborns

▪ 1

• 2

• 3

• 4

. 5

• 6

. 7

• 8

• 9

• 10

• 11

• 12

• 13

• 14

• 15

• 16

• 17

• 18

• 19

• 20

• 21

• 22

• 23

• 24

• 25

• 26

• 27

28

• 29

• 30

• 31

Item: 28 of 31

V-Mark Previous

Next

Lab Values. Notes

Calculator.

A. Phenytoin [11%]

B. Phenobarbital [8%]

▪ C. Valproic acid [13%]

ID. Prednisone [2%]

E Gabapentin [5%]

Explanation:

The patient is having absence (petit mal) seizures. His presentation (i.e., daydreaming episodes, brief staring spells, declining school performance) is classic. Other characteristic features are: absent warning and postictal phases, unresponsiveness during the seizure, and no memory of what just occured after the seizure. When the seizure occurs while the patient is engaged in gross motor activity such as walking, he may stop and stand motionless, or continue to walk. Generally, patients are unaware that a seizure has occured. The classic EEG pattern is generalized, symmetrical 3-Hz spike-and-wave activity on a normal background (this finding is provoked by hyperventilation of the patient during EEG). Absence seizures usually occur in children aged 4-8 years. The preferred treatment is ethosuximide or valproic acid.

(Choices ID) Prednisone can be used for treatment of infantile spasms and (Choice E) gabapentin is effective for partial seizures.

(Choice A) Phenytoin is currently the initial drug of choice for partial seizures.

(Choice B) Phenobarbital is not used for the treatment of absence seizures. It is used as an alternative agent in primary generalized and partial seizures.

Educational objective: Suspect absence seizures in a 4- to 8-year-old child with frequent daydreaming episodes, brief staring spells, decline in school performance, and no post-ictal phase. The preferred treatment is ethosuximide or valproate.

References:

1. Ethasuximide, valproic acid, and lamotrigine in childhood absence epilepsy.

Copyright © LIWorld

Last updated: [12/29/2014]

Feedback End Block

Page 139: toddlers, infants, newborns

1 2

▪ 3

▪ 4-

5 ▪ 6

▪ 7

▪ 8 ▪ 9

• 10

• 11

• 12

• 13

• 14

• 15

• 16

• 17

• 18

• 19

• 20

• 21

• 22

• 23

• 24

• 25

• 26

• 27

• 28

• 29

- 30

- 31

se

While making rounds in the newborn nursery, the nurses ask you to examine a 2-day-old infant who is not feeding well. The nurse reports that the infant is irritable, is not sleeping well, and has had several episodes of vomiting and loose stools today. A review of the maternal history reveals that she had poor prenatal care and the pregnancy was complicated by intrauterine growth restriction. On examination, the infant is diaphoretic and has a high-pitched cry_ The infant is also noted to have occasional sneezing and is mildly tachypneic. No dysmorphic features are noted and the remainder of the examination is unremarkable. This infant's symptoms are most likely caused by prenatal exposure to which of the following?

A. Valproic acid [2%]

B. Phenytoin [3%]

C. Alcohol [6%]

ID. Cocaine [26%]

• E Heroin [6196]

Explanation:

Although most drugs of abuse cause an addiction in both the mother and the fetus, opiates have the most dramatic effects on the fetus. The opiates that infants are most commonly exposed to are heroin and methadone. Methadone is prescribed for pregnant women who are addicted to heroin to prevent uncontrolled withdrawal in the fetus. Heroin and methadone are not associated with dysmorphic features or congenital anomalies in the fetus, but prenatal exposure can lead to an increased risk of intrauterine growth retardation, macrocephaly, sudden infant death syndrome, and neonatal abstinence syndrome (NAS).

NAB usually presents in the first few days of life and is characterized by irritability, a high-pitched cry, poor sleeping, tremors, seizures, sweating, sneezing, tachypnea, poor feeding, vomiting, and diarrhea Withdrawal usually presents within 48 hours after birth for heroin withdrawal and between 48 and 72 hours for methadone withdrawal, although it can be delayed up to four weeks. The treatment for NA S includes symptomatic care to calm the infant and help the infant sleep, such as swaddling, providing small frequent feeds, and keeping the infant in a low stimulation environment Pharmacologic treatment should be used when supportive treatment does not control the infant's withdrawal symptoms. Morphine can be administered and systematically weaned to help control opiate withdrawal.

C..+,1 r 1,...,11,-11 +I ikr.. r roN,1111-

Item: 29 of 31 V-• Mark Previous Next

Lab Values. Notes Calculator.

Feedback End Block

Page 140: toddlers, infants, newborns

1 2

▪ 3

▪ 4-

5 ▪ 6

▪ 7

▪ 8 ▪ 9

• 10

• 11

• 12

• 13

• 14

• 15

• 16

• 17

• 18

• 19

• 20

• 21

• 22

• 23

• 24

• 25

• 26

• 27

• 28

• 29

- 30

- 31

Although most drugs of abuse cause an addiction in both the mother and the fetus, opiates have the most dramatic effects on the fetus. The opiates that infants are most commonly exposed to are heroin and methadone. Methadone is prescribed for pregnant women who are addicted to heroin to prevent uncontrolled withdrawal in the fetus. Heroin and methadone are not associated with dysmorphic features or congenital anomalies in the fetus, but prenatal exposure can lead to an increased risk of intrauterine growth retardation, macrocephaly, sudden infant death syndrome, and neonatal abstinence syndrome (NAS).

NA S usually presents in the first few days of life and is characterized by irritability, a high-pitched cry, poor sleeping, tremors, seizures, sweating, sneezing, tachypnea, poor feeding, vomiting, and diarrhea Withdrawal usually presents within 48 hours after birth for heroin withdrawal and between 48 and 72 hours for methadone withdrawal, although it can be delayed up to four weeks. The treatment for NA S includes symptomatic care to calm the infant and help the infant sleep, such as swaddling, providing small frequent feeds, and keeping the infant in a low stimulation environment. Pharmacologic treatment should be used when supportive treatment does not control the infant's withdrawal symptoms. Morphine can be administered and systematically weaned to help control opiate withdrawal.

(Choice A) Fetal exposure to valproic acid significantly increases the risk of neural tube defects, may result in cardiac anomalies, and can produce dysmorphic facies, including a cleft lip, narrow bifrontal diameter, midface hypoplasia, broad and depressed nasal bridge, and long philtrum.

(Choice B) Prenatal exposure to phenytoin can result in fetal hydantoin syndrome, which is characterized by nail and digit hypoplasia, dysmorphic facies, and mental retardation. Growth deficiency can also be seen.

(Choice C) Fetal alcohol syndrome consists of growth deficiency, presence of a long, smooth philtrum, small palpebral fissures, and a thin upper lip in addition to central nervous system abnormalities such as mental retardation. Withdrawal symptoms are usually not seen unless the mother was drinking just prior to delivery.

(Choice 0) Prenatal exposure to cocaine can result in jitteriness, excessive sucking, and a hyperactive Moro reflex. Withdrawal symptoms are usually not as severe as with opiates. Long-term effects on behavior, attention level, and intelligence may be seen.

Educational objective: Neonatal abstinence syndrome is caused by infant withdrawal to opiates and usually presents in the first few days of life. It is characterized by irritability, a high-pitched cry, poor sleeping, tremors, seizures, sweating, sneezing, tachypnea, poor feeding, vomiting, and diarrhea

Item: 29 of 31 V-Mark Previous Next

Lab Values. Notes Calculator.

Copyright © LIWorld Last updated: [12/29/2014]

Feedback End Block

Page 141: toddlers, infants, newborns

1 2

▪ 3

▪ 4-

▪ 5 ▪

6 ▪ 7

▪ 8 ▪ 9

• 10

• 11

• 12

• 13

• 14

• 15

• 16

• 17

• 18

• 19

• 20

• 21

• 22

• 23

• 24

• 25

• 26

• 27

28

▪ 29

• 30

▪ 31

se

A 1-year-old boy is brought to the physician by his parents for evaluation of bruising and blood in his stool_ The child has had multiple episodes of otitis media and has been hospitalized twice with pneumonia, but has never had bleeding or easy bruising in the past He has been eating and drinking well with no vomiting or diarrhea. He is afebrile with a heart rate of 150/min and a blood pressure of 80/40 mmHg. On examination, he is well-developed, well-nourished, and has a fair complexion_ There are dry, scaly patches on his cheeks and lower extremities. He also has bruising and purpura on his lower extremities. His diaper contains a small amount of occult blood positive stool, but no fissures are seen on rectal examination. The remainder of the physical examination is unremarkable. Laboratory studies show:

Complete blood count

Hemoglobin 11_5 g/dL Platelet count 20,COGImm' Leukocyte count 6,50eimm3

Neutrophils 76% Eosinophils 1% Lymphocytes 13% Monocytes 10%

A peripheral smear reveals a low number of platelets. and the platelets that are seen are small. Which of the following is the most likely cause of his bleeding?

A. Bone marrow infiltration [7%]

B. Platelet antibodies [34%]

C. Nutritional deficiency [5%]

D. Platelet activation and consumption [13%]

E Impaired platelet production [40%]

Explanation:

This patient has thrombocytopenia associated with skin lesions characteristic of eczema and a history of multiple bacterial infections, which taken together are characteristic of Wiskott-Aldrich syndrome (WAS). Rp.e.Hrrp.nt inferAinns are. usually due. to Strentoryirrois nneurnorliae Neisseria merlineitinris and iLiaemonhilois

Item: 30 of 31 V-Mark Previous Next

Lab Values. Notes Calculator.

Feedback End Block

Page 142: toddlers, infants, newborns

1 2

▪ 3

▪ 4-

▪ 5 ▪

6 ▪ 7

▪ 8 ▪ 9

• 10

• 11

• 12

• 13

• 14

• 15

• 16

• 17

• 18

• 19

• 20

• 21

• 22

• 23

• 24

• 25

• 26

• 27

▪ 28

▪ 29

• 30

▪ 31

Item: 30 of 31 VI/lark Previous

Next

Lab Values. Notes

Calculator.

Explanation:

This patient has thrombocytopenia associated with skin lesions characteristic of eczema and a history of multiple bacterial infections, which taken together are characteristic of Wiskott-Aldrich syndrome (WAS). Recurrent infections are usually due to Streptococcus pneurnoniae, Neissena rneningitiths, and Haemophilus influenzae. Eczema occurs in about half of all patients with WAS, and secondary skin infections are common. Despite these two features, however, thrombocytopenia is the most common manifestation of WAS, as over 90 percent of patients with WAS have significantly decreased platelet levels at the time of diagnosis. The clinical effects of thrombocytopenia can vary from minor petechiae or purpura to potentially life-threatening bleeding such as intracranial hemorrhage, hematemesis, or hematochezia. Platelet counts in patients with WAS are typically less than 50,000/mm' and the mean platelet volume is also usually reduced, both features that are seen in this patient. Decreased platelet production is the primary pathophysiologic cause of thrombocytopenia in patients with WAS.

(Choice A) Childhood leukemias can cause anemia, leukopenia, and thrombocytopenia by infiltrating the bone marrow_ An abnormal leukocyte differential with a high percentage of blast cells would be seen on the laboratory studies in children with leukemia_

(Choice B) Idiopathic thrombocytopenic purpura (ITP) is an immune-mediated disorder caused by antibodies that bind to platelets and facilitate their removal from the circulation. Affected children typically present with easy bruisability or bleeding, but would not have the associated findings of previous bacterial infections and eczema seen in this patient

(Choice C) Folate and vitamin 912 deficiencies can both impair bone marrow production and lead to thrombocytopenia. However, recurrent infections and eczema are not seen with these conditions.

(Choice 0) Thrombocytopenia as a result of increased activation, aggregation, and consumption can occur in patients with disseminated intravascular coagulation (DID), thrombotic thrombocytopenic purpura (TTP), or hemolytic uremic syndrome (HLIS).

Educational objective: Wiskott-Aldrich syndrome is an X-linked disorder characterized by the triad of thrombocytopenia, eczema, and recurrent bacterial infections_ The thrombocytopenia is caused by decreased platelet production, and the few platelets that exist are typically quite small.

Copyright © LIWorld Last updated: [12/29/2014]

Feedback End Block

Page 143: toddlers, infants, newborns

• 30

i 31

2

▪ 3

▪ 4-

5 ▪ 6

▪ 7

▪ 8 ▪ 9

• 10

• 11

• 12

• 13

• 14

• 15

• 15

• 17

• 18

• 19

• 20

• 21

• 22

• 23

• 24

• 25

• 25

• 27

• 28

• 29

1

A 2-year-old girl is brought to the clinic due to fever, irritability and lethargy for the past two weeks. Over the past two months, she has complained of intermittent abdominal discomfort and has lost weight Abdominal palpation reveals a firm nodular mass in the right flank. No bruits are heard. Abdominal x-ray reveals multiple calcifications in the renal area. Urine examination reveals increased levels of homovanillic acid and vanillylmandelic acid_ Which embryonic structure has this mass most likely arisen from?

A. Metanephros [7%]

B. Mesonephron [5%]

C. Paramesonephron [3%]

▪ ID. Neural crest cells [85%]

E Lymphoid stem cells [0%]

Explanation:

This patient has a neuroblastoma (NBL), which is the most common extracranial solid tumor of childhood. The median age at diagnosis is 2 years. The tumor arises from neural crest cells, which are also the precursor cells of the sympathetic chains and adrenal medulla. For this reason, NBL may arise from the adrenal gland or any location along the paravertebral sympathetic chains. The most common site involved is the abdomen, either from the adrenals or retroperitoneal ganglia. The mass is usually firm and nodular in consistency. Calcifications and hemorrhages are seen on plain x-ray and CT scan. Up to 70 percent of patients have metastatic disease at the time of initial presentation, and the most common metastatic sites are the long bones, skull, bone marrow, liver, lymph nodes and skin. The levels of serum and urine catecholamines and their metabolites (i_e_, HVA and VMA) are usually elevated; however, patients do not present with fainting spells, sweating, palpitations and hypertension, as in pheochromocytoma. The prognosis depends on clinical factors, tumor histology and genetic characteristics (amplification of N-myc proto-oncogene and hyperdiploidy).

Item: 31 of 31 F' Mark Previous Next

Lab Values. Notes Calculator.

(Choice A) Wilms' tumor arises from metanephros, which is the embryologic precursor of the renal parenchyma.

(Choice B) Mesonephros is the embryologic structure that gives rise to the seminal vesicles, epididymis, ejaculatory ducts and ductus deferens.

(Choice C) Paramesonephron gives rise to the fallopian tubes, uterus and part of the vagina. se

Feedback End Block

Page 144: toddlers, infants, newborns

• 30

i 31

1 2

▪ 3

▪ 4-

5 ▪ 6

▪ 7

▪ 8

▪ 9

• 10

• 11

• 12

• 13

• 14

• 15

• 15

• 17

• 18

• 19

• 20

• 21

• 22

• 23

• 24

• 25

• 25

• 27

• 28

• 29

Explanation:

This patient has a neuroblastoma (NEL), which is the most common extracranial solid tumor of childhood. The median age at diagnosis is 2 years. The tumor arises from neural crest cells, which are also the precursor cells of the sympathetic chains and adrenal medulla. For this reason, NEL may arise from the adrenal gland or any location along the paravertebral sympathetic chains. The most common site involved is the abdomen, either from the adrenals or retroperitoneal ganglia. The mass is usually firm and nodular in consistency. Calcifications and hemorrhages are seen on plain x-ray and CT scan. Up to 70 percent of patients have metastatic disease at the time of initial presentation, and the most common metastatic sites are the long bones, skull, bone marrow, liver, lymph nodes and skin. The levels of serum and urine catecholamines and their metabolites (i_e_, HVA and VMA) are usually elevated; however, patients do not present with fainting spells, sweating, palpitations and hypertension, as in pheochromocytoma. The prognosis depends on clinical factors, tumor histology and genetic characteristics (amplification of N-myc proto-oncogene and hyperdiploidy).

(Choice A) Wilms' tumor arises from metanephros, which is the embryologic precursor of the renal parenchyma.

(Choice B) Mesonephros is the embryologic structure that gives rise to the seminal vesicles. epididymis. ejaculatory ducts and ductus deferens.

(Choice C) Paramesonephron gives rise to the fallopian tubes, uterus and part of the vagina.

(Choice E) Lymphoid stem cells are at the origin of all lymphocytic leukemias.

Educational Objective: Neuroblastoma is the third most common cancer in the pediatric population (after leukemia and CNS tumors)_ The tumor arises from neural crest cells, which are also the precursor cells of the sympathetic chains and adrenal medulla. The most common site is the abdomen. Calcifications and hemorrhages are seen on plain x-ray and CT scan. The levels of serum and urine catecholamines and their metabolites (i.e., HVA and VMA) are usually elevated.

Item: 31 of 31 V-Mark Previous Next

Lab Values. Notes Calculator.

• D. Neural crest cells [85%]

E Lymphoid stem cells [09i]

Copyright © UWorld Last updated: [8/22/2014]

Feedback End Block